and 20 20.1 Introduction to Aldehydes and Ketones DID YOU EVER WONDER... 20.2 Nomenclature why beta-carotene, which makes carrots 20.3 Preparing Aldehydes and orange, is reportedly good for your eyes? Ketones: A Review 20.4 Introduction to Nucleophilic Addition Reactions his chapter will explore the reactivity of aldehydes and ketones. 20.5 Oxygen Nucleophiles TSpecifically, we will see that a wide variety of nucleophiles will 20.6 Nitrogen Nucleophiles react with aldehydes and ketones. Many of these reactions are com- 20.7 Mechanism Strategies mon in biological pathways, including the role that beta-carotene 20.8 Sulfur Nucleophiles plays in promoting healthy vision. As we will see several times in 20.9 Hydrogen Nucleophiles this chapter, the reactions of aldehydes and ketones are also cleverly 20.10 Carbon Nucleophiles exploited in the design of drugs. The reactions and principles out- 20.11 Baeyer-Villiger Oxidation of lined in this chapter are central to the study of organic chemistry and Aldehydes and Ketones will be used as guiding principles throughout the remaining chapters 20.12 Synthesis Strategies of this textbook. 20.13 Spectroscopic Analysis of Aldehydes and Ketones

klein_c20_001-056v1.4.indd 1 30/04/10 16.46 2 CHAPTER 20 Aldehydes and Ketones

DO YOU REMEMBER? Before you go on, be sure you understand the following topics. If necessary, review the suggested sections to prepare for this chapter: UÊ Àˆ}˜>À`ÊÀi>}i˜ÌÃÊ­-iV̈œ˜Ê£Î°È®ÊÊ UÊ ,iÌÀœÃÞ˜Ì ïVÊ>˜>ÞÈÃÊ­-iV̈œ˜Ê£Ó°Î® UÊ "݈`>̈œ˜ÊœvÊ>Vœ œÃÊ­-iV̈œ˜Ê£Î°£ä® 6ˆÃˆÌÊÜÜܰ܈iÞ«ÕðVœ“Ê̜ÊV iVŽÊޜÕÀÊ՘`iÀÃÌ>˜`ˆ˜}Ê>˜`ÊvœÀÊÛ>Õ>LiÊ«À>V̈Vi°

20.1 Introduction to Aldehydes and Ketones

Aldehydes (RCHO) and ketones (R2CO) are similar in structure in that both classes of com- pounds possess a C5O bond, called a carbonyl group:

Carbonyl Group O O

RH RR An A

The carbonyl group of an aldehyde is flanked by one carbon atom and one hydrogen atom, while the carbonyl group of a ketone is flanked by two carbon atoms. Aldehydes and ketones are responsible for many flavors and odors that you will readily recognize: H CO O O 3 O O H H HO H

Vanillin Cinnamaldehyde (R)-Carvone Benzaldehyde (Vanilla flavor) (Cinnamon flavor) (Spearmint flavor) (Almond flavor) Many important biological compounds also exhibit the carbonyl moiety, including progester- one and testosterone, the female and male sex hormones.

O OH

H H

H H H H O O Progesterone Testosterone Simple aldehydes and ketones are industrially important; for example: O O

HH H3C CH3 Formaldehyde Acetone Acetone is used as a solvent and is commonly found in nail polish remover, while formaldehyde is used as a preservative in some vaccine formulations. Aldehydes and ketones are also used as build- ing blocks in the syntheses of commercially important compounds, including pharmaceuticals and polymers. Compounds containing a carbonyl group react with a large variety of nucleophiles, affording a wide range of possible products. Due to the versatile reactivity of the carbonyl group, aldehydes and ketones occupy a central role in organic chemistry.

klein_c20_001-056v1.4.indd 2 30/04/10 16.46 20.2 Nomenclature 3

20.2 Nomenclature

Nomenclature of Aldehydes Recall that four discrete steps are required to name most classes of organic compounds (as we saw with alkanes, alkenes, , and alcohols): 1. Identify and name the parent. 2. Identify and name the . 3. Assign a locant to each . 4. Assemble the substituents alphabetically. Aldehydes are also named using the same four-step procedure. When applying this procedure for naming aldehydes, the following guidelines should be followed: When naming the parent, the suffix “-al” indicates the presence of an aldehyde group:

O

H Butane Butanal

When choosing the parent of an aldehyde, identify the longest chain that includes the carbon atom of the aldehydic group:

The parent must include HO this carbon atom

Parent=Octane Parent=Hexanal

When numbering the parent chain of an aldehyde, the aldehydic carbon is assigned num- ber 1, despite the presence of alkyl substituents, U bonds, or hydroxyl groups:

Correct Incorrect

24 6 64 2 H 135 7 H 753 1

O OH O OH

It is not necessary to include the locant in the name, because it is understood that the aldehydic carbon is the number 1 position. As with all compounds, when a chirality center is present, the configuration is indicated at the beginning of the name; for example:

O

H Cl

(R)-2-chloro-3-phenylpropanal A cyclic compound containing an aldehyde group immediately adjacent to the ring is named as a carbaldehyde:

O

H

Cyclohexanecarbaldehyde

klein_c20_001-056v1.4.indd 3 30/04/10 16.46 4 CHAPTER 20 Aldehydes and Ketones

The International Union of Pure and Applied Chemistry (IUPAC) nomenclature also recognizes the common names of many simple aldehydes, including the three examples shown below: O O O

HH H3C H H Formaldehyde Acetaldehyde Benzaldehyde Nomenclature of Ketones Ketones, like aldehydes, are named using the same four-step procedure. When naming the par- ent, the suffix “-one” indicates the presence of a ketone group: O

Butane Butanone The position of the ketone group is indicated using a locant. The IUPAC rules published in 1979 dictate that this locant be placed immediately before the parent, while the IUPAC recom- mendations released in 1993 and 2004 allow for the locant to be placed immediately before the suffix “-one”: O 3-heptanone or 1357heptan-3-one 2 4 6 Both names above are acceptable IUPAC names. IUPAC nomenclature recognizes the common names of many simple ketones, including the three examples shown below: O O O CH3

H3CCH3

Acetone Acetophenone Benzophenone Although rarely used, IUPAC rules also allow simple ketones to be named as alkyl alkyl ketones. For example, 3-hexanone can also be called ethyl propyl ketone: O

C

Ethyl propyl ketone

SKILLBUILDER 20.1 NAMING ALDEHYDES AND KETONES

LEARN the skill *ÀœÛˆ`iÊ>ÊÃÞÃÌi“>̈VÊ­1* ®Ê˜>“iÊvœÀÊÌ iÊvœœÜˆ˜}ÊVœ“«œÕ˜`\

O SOLUTION 8 79 STEP 1 / iÊwÀÃÌÊÃÌi«ÊˆÃÊ̜ʈ`i˜ÌˆvÞÊ>˜`ʘ>“iÊÌ iÊ«>Ài˜Ì°Ê œœÃiÊÌ iʏœ˜- 2 46 `i˜ÌˆvÞÊ>˜`ʘ>“iÊÌ iÊ }iÃÌÊV >ˆ˜ÊÌ >Ìʈ˜VÕ`iÃÊÌ iÊV>ÀLœ˜ÞÊ}ÀœÕ«]Ê>˜`ÊÌ i˜Ê˜Õ“LiÀÊÌ iÊ 1 «>Ài˜Ì° V >ˆ˜Ê̜Ê}ˆÛiÊÌ iÊV>ÀLœ˜ÞÊ}ÀœÕ«ÊÌ iʏœÜiÃÌʘՓLiÀÊ«œÃÈLi\ 3 5 O 3-nonanone

klein_c20_001-056v1.4.indd 4 30/04/10 16.46 20.2 Nomenclature 5

iÝÌ]ʈ`i˜ÌˆvÞÊÌ iÊÃÕLÃ̈ÌÕi˜ÌÃÊ>˜`Ê>ÃÈ}˜ÊœV>˜ÌÃ\ STEP 2 4,4-dimethyl 8 `i˜ÌˆvÞÊ>˜`ʘ>“iÊÌ iÊ 79 ÃÕLÃ̈ÌÕi˜Ìð STEP 3 2 46 1 ÃÈ}˜Ê>ʏœV>˜ÌÊ̜Êi>V Ê 3 5 ÃÕLÃ̈ÌÕi˜Ì° 6-ethyl O ˆ˜>Þ]Ê>ÃÃi“LiÊÌ iÊÃÕLÃ̈ÌÕi˜ÌÃÊ>« >LïV>Þ\ÊȇiÌ Þ‡{]{‡`ˆ“iÌ Þ‡Î‡˜œ˜>˜œ˜i°Ê ivœÀiÊ Vœ˜VÕ`ˆ˜}]ÊÜiʓÕÃÌÊ>Ü>ÞÃÊV iVŽÊ̜ÊÃiiʈvÊÌ iÀiÊ>ÀiÊ>˜ÞÊV ˆÀ>ˆÌÞÊVi˜ÌiÀðÊ/ ˆÃÊVœ“«œÕ˜`Ê `œiÃÊiÝ ˆLˆÌʜ˜iÊV ˆÀ>ˆÌÞÊVi˜ÌiÀ°Ê1Ș}ÊÌ iÊΈÃÊvÀœ“Ê-iV̈œ˜Êx°Î]ÊÌ iÊRÊVœ˜w}ÕÀ>̈œ˜ÊˆÃÊ STEP 4 ÃÃi“LiÊÌ iÊ >ÃÈ}˜i`Ê̜ÊÌ ˆÃÊV ˆÀ>ˆÌÞÊVi˜ÌiÀ\ ÃÕLÃ̈ÌÕi˜ÌÃÊ >« >LïV>Þ° STEP 5 ÃÈ}˜ÊÌ iÊVœ˜w}ÕÀ>̈œ˜Ê R œvÊ>˜ÞÊV ˆÀ>ˆÌÞÊVi˜ÌiÀð O / iÀivœÀi]ÊÌ iÊVœ“«iÌiʘ>“iʈÃÊ(R)-6-ethyl-4,4-dimethyl-3-nonanone.

PRACTICE the skill 20.1 ÃÈ}˜Ê>ÊÃÞÃÌi“>̈VÊ­1* ®Ê˜>“iÊ̜Êi>V ʜvÊÌ iÊvœœÜˆ˜}ÊVœ“«œÕ˜`Ã\

O O H

Br Br (a) O (b) (c) O OH H

(d) (e)

APPLY the skill 20.2 À>ÜÊÌ iÊÃÌÀÕVÌÕÀiʜvÊi>V ʜvÊÌ iÊvœœÜˆ˜}ÊVœ“«œÕ˜`Ã\ ­>® ­S®‡Î]·`ˆLÀœ“œ‡{‡iÌ ÞVÞVœ iÝ>˜œ˜iÊ Ê ­L®Ê Ó]{‡`ˆ“iÌ Þ‡Î‡«i˜Ì>˜œ˜i ­V® ­R®‡Î‡LÀœ“œLÕÌ>˜>

20.3 *ÀœÛˆ`iÊ>ÊÃÞÃÌi“>̈VÊ­1* ®Ê˜>“iÊvœÀÊÌ iÊVœ“«œÕ˜`ÊLiœÜ°Ê iÊV>ÀivՏ\Ê/ ˆÃÊVœ“- «œÕ˜`Ê >ÃÊÌܜÊV ˆÀ>ˆÌÞÊVi˜ÌiÀÃÊ­V>˜ÊޜÕÊw˜`ÊÌ i“¶®°

O

20.4 œ“«œÕ˜`ÃÊÜˆÌ ÊÌܜÊV>ÀLœ˜ÞÊ“œˆïiÃÊ>Àiʘ>“i`Ê>ÃÊ>Ž>˜iÊ`ˆœ˜iÃÆÊvœÀÊiÝ>“«i\ O O

2,3-butanedione / iÊVœ“«œÕ˜`Ê>LœÛiʈÃÊ>˜Ê>À̈wVˆ>Êy>ۜÀÊ>``i`Ê̜ʓˆVÀœÜ>ÛiÊ«œ«VœÀ˜Ê>˜`ʓœÛˆi‡Ê Ì i>ÌiÀÊ«œ«VœÀ˜Ê̜ÊȓՏ>ÌiÊÌ iÊLÕÌÌiÀÊy>ۜÀ°Ê˜ÌiÀiÃ̈˜}Þ]ÊÌ ˆÃÊÛiÀÞÊÃ>“iÊVœ“«œÕ˜`ʈÃÊ >ÃœÊŽ˜œÜ˜Ê̜ÊVœ˜ÌÀˆLÕÌiÊ̜ÊLœ`Þʜ`œÀ°Ê >“iÊÌ iÊvœœÜˆ˜}ÊVœ“«œÕ˜`Ã\

O OO O

(a)(b) (c) O OO

need more PRACTICE? Try Problems 20.44–20.49

klein_c20_001-056v1.4.indd 5 30/04/10 16.46 6 CHAPTER 20 Aldehydes and Ketones

20.3 Preparing Aldehydes and Ketones: A Review

In previous chapters, we have studied a variety of methods for preparing aldehydes and ketones, which are summarized in Tables 20.1 and 20.2, respectively.

TABLE 20.1 A SUMMARY OF ALDEHYDE PREPARATION TABLE 20.2 A SUMMARY OF KETONE PREPARATION METHODS COVERED IN PREVIOUS CHAPTERS METHODS COVERED IN PREVIOUS CHAPTERS

REACTION SECTION REACTION SECTION

"݈`>̈œ˜ÊœvÊ*Àˆ“>ÀÞʏVœ œÃÊ £Î°£äÊ "݈`>̈œ˜ÊœvÊ-iVœ˜`>ÀÞʏVœ œÃÊ £Î°£äÊ Ê OH O Ê OH O Ê Ê PCC Na2Cr2O7 Ê Ê CH Cl H SO ,HO ÊR 2 2 RH ÊRR 2 4 2 RR 7 i˜ÊÌÀi>Ìi`ÊÜˆÌ Ê>ÊÃÌÀœ˜}ʜ݈`ˆâˆ˜}Ê>}i˜Ì]Ê«Àˆ“>ÀÞÊ>Vœ œÃÊ ÊÊÛ>ÀˆiÌÞʜvÊÃÌÀœ˜}ʜÀʓˆ`ʜ݈`ˆâˆ˜}Ê>}i˜ÌÃÊV>˜ÊLiÊÕÃi`ÊÌœÊ >Àiʜ݈`ˆâi`Ê̜ÊV>ÀLœÝޏˆVÊ>Vˆ`ðÊœÀ“>̈œ˜ÊœvÊ>˜Ê>`i Þ`iÊ œÝˆ`ˆâiÊÃiVœ˜`>ÀÞÊ>Vœ œÃ°Ê/ iÊÀiÃՏ̈˜}ʎi̜˜iÊ`œiÃʘœÌÊ ÀiµÕˆÀiÃÊ>˜ÊœÝˆ`ˆâˆ˜}Ê>}i˜Ì]ÊÃÕV Ê>ÃÊ* ]ÊÌ >ÌÊ܈Ê˜œÌÊvÕÀÌ iÀÊ Õ˜`iÀ}œÊvÕÀÌ iÀʜ݈`>̈œ˜° œÝˆ`ˆâiÊÌ iÊÀiÃՏ̈˜}Ê>`i Þ`i° "✘œÞÈÃʜvʏŽi˜iÃÊ ™°£™Ê "✘œÞÈÃʜvʏŽi˜iÃÊ ™°£™Ê ÊR R R R Ê Ê 1) O3 H H H H O O Ê 1) O Ê 2) DMS Ê 3 O O ÊR R R R 2) DMS ÊR R R R Ê/iÌÀ>ÃÕLÃ̈ÌÕÌi`Ê>Ži˜iÃÊ>ÀiÊVi>Ûi`Ê̜ÊvœÀ“ÊŽi̜˜ið Ê Ê"✘œÞÈÃÊ܈ÊVi>ÛiÊ>Ê 5 Ê`œÕLiÊLœ˜`°ÊvÊiˆÌ iÀÊV>ÀLœ˜Ê Vˆ`‡ >Ì>Þâi`ÊÞ`À>̈œ˜ÊœvÊ/iÀ“ˆ˜>ÊŽÞ˜iÃÊ £ä°nÊ >̜“ÊLi>ÀÃÊ>Ê Þ`Àœ}i˜Ê>̜“]Ê>˜Ê>`i Þ`iÊ܈ÊLiÊvœÀ“i`° Ê O Ê H2SO4,H2O Þ`ÀœLœÀ>̈œ˜‡"݈`>̈œ˜ÊœvÊ/iÀ“ˆ˜>ÊŽÞ˜iÃÊ £ä°nÊ Ê HgSO4 R CH Ê ÊR 3 1) R BH H Ê 2 R Ê/ ˆÃÊ«ÀœVi`ÕÀiÊÀiÃՏÌÃʈ˜Ê>Ê >ÀŽœÛ˜ˆŽœÛÊ>``ˆÌˆœ˜ÊœvÊÜ>ÌiÀÊ ÊR 2) H2O2, NaOH >VÀœÃÃÊÌ iÊUÊLœ˜`]ÊvœœÜi`ÊLÞÊÌ>Õ̜“iÀˆâ>̈œ˜Ê̜ÊvœÀ“Ê>Ê Ê O “iÌ ÞÊŽi̜˜i° ÊÞ`ÀœLœÀ>̈œ˜‡œÝˆ`>̈œ˜ÊÀiÃՏÌÃʈ˜Ê>˜Ê>˜Ìˆ‡ >ÀŽœÛ˜ˆŽœÛÊ>``ˆ- ̈œ˜ÊœvÊÜ>ÌiÀÊ>VÀœÃÃÊ>ÊUÊLœ˜`]ÊvœœÜi`ÊLÞÊÌ>Õ̜“iÀˆâ>̈œ˜ÊœvÊ Àˆi`i‡ À>vÌÃÊVޏ>̈œ˜Ê £™°ÈÊ Ì iÊÀiÃՏ̈˜}Êi˜œÊ̜ÊvœÀ“Ê>˜Ê>`i Þ`i°Ê Ê O Ê O Ê R Ê Cl R Ê AlCl3 Ê Àœ“>̈VÊÀˆ˜}ÃÊÌ >ÌÊ>ÀiʘœÌÊ̜œÊÃÌÀœ˜}ÞÊ`i>V̈Û>Ìi`Ê܈ÊÀi>VÌÊ ÜˆÌ Ê>˜Ê>Vˆ`Ê >ˆ`iʈ˜ÊÌ iÊ«ÀiÃi˜ViʜvÊ>Êi܈ÃÊ>Vˆ`Ê̜ʫÀœ`ÕViÊ >˜Ê>Àޏʎi̜˜i°

CONCEPTUAL CHECKPOINT

20.5 `i˜ÌˆvÞÊÌ iÊÀi>}i˜ÌÃʘiViÃÃ>ÀÞÊ̜Ê>V ˆiÛiÊi>V ʜvÊÌ iÊvœœÜˆ˜}ÊÌÀ>˜ÃvœÀ“>̈œ˜Ã\

OH O O OH O H

(a) (b) (c) O H O O

O H (d) (e) (f)

klein_c20_001-056v1.4.indd 6 30/04/10 16.46 20.4 Introduction to Nucleophilic Addition Reactions 7

20.4 Introduction to Nucleophilic Addition Reactions

The electrophilicity of a carbonyl group derives from resonance effects as well as inductive effects: Resonance Induction

- d- O O O

+ d±

One of the resonance structures exhibits a positive charge on the carbon atom, indicating that the carbon atom is deficient in electron density (I+). Inductive effects also render the carbon atom deficient in electron density. As a result, this carbon atom is particularly electrophilic and is susceptible to attack by a nucleophile. Molecular orbital calculations suggest that nucleophilic attack occurs at an angle of approximately 107° to the plane of the carbonyl group, and in the process, the hybridization state of the carbon atom changes (Figure 20.1).

107° angle – Nuc

Nuc R – FIGURE 20.1 O ¡ O R 7 i˜Ê>ÊV>ÀLœ˜ÞÊ}ÀœÕ«ÊˆÃÊ R >ÌÌ>VŽi`ÊLÞÊ>ʘÕViœ« ˆi]Ê R Ì iÊV>ÀLœ˜Ê>̜“Ê՘`iÀ}œiÃÊ >ÊV >˜}iʈ˜Ê ÞLÀˆ`ˆâ>̈œ˜Ê>˜`Ê sp 2 sp 3 }iœ“iÌÀÞ° (Trigonal planar) (Tetrahedral )

The carbon atom is originally sp2 hybridized with a trigonal planar geometry. After the attack, the carbon atom is sp3 hybridized with a tetrahedral geometry. In recognition of this geometric change, the resulting alkoxide ion is often called a tetrahedral intermediate. This term appears many times throughout the remainder of this chapter. In general, aldehydes are more reactive than ketones toward nucleophilic attack. This observation can be explained in terms of both steric and electronic effects: 1. Steric effects. A ketone has two alkyl groups (one on either side of the carbonyl) that contribute to steric hindrance in the transition state of a nucleophilic attack. In contrast, an aldehyde has only one alkyl group, so the transition state is less crowded and lower in energy. 2. Electronic effects. Recall that alkyl groups are electron donating. A ketone has two electron- donating alkyl groups that can stabilize the I+ on the carbon atom of the carbonyl group. In contrast, aldehydes have only one electron-donating group:

O O

R d± R R d± H A ketone An aldehyde has two electron-donating alkyl groups has only one electron-donating alkyl group that stabilize the partial positive charge that stabilizes the partial positive charge

The I+ charge of an aldehyde is less stabilized than a ketone. As a result, aldehydes are more electrophilic than ketones and therefore more reactive. Aldehydes and ketones react with a wide variety of nucleophiles. As we will see in the com- ing sections of this chapter, some nucleophiles require basic conditions, while others require acidic conditions. For example, recall from Chapter 13 that Grignard reagents are very strong nucleophiles that will attack aldehydes and ketones to produce alcohols:

O OH 1) MeMgBr

R R 2) H2O R Me R

klein_c20_001-056v1.4.indd 7 30/04/10 16.46 8 CHAPTER 20 Aldehydes and Ketones

The Grignard reagent itself provides for strongly basic conditions, because Grignard reagents are both strong nucleophiles and strong bases. This reaction cannot be achieved under acidic conditions, because, as explained in Section 13.6, Grignard reagents are destroyed in the presence of an acid. The Grignard reaction above follows a general mechanism for the reac- tion between a nucleophile and a carbonyl group under basic conditions (Mechanism 20.1). This general mechanism has two steps: (1) nucleophilic attack followed by (2) proton transfer.

MECHANISM 20.1 NUCLEOPHILIC ADDITION UNDER BASIC CONDITIONS

Nucleophilic attack Proton transfer

- O - O Nuc O H H OH

Nuc Nuc

The carbonyl group is attacked by a The tetrahedral intermediate is protonated nucleophile, forming a tetrahedral intermediate upon treatment with a mild proton source

Aldehydes and ketones also react with a wide variety of other nucleophiles under acidic conditions. In acidic conditions, the same two mechanistic steps are observed, but in reverse order—that is, the carbonyl group is first protonated and then undergoes a nucleophilic attack (Mechanism 20.2).

MECHANISM 20.2 NUCLEOPHILIC ADDITION UNDER ACIDIC CONDITIONS

Proton transfer Nucleophilic attack

+ H - O HA O Nuc OH

Nuc

The carbonyl group is first protonated, The protonated carbonyl group is rendering it even more electrophilic then attacked by a nucleophile

In acidic conditions, the first step plays an important role. Specifically, protonating the carbonyl group generates a very powerful electrophile:

+ H H OOO H¬A · +

Very powerful electrophile It is true that the carbonyl group is already a fairly strong electrophile; however, a protonated carbonyl group bears a full positive charge, rendering the carbon atom even more electrophilic. This is especially important when weak nucleophiles, such as H2O or ROH, are employed, as we will see in the upcoming sections. When a nucleophile attacks a carbonyl group under either acidic or basic conditions, the position of equilibrium is highly dependent on the ability of the nucleophile to function as a leaving group. A Grignard reagent is a very strong nucleophile, but it does not function as a leaving group (a carbanion is too unstable to leave). As a result, the equilibrium so greatly favors products that the reaction effectively occurs in only one direction. With a sufficient amount of nucleophile present, the ketone is not observed in the product mixture. In contrast, halides are

klein_c20_001-056v1.4.indd 8 30/04/10 16.46 20.5 Oxygen Nucleophiles 9

good nucleophiles, but they are also good leaving groups. Therefore, when a halide functions as the nucleophile, the equilibrium actually favors the starting ketone:

O HO Cl + HCl RR RR Once equilibrium has been achieved, the mixture consists primarily of the ketone, and only small quantities of the addition product. In this chapter, we will explore a wide variety of nucleophiles, which will be classified according to the nature of the attacking atom. Specifically, we will see nucleophiles based on oxygen, sulfur, nitrogen, hydrogen, and carbon (Figure 20.2).

Oxygen Sulfur Nitrogen Hydrogen Carbon Nucleophiles Nucleophiles Nucleophiles Nucleophiles Nucleophiles H H RMgBr O S - HH HH N H Al H RH - H C N O RH H SSH H H - Ph + H FIGURE 20.2 N H B H Ph P C - 6>ÀˆœÕÃʘÕViœ« ˆiÃÊÌ >ÌÊV>˜Ê H OOH RR Ph H >ÌÌ>VŽÊ>ÊV>ÀLœ˜ÞÊ}ÀœÕ«° H The remainder of the chapter will be a methodical survey of the reactions that occur between the reagents in Figure 20.2 and ketones and aldehydes. We will begin our survey with oxygen nucleophiles.

CONCEPTUAL CHECKPOINT

20.6 À>ÜÊ>ʓiV >˜ˆÃ“ÊvœÀÊi>V ʜvÊÌ iÊ O HO O Ài>V̈œ˜ÃÊLiœÜ\ HO Cl 1) EtMgBr + HCl 2) H O (a)2 (b)

20.5 Oxygen Nucleophiles

Hydrate Formation When an aldehyde or ketone is treated with water, the carbonyl group can be converted into a hydrate:

O HO OH

+ H2O

Hydrate The position of equilibrium generally favors the carbonyl group rather than the hydrate, except in the case of very simple aldehydes, such as formaldehyde: O HO OH

+ H2O H3C CH3 H3C CH3

99.9%

O HO OH

+ H2O HH HH

> 99.9%

klein_c20_001-056v1.4.indd 9 30/04/10 16.46 10 CHAPTER 20 Aldehydes and Ketones

The rate of reaction is relatively slow under neutral conditions but is readily enhanced in the presence of either acid or base. That is, the reaction can be either acid catalyzed or base cata- lyzed, allowing the equilibrium to be achieved much more rapidly. Consider the base-catalyzed hydration of formaldehyde (Mechanism 20.3).

MECHANISM 20.3 BASE-CATALYZED HYDRATION

Nucleophilic attack Proton transfer

- - O O OH O H H OH

HH H OH H OH H H The carbonyl group is attacked by The tetrahedral intermediate is protonated hydroxide, forming a tetrahedral intermediate by water to form the hydrate

In the first step, a hydroxide ion (rather than water) functions as a nucleophile. Then, in the second step, the tetrahedral intermediate is protonated with water, regenerating a hydroxide ion. In this way, hydroxide serves as a catalyst for the addition of water across the carbonyl group. Now consider the acid-catalyzed hydration of formaldehyde (Mechanism 20.4).

MECHANISM 20.4 ACID-CATALYZED HYDRATION

Proton transfer Nucleophilic attack Proton transfer

H H H + + H O O O H O O OH OH H H H H HH HH H O+ H OH H H H The carbonyl group The protonated carbonyl The tetrahedral intermediate is protonated, rendering group is attacked by water, is deprotonated by water it more electrophilic forming a tetrahedral intermediate to form the hydrate

Under acid-catalyzed conditions, the carbonyl group is first protonated, generating a positively charged intermediate that is extremely electrophilic (it bears a full positive charge). This inter- mediate is then attacked by water to form a tetrahedral intermediate, which is deprotonated to give the product.

CONCEPTUAL CHECKPOINT

20.7 œÀʓœÃÌʎi̜˜iÃ]Ê Þ`À>ÌiÊvœÀ“>̈œ˜ÊˆÃÊ՘v>ۜÀ>Li]ÊLiV>ÕÃiÊ O HO OH Ì iÊiµÕˆˆLÀˆÕ“Êv>ۜÀÃÊÌ iʎi̜˜iÊÀ>Ì iÀÊÌ >˜ÊÌ iÊ Þ`À>Ìi°ÊœÜiÛiÀ]Ê + H2O Ì iÊiµÕˆˆLÀˆÕ“ÊvœÀÊ Þ`À>̈œ˜ÊœvÊ iÝ>y՜Àœ>Vi̜˜iÊv>ۜÀÃÊvœÀ“>̈œ˜Ê F3C CF3 F3C CF3 œvÊÌ iÊ Þ`À>Ìi\Ê*ÀœÛˆ`iÊ>Ê«>ÕÈLiÊiÝ«>˜>̈œ˜ÊvœÀÊÌ ˆÃʜLÃiÀÛ>̈œ˜° > 99.99%

klein_c20_001-056v1.4.indd 10 30/04/10 16.46 20.5 Oxygen Nucleophiles 11

Acetal Formation The previous section discussed a reaction that can occur when water attacks an aldehyde or ketone. This section will explore a similar reaction, in which an alcohol attacks an aldehyde or ketone: O [H+] RO OR ++2 ROH H2O BY THE WAY 7 i˜ÊÌ iÊÃÌ>À̈˜}ÊVœ“«œÕ˜`Ê Acetal ˆÃÊ>ʎi̜˜i]ÊÌ iÊ«Àœ`ÕVÌÊV>˜Ê In acidic conditions, an aldehyde or ketone will react with two of alcohol to form an >ÃœÊLiÊV>i`Ê>ʺŽiÌ>°»Ê acetal. The brackets surrounding the H+ indicate that the acid is a catalyst. Common acids used AcetalʈÃÊ>ʓœÀiÊ}i˜iÀ>Ê for this purpose include para-toluenesulfonic acid (TsOH) and sulfuric acid (H2SO4): ÌiÀ“]Ê>˜`ʈÌÊ܈ÊLiÊÕÃi`Ê iÝVÕÈÛiÞÊvœÀÊÌ iÊÀi“>ˆ˜`iÀÊ O O œvÊÌ ˆÃÊ`ˆÃVÕÃȜ˜° SOH HO SOH O O

p-toluenesulfonic acid Sulfuric acid (TsOH) As mentioned earlier, the acid catalyst serves an important role in this reaction. Specifically, in the presence of an acid, the carbonyl group is protonated, rendering the carbon atom even more electrophilic. This is necessary because the nucleophile (an alcohol) is weak; it reacts with the carbonyl group more rapidly if the carbonyl group is first protonated. A mechanism for acetal formation is shown in Mechanism 20.5. This mechanism has many steps, and it is best to divide it conceptually into two parts: (1) The first three steps produce an intermediate called a hemiacetal and (2) the last four steps convert the hemiacetal into an acetal:

MECHANISM 20.5 ACETAL FORMATION

Proton transfer Nucleophilic attack Proton transfer

H + H O O + O OH OH HA R A H O+ OR The carbonyl group The alcohol attacks the The tetrahedral R Hemiacetal is protonated, rendering protonated carbonyl intermediate is Proton transfer it more electrophilic to generate a deprotonated to tetrahedral intermediate form a hemiacetal + HA The OH group is protonated, thereby converting it into an excellent leaving group H + H O

OR Loss of a leaving group

Proton transfer Nucleophilic attack –H2O Water leaves to regenerate the + H C“O double bond H R O + R OR O O A R

OR OR

Acetal The intermediate is The second of the deprotonated, alcohol attacks the C“O generating an acetal double bond to generate another tetrahedral intermediate

klein_c20_001-056v1.4.indd 11 30/04/10 16.46 12 CHAPTER 20 Aldehydes and Ketones

Let’s begin our analysis of this mechanism by focusing on the first part: formation of the hemi- acetal, which involves the three steps in Figure 20.3.

FIGURE 20.3 / iÊÃiµÕi˜ViʜvÊÃÌi«Ãʈ˜ÛœÛi`Ê Proton transfer Nucleophilic attack Proton transfer ˆ˜ÊvœÀ“>̈œ˜ÊœvÊ>Ê i“ˆ>ViÌ>°

Notice that the sequence of steps begins and ends with a proton transfer. This will be a recurring pattern in this chapter. Let’s focus on the details of these three steps:

H 1. The carbonyl is protonated in the presence of an acid. The + + identity of the acid, HA+, is most likely a protonated alco- H A H O hol, which received its extra proton from the acid catalyst: R 2. The protonated carbonyl is a very powerful electrophile and is attacked by a molecule of alcohol (ROH) to form a tetrahedral intermediate that bears a positive charge. 3. The tetrahedral intermediate is deprotonated by a weak base (A), which is likely to be a molecule of alcohol present in solution. Notice that the acid is not consumed in this process. A proton is used in step 1 and then returned in step 3, confirming the catalytic nature of the proton in the reaction. It is important to remember the specific order of these three steps, as we will soon encounter many other reactions that begin with the same three steps. These three steps are typical of reac- tions involving acid-catalyzed nucleophilic attack. Now let’s focus on the second part of the mechanism, conversion of the hemiacetal into an acetal, which is accomplished with the four steps in Figure 20.4.

FIGURE 20.4 / iÊÃiµÕi˜ViʜvÊÃÌi«ÃÊÌ >ÌÊ Proton Loss of a Nucleophilic Proton Vœ˜ÛiÀÌÊ>Ê i“ˆ>ViÌ>Êˆ˜ÌœÊ>˜Ê transfer leaving group attack transfer >ViÌ>° Notice, once again, that the sequence of steps begins and ends with a proton transfer. A proton is used in the first step and then returned in the last step, but this time there are two middle steps rather than just one. When drawing the mechanism of acetal formation, make sure to draw these two steps separately. Combining these two steps is incorrect and represents one of the most common student errors when drawing this mechanism:

Loss of a + leaving OH2 group An SN2 process + ROH ¡ cannot occur OR at this substrate

Nucleophilic attack

These two steps cannot occur simultaneously, because that would represent an SN2 process occurring at a sterically hindered substrate. Such a process is disfavored and does not occur at an appreciable rate. Instead, the leaving group leaves first to form a resonance-stabilized intermedi- ate, which is then attacked by the nucleophile in a separate step. The equilibrium arrows in the full mechanism of acetal formation indicate that the process is governed by an equilibrium. For many simple aldehydes, the equilibrium favors formation of the acetal, so aldehydes are readily converted into acetals by treatment with two equivalents of alcohol in acidic conditions:

O EtO OEt [H+] + + 2 EtOH H2O HH HH Products are favored at equilibrium

klein_c20_001-056v1.4.indd 12 30/04/10 16.46 20.5 Oxygen Nucleophiles 13

BY THE WAY However, for most ketones, the equilibrium favors reactants rather than products: / ˆÃÊÌiV ˜ˆµÕiÊiÝ«œˆÌÃÊiÊ O EtO OEt @ÌiˆiÀ½ÃÊ«Àˆ˜Vˆ«i]ÊÜ ˆV Ê [H+] + + Ü>ÃÊVœÛiÀi`ʈ˜ÊޜÕÀÊ}i˜iÀ>Ê 2 EtOH H2O V i“ˆÃÌÀÞÊVœÕÀÃi°ÊVVœÀ`ˆ˜}Ê H3CCH3 H3CCH3 ̜ÊiÊ @ÌiˆiÀ½ÃÊ«Àˆ˜Vˆ«i]Ê Reactants are favored ˆvÊ>ÊÃÞÃÌi“Ê>ÌÊiµÕˆˆLÀˆÕ“ʈÃÊ at equilibrium Õ«ÃiÌÊLÞÊܓiÊ`ˆÃÌÕÀL>˜Vi]Ê In such cases, formation of the acetal can be accomplished by removing one of the products Ì iÊÃÞÃÌi“Ê܈ÊV >˜}iʈ˜Ê>Ê (water) via a special distillation technique. By removing water as it is formed, the reaction can Ü>ÞÊÌ >ÌÊÀiÃ̜ÀiÃÊiµÕˆˆLÀˆÕ“° be forced to completion. Notice that acetal formation requires two equivalents of the alcohol. That is, two molecules of ROH are required for every molecule of ketone. Alternatively, a compound containing two OH groups can be used, forming a cyclic acetal. This reaction proceeds via the regular seven-step mechanism for acetal formation: three steps for formation of the hemiacetal followed by four steps for formation of the cyclic acetal:

OH OH OH O 3 steps O OH 4 steps O O + H2O

Hemiacetal Cyclic acetal The seven-step mechanism for acetal formation is very similar to other mechanisms that we will explore. It is therefore critical to master these seven steps. To help you draw the mechanism properly, remember to divide the entire mechanism into two parts, where each part begins and ends with proton transfers. Let’s get some practice.

SKILLBUILDER 20.2 DRAWING THE MECHANISM OF ACETAL FORMATION

O EtO OEt

LEARN the skill À>ÜÊ >Ê «>ÕÈLiÊ “iV >˜ˆÃ“Ê vœÀÊ Ì iÊ vœœÜˆ˜}Ê [H2SO4] ÌÀ>˜ÃvœÀ“>̈œ˜\ excess EtOH –H2O SOLUTION / iÊÀi>V̈œ˜Ê>LœÛiʈÃÊ>˜ÊiÝ>“«iʜvÊ>Vˆ`‡V>Ì>Þâi`Ê>ViÌ>ÊvœÀ“>̈œ˜]ʈ˜ÊÜ ˆV ÊÌ iÊ«Àœ`- ÕVÌÊ ˆÃÊ v>ۜÀi`Ê LÞÊ Ì iÊ Ài“œÛ>Ê œvÊ Ü>ÌiÀ°Ê / iÊ “iV >˜ˆÃ“Ê V>˜Ê LiÊ `ˆÛˆ`i`Ê ˆ˜ÌœÊ ÌÜœÊ «>ÀÌÃ\ÊÊ ­£®ÊvœÀ“>̈œ˜ÊœvÊÌ iÊ i“ˆ>ViÌ>Ê>˜`Ê­Ó®ÊvœÀ“>̈œ˜ÊœvÊÌ iÊ>ViÌ>°ÊœÀ“>̈œ˜ÊœvÊÌ iÊ i“ˆ>ViÌ>Ê STEP 1 ˆ˜ÛœÛiÃÊÌ ÀiiʓiV >˜ˆÃ̈VÊÃÌi«Ã\ À>ÜÊÌ iÊÌ ÀiiÊÃÌi«ÃÊ ˜iViÃÃ>ÀÞÊvœÀÊ i“ˆ>ViÌ>Ê Proton transfer Nucleophilic attack Proton transfer vœÀ“>̈œ˜° 7 i˜Ê`À>܈˜}ÊÌ iÃiÊÌ ÀiiÊÃÌi«Ã]ʓ>ŽiÊÃÕÀiÊ̜ÊvœVÕÃʜ˜Ê«Àœ«iÀÊ>ÀÀœÜÊ«>Vi“i˜ÌÊ­>ÃÊ`iÃVÀˆLi`Ê ˆ˜Ê >«ÌiÀÊÈ®]Ê>˜`ʓ>ŽiÊÃÕÀiÊ̜ʫ>ViÊ>Ê«œÃˆÌˆÛiÊV >À}iÃʈ˜ÊÌ iˆÀÊ>««Àœ«Àˆ>ÌiʏœV>̈œ˜Ã°Ê œÌˆViÊ Ì >ÌÊiÛiÀÞÊÃÌi«ÊÀiµÕˆÀiÃÊÌܜÊVÕÀÛi`Ê>ÀÀœÜð

The tail of this arrow should Don’t forget the be placed on a lone pair positive charges

+ H H O O + H H HO O Et H HO OEt HO+ O O Et Et Et

Don’t forget the second curved arrow Hemiacetal that shows release of the proton

klein_c20_001-056v1.4.indd 13 30/04/10 16.46 14 CHAPTER 20 Aldehydes and Ketones

œÜʏi̽ÃÊvœVÕÃʜ˜ÊÌ iʏ>ÃÌÊvœÕÀÊÃÌi«ÃʜvÊÌ iʓiV >˜ˆÃ“]ʈ˜ÊÜ ˆV ÊÌ iÊ i“ˆ>ViÌ>ÊˆÃÊVœ˜- ÛiÀÌi`ʈ˜ÌœÊ>˜Ê>ViÌ>\ STEP 2 À>ÜÊÌ iÊvœÕÀÊÃÌi«ÃÊ Proton Loss of a Nucleophilic Proton ˜iViÃÃ>ÀÞÊ̜ÊVœ˜ÛiÀÌÊ transfer leaving group attack transfer Ì iÊ i“ˆ>ViÌ>Êˆ˜ÌœÊ>˜Ê >ViÌ>° "˜ViÊ>}>ˆ˜]ÊÌ ˆÃÊÃiµÕi˜ViʜvÊÃÌi«ÃÊLi}ˆ˜ÃÊÜˆÌ Ê>Ê«ÀœÌœ˜ÊÌÀ>˜ÃviÀÊ>˜`Êi˜`ÃÊÜˆÌ Ê>Ê«ÀœÌœ˜Ê ÌÀ>˜ÃviÀ°Ê7 i˜Ê`À>܈˜}ÊÌ iÃiÊvœÕÀÊÃÌi«Ã]ʓ>ŽiÊÃÕÀiÊ̜Ê`À>ÜÊÌ iʓˆ``iÊÌܜÊÃÌi«ÃÊÃi«>À>ÌiÞ]Ê >ÃÊ`ˆÃVÕÃÃi`Êi>ÀˆiÀ°Ê˜Ê>``ˆÌˆœ˜]ʓ>ŽiÊÃÕÀiÊ̜ÊvœVÕÃʜ˜Ê«Àœ«iÀÊ>ÀÀœÜÊ«>Vi“i˜Ì]Ê>˜`ʓ>ŽiÊ ÃÕÀiÊ̜ʫ>ViÊ>Ê«œÃˆÌˆÛiÊV >À}iÃʈ˜ÊÌ iˆÀÊ>««Àœ«Àˆ>ÌiʏœV>̈œ˜Ã\

The tail of this arrow should be placed on a lone pair Don’t forget the positive charges H H + + Et + O HO OEt H H O OEt H EtO O Et H EtO OEt HO+ O O

Et –H2O Et Et

Don’t forget the second Acetal curved arrow that shows release of the proton

PRACTICE the skill 20.8 À>ÜÊ>Ê«>ÕÈLiʓiV >˜ˆÃ“ÊvœÀÊi>V ʜvÊÌ iÊvœœÜˆ˜}ÊÌÀ>˜ÃvœÀ“>̈œ˜Ã\

O MeO OMe OEt O [H2SO4] [TsOH] excess MeOH excess EtOH OEt

–H2O –H2O (a) (b) O O EtO OEt MeO OMe [TsOH] [H SO ] 2 4 excess MeOH excess EtOH

–H2O –H2O (c) (d)

APPLY the skill 20.9 À>ÜÊ>Ê«>ÕÈLiʓiV >˜ˆÃ“ÊvœÀÊi>V ʜvÊÌ iÊvœœÜˆ˜}ÊÀi>V̈œ˜Ã\ HO OH O O O [H2SO4]

–H2O (a)

O HO OH [H2SO4] OO –H2O (b)

20.10 *Ài`ˆVÌÊÌ iÊ«Àœ`ÕVÌʜvÊi>V ʜvÊÌ iÊvœœÜˆ˜}ÊÀi>V̈œ˜Ã\ O O

[H2SO4] HO OH excess MeOH ? [H2SO4] ? –H2O –H2O

(a) (b)

need more PRACTICE? Try Problems 20.57, 20.62, 20.67

klein_c20_001-056v1.4.indd 14 30/04/10 16.46 20.5 Oxygen Nucleophiles 15

Acetals as Protecting Groups HO OH Acetal formation is a reversible process that can O + [H ] O O be controlled by carefully choosing reagents and –H2O conditions: As mentioned in the previous section, acetal for- H2O mation is favored by removal of water. To con- + vert an acetal back into the corresponding alde- [H ] hyde or ketone, it is simply treated with water in the presence of an acid catalyst. In this way, acetals can be used to protect ketones or aldehydes. For example, consider how the following transformation might be accomplished: O O O

OR ? OH

This transformation involves reduction of an ester to form an alcohol. Recall that lithium alu- minum hydride (LAH) can be used to accomplish this type of reaction. However, under these conditions, the ketone moiety will also be reduced. The problem above requires reduction of the ester moiety without also reducing the ketone moiety. To accomplish this, a protecting group can be used. The first step is to convert the ketone into an acetal:

O O O O O

OR HO OH OR [H+] –H2O Notice that the ketone moiety is converted into an acetal, but the ester moiety is not. The result- ing acetal group is stable under strongly basic conditions and will not react with LAH. This makes it possible to reduce only the ester, after which the acetal can be removed to regenerate the ketone: The three steps are summarized below: O O O

+ OR 1) [H ], HO OH, –H2O OH 2) LAH + 3) H3O

CONCEPTUAL CHECKPOINT

20.11 *Àœ«œÃiÊ>˜ÊivwVˆi˜ÌÊÃÞ˜Ì iÈÃÊvœÀÊi>V ʜvÊÌ iÊvœ- 20.12 *Ài`ˆVÌÊÌ iÊ«Àœ`ÕV̭îÊvœÀÊi>V ÊÀi>V̈œ˜ÊLiœÜ\ œÜˆ˜}ÊÌÀ>˜ÃvœÀ“>̈œ˜Ã\ O O O O OMe + + H3O H3O (a) OMe (a) ??(b) O O O OH O (c) O Ph Ph + + O O H3O O H3O

O O O OH (c) ??(d)

(b) O H H

klein_c20_001-056v1.4.indd 15 30/04/10 16.47 16 CHAPTER 20 Aldehydes and Ketones

MEDICALLYSPEAKING Acetals as Prodrugs

˜Ê >«ÌiÀÊ£™ÊÜiÊiÝ«œÀi`ÊÌ iÊVœ˜Vi«ÌÊ -Žˆ˜Ê >ÃÊ ÃiÛiÀ>Ê ˆ“«œÀÌ>˜ÌÊ v՘V̈œ˜Ã]Ê ˆ˜VÕ`ˆ˜}Ê «ÀiÛi˜Ìˆ˜}Ê œvÊ «Àœ`ÀÕ}Ãp« >À“>Vœœ}ˆV>ÞÊ ˆ˜>V- Ì iÊ>LÜÀ«Ìˆœ˜ÊœvÊvœÀiˆ}˜ÊÃÕLÃÌ>˜ViÃʈ˜ÌœÊÌ iÊ}i˜iÀ>ÊVˆÀVՏ>̈œ˜°Ê ̈ÛiÊ Vœ“«œÕ˜`ÃÊ Ì >ÌÊ >ÀiÊ Vœ˜ÛiÀÌi`Ê / ˆÃÊvi>ÌÕÀiÊ«ÀœÌiVÌÊÕÃÊvÀœ“Ê >À“vՏÊÃÕLÃÌ>˜ViÃ]ÊLÕÌʈÌÊ>ÃœÊ«Ài- LÞÊ Ì iÊ Lœ`ÞÊ ˆ˜ÌœÊ >V̈ÛiÊ Vœ“«œÕ˜`Ã°Ê Ûi˜ÌÃÊLi˜iwVˆ>Ê`ÀÕ}ÃÊvÀœ“Ê«i˜iÌÀ>̈˜}Ê`ii«Êˆ˜ÌœÊÌ iÊΈ˜°Ê/ ˆÃÊ >˜ÞÊÃÌÀ>Ìi}ˆiÃÊ>ÀiÊÕÃi`ʈ˜ÊÌ iÊ`iÈ}˜Ê ivviVÌʈÃʓœÃÌÊ«Àœ˜œÕ˜Vi`ÊvœÀÊ`ÀÕ}ÃÊVœ˜Ì>ˆ˜ˆ˜}Ê"Ê}ÀœÕ«ÃÊÌ >ÌÊ œvÊ «Àœ`ÀÕ}Ã°Ê "˜iÊ ÃÕV Ê ÃÌÀ>Ìi}ÞÊ ˆ˜ÛœÛiÃÊ V>˜Êˆ˜ÌiÀ>VÌÊÜˆÌ ÊLˆ˜`ˆ˜}ÊÈÌiÃʜ˜ÊÌ iÊΈ˜½ÃÊÃÕÀv>Vi°Ê/œÊVˆÀVՓÛi˜ÌÊ >˜Ê>ViÌ>Ê“œˆiÌÞ° Ì ˆÃÊ«ÀœLi“]ÊÌܜÊ"Ê}ÀœÕ«ÃÊV>˜ÊLiÊÌi“«œÀ>ÀˆÞÊVœ˜ÛiÀÌi`ʈ˜ÌœÊ ÃÊ >˜Ê iÝ>“«i]Ê y՜Vˆ˜œ˜ˆ`iÊ ˆÃÊ >Ê «Àœ- >˜Ê>ViÌ>°Ê/ iÊ>ViÌ>Ê«Àœ`ÀÕ}ʈÃÊV>«>LiʜvÊ«i˜iÌÀ>̈˜}ÊÌ iÊΈ˜Ê `ÀÕ}Ê Ì >ÌÊ Vœ˜Ì>ˆ˜ÃÊ >˜Ê >ViÌ>Ê “œˆiÌÞ]Ê >˜`Ê ˆÃÊ “œÀiÊ `ii«Þ]Ê LiV>ÕÃiÊ ˆÌÊ >VŽÃÊ Ì iÊ "Ê }ÀœÕ«ÃÊ Ì >ÌÊ Lˆ˜`Ê ÌœÊ Ì iÊ Ãœ`ʈ˜Ê>ÊVÀi>“ÊÕÃi`ÊvœÀÊÌ iÊ̜«ˆV>ÊÌÀi>̓i˜ÌÊ ÃŽˆ˜°Ê "˜ViÊ Ì iÊ «Àœ`ÀÕ}Ê Ài>V iÃÊ ˆÌÃÊ Ì>À}iÌ]Ê Ì iÊ >ViÌ>Ê “œˆiÌÞÊ ˆÃÊ œvÊiVâi“>Ê>˜`ÊœÌ iÀÊΈ˜ÊVœ˜`ˆÌˆœ˜Ã°Ê Ϝ܏ÞÊ Þ`ÀœÞâi`]ÊÌ iÀiLÞÊÀii>Ș}ÊÌ iÊ>V̈ÛiÊ`ÀÕ}\

O O + O O O O Acetal O O moiety is OH HO removed HO O H H OH

F H Fluocinonide F H O O Active drug

F F

/Ài>̓i˜ÌÊ ÜˆÌ Ê y՜Vˆ˜œ˜ˆ`iÊ ˆÃÊ Ãˆ}˜ˆwV>˜ÌÞÊ “œÀiÊ ivviV̈ÛiÊ Ì >˜Ê `ˆÀiVÌÊÌÀi>̓i˜ÌÊÜˆÌ ÊÌ iÊ>V̈ÛiÊ`ÀÕ}]ÊLiV>ÕÃiÊÌ iʏ>ÌÌiÀÊV>˜˜œÌÊ Ài>V Ê>ÊœvÊÌ iÊ>vviVÌi`Ê>Ài>ðÊ

Stable Hemiacetals In the previous section, we saw how to convert an aldehyde or ketone into an acetal. In most cases, it is very difficult to isolate the intermediate hemiacetal:

O RO OH RO OR + ∆∆+ + 2 ROH ROH H2O

Hemiacetal Acetal

Favored by Difficult to Favored when the equilibrium isolate water is removed

For ketones we saw that the equilibrium generally favors the reactants unless water is removed, which enables formation of the acetal. The hemiacetal is not favored under either set of condi- tions (with or without removal of water). However, when a compound contains both a carbonyl group and a hydroxyl group, the resulting cyclic hemiacetal can often be isolated; for example:

klein_c20_001-056v1.4.indd 16 30/04/10 16.47 20.6 Nitrogen Nucleophiles 17

OH O O [H+]

HO

Cyclic hemiacetal

This will be important when we learn about carbohydrate chemistry in Chapter 24. Glucose, the major source of energy for the body, exists primarily as a cyclic hemiacetal:

HO OH OH O O HO HO ∆ H HO OH OH OH OH

Glucose Glucose (Open-chain) (Cyclic hemiacetal)

CONCEPTUAL CHECKPOINT O HO O 20.13 À>ÜÊ>Ê«>ÕÈLiʓiV >˜ˆÃ“ÊvœÀÊÌ iÊvœœÜˆ˜}ÊÌÀ>˜ÃvœÀ- [H SO ] “>̈œ˜\ OH 2 4

20.14 œ“«œÕ˜`Ê Ê >ÃÊ “œiVՏ>ÀÊ vœÀ“Տ>Ê H " °Ê 1«œ˜Ê O n £{ Ó HO ÌÀi>̓i˜ÌÊÜˆÌ ÊV>Ì>Þ̈VÊ>Vˆ`]ÊVœ“«œÕ˜`ÊʈÃÊVœ˜ÛiÀÌi`ʈ˜ÌœÊÌ iÊ VÞVˆVÊ i“ˆ>ViÌ>°Ê`i˜ÌˆvÞÊÌ iÊÃÌÀÕVÌÕÀiʜvÊVœ“«œÕ˜`Ê° [H+] Compound A

20.6 Nitrogen Nucleophiles

Primary Amines In mildly acidic conditions, an aldehyde or ketone will react with a primary amine to form an imine:

CH3 O N

H [H+] H

CH3NH2

Imines are compounds that possess a C5N double bond and are common in biological path- ways. Imines are also called Schiff bases, named after Hugo Schiff, a German chemist who first described their formation. A six-step mechanism for imine formation is shown in Mechanism 20.6. It is best to divide the mechanism conceptually into two parts (just as we did to concep- tualize the mechanism of acetal formation): (1) The first three steps produce an intermediate called a carbinolamine and (2) the last three steps convert the carbinolamine into an imine:

klein_c20_001-056v1.4.indd 17 30/04/10 16.47 18 CHAPTER 20 Aldehydes and Ketones

MECHANISM 20.6 IMINE FORMATION

Proton transfer Nucleophilic attack Proton transfer

H H + H N O + O OH OH HA R H A H N N +H The carbonyl group The amine attacks R The tetrahedral R is protonated, the protonated intermediate is rendering it carbonyl to generate deprotonated Carbinolamine more electrophilic a tetrahedral to form a intermediate carbinolamine Proton transfer

+ HA The OH group is protonated, thereby Loss of a leaving converting it into an Proton transfer group excellent leaving group

R H + R H + H N N O A –H2O H N R Imine The intermediate is Water leaves, deprotonated, to forming a generate an imine C“N double bond

Note: There is experimental evidence that the first two steps only rarely occurs), because this sequence enables a more of this mechanism (protonation and nucleophilic attack) more effective comparison of all acid-catalyzed mechanisms in this likely occur either simultaneously or in the reverse order of what chapter and also unifies the rationale behind proton transfers, is shown above. Most nitrogen nucleophiles are sufficiently as we will discuss in Sections 20.6 and 20.7. Interested students nucleophilic to attack a carbonyl group directly, before can learn more from the following literature references: protonation occurs. Nevertheless, the first two steps of the 1. J. Am. Chem. Soc., 1974, 96(26), 7998–09 mechanism above have been drawn in the order shown (which 2. J. Org. Chem., 2007, 72(22), 8202–8215

FIGURE 20.5 / iÊÃiµÕi˜ViʜvÊÃÌi«Ãʈ˜ÛœÛi`Ê Proton transfer Nucleophilic attack Proton transfer ˆ˜ÊvœÀ“>̈œ˜ÊœvÊ>ÊV>ÀLˆ˜œ>“ˆ˜i°

Let’s begin our analysis of this mechanism by focusing on the first part: formation of the car- binolamine, which involves the three steps in Figure 20.5. Notice that these three steps are identical to the first three steps of acetal formation. Specifically, this sequence of steps involves a nucleophilic attack that is sandwiched between proton transfer steps. The identity of the acid, HA+, is most likely a protonated amine, which received its extra proton from the acid source: N + + H A H N H R Once the carbinolamine has been formed, formation of the imine is accomplished with three steps (Figure 20.6). Notice again that this reaction sequence begins and ends with proton transfers.

FIGURE 20.6 / iÊÃiµÕi˜ViʜvÊÃÌi«ÃÊÌ >ÌÊ Vœ˜ÛiÀÌÊ>ÊV>ÀLˆ˜œ>“ˆ˜iʈ˜ÌœÊ>˜Ê Proton transfer Loss of a leaving group Proton transfer ˆ“ˆ˜i°

klein_c20_001-056v1.4.indd 18 30/04/10 16.47 20.6 Nitrogen Nucleophiles 19

The pH of the solution is an important consideration during imine formation, with the rate of reaction being greatest when the pH is around 4.5 (Figure 20.7). If the pH is too high (i.e., if no acid catalyst is used), the carbonyl group is not protonated (step 1 of the mechanism) and the carbinolamine is also not protonated (step 4 of the mechanism); so the reaction occurs more slowly. Rate If the pH is too low (too much acid is used), most of the amine molecules will be protonated:

H 02461573 H O+ H pH H H + FIGURE 20.7 R N RHN / iÊÀ>Ìiʜvʈ“ˆ˜iÊvœÀ“>̈œ˜Ê>ÃÊ>Ê H H v՘V̈œ˜ÊœvÊ«° A nucleophile NOT a nucleophile

Under these conditions, step 2 of the mechanism occurs too slowly. As a result, care must be taken to ensure optimal pH of the solution during imine formation.

SKILLBUILDER 20.3 DRAWING THE MECHANISM OF IMINE FORMATION

LEARN the skill À>ÜÊ>Ê«>ÕÈLiʓiV >˜ˆÃ“ÊvœÀÊÌ iÊvœœÜˆ˜}ÊÌÀ>˜ÃvœÀ“>̈œ˜\

Et O N

[H2SO4] EtNH2 – H2O

SOLUTION / iÊÀi>V̈œ˜Ê>LœÛiʈÃÊ>˜ÊiÝ>“«iʜvʈ“ˆ˜iÊvœÀ“>̈œ˜°Ê/ iʓiV >˜ˆÃ“ÊV>˜ÊLiÊ`ˆÛˆ`i`ʈ˜ÌœÊ Ìܜʫ>ÀÌÃ\Ê­£®ÊvœÀ“>̈œ˜ÊœvÊÌ iÊV>ÀLˆ˜œ>“ˆ˜iÊ>˜`Ê­Ó®ÊvœÀ“>̈œ˜ÊœvÊÌ iʈ“ˆ˜i° œÀ“>̈œ˜ÊœvÊÌ iÊV>ÀLˆ˜œ>“ˆ˜iʈ˜ÛœÛiÃÊÌ ÀiiʓiV >˜ˆÃ̈VÊÃÌi«Ã\

Proton transfer Nucleophilic attack Proton transfer

7 i˜Ê `À>܈˜}Ê Ì iÃiÊ Ì ÀiiÊ ÃÌi«Ã]Ê “>ŽiÊ ÃÕÀiÊ ÌœÊ «>ViÊ Ì iÊ i>`Ê >˜`Ê Ì>ˆÊ œvÊ iÛiÀÞÊ VÕÀÛi`Ê >ÀÀœÜʈ˜ÊˆÌÃÊ«ÀiVˆÃiʏœV>̈œ˜]Ê>˜`ʓ>ŽiÊÃÕÀiÊ̜ʫ>ViÊ>Ê«œÃˆÌˆÛiÊV >À}iÃʈ˜ÊÌ iˆÀÊ>««Àœ«Àˆ>ÌiÊÊ œV>̈œ˜Ã°Ê œÌˆViÊÌ >ÌÊiÛiÀÞÊÃÌi«ÊÀiµÕˆÀiÃÊÌܜÊVÕÀÛi`Ê>ÀÀœÜð

The tail of this arrow should be placed on a lone pair Don’t forget the positive charges STEP 1 H H H + H H + À>ÜÊÌ iÊÌ ÀiiÊÃÌi«ÃÊ O + O H H HO N H H HO N ˜iViÃÃ>ÀÞÊ̜ÊvœÀ“Ê>Ê H N H N Et N Et V>ÀLˆ˜œ>“ˆ˜i° Et Et Et

Don’t forget the second curved arrow Carbinolamine that shows release of the proton

klein_c20_001-056v1.4.indd 19 30/04/10 16.47 20 CHAPTER 20 Aldehydes and Ketones

œÜʏi̽ÃÊvœVÕÃʜ˜ÊÌ iÊÃiVœ˜`Ê«>ÀÌʜvÊÌ iʓiV >˜ˆÃ“]ʈ˜ÊÜ ˆV ÊÌ iÊV>ÀLˆ˜œ>“ˆ˜iʈÃÊ Vœ˜ÛiÀÌi`ʈ˜ÌœÊ>˜Êˆ“ˆ˜i°Ê/ ˆÃÊÀiµÕˆÀiÃÊÌ ÀiiÊÃÌi«Ã\

Proton transfer Loss of a leaving group Proton transfer

"˜ViÊ>}>ˆ˜]ÊÌ ˆÃÊÃiµÕi˜ViʜvÊÃÌi«ÃÊLi}ˆ˜ÃÊÜˆÌ Ê>Ê«ÀœÌœ˜ÊÌÀ>˜ÃviÀÊ>˜`Êi˜`ÃÊÜˆÌ Ê>Ê«ÀœÌœ˜Ê ÌÀ>˜ÃviÀ°Ê >ŽiÊÃÕÀiÊ̜ʫ>ViÊÌ iÊ i>`Ê>˜`ÊÌ>ˆÊœvÊiÛiÀÞÊVÕÀÛi`Ê>ÀÀœÜʈ˜ÊˆÌÃÊ«ÀiVˆÃiʏœV>̈œ˜]Ê >˜`ʓ>ŽiÊÃÕÀiÊ̜ʫ>ViÊ>Ê«œÃˆÌˆÛiÊV >À}iÃʈ˜ÊÌ iˆÀÊ>««Àœ«Àˆ>ÌiʏœV>̈œ˜Ã\

The tail of this arrow Don’t forget the should be placed on a lone pair positive charges STEP 2 À>ÜÊÌ iÊÌ ÀiiÊÃÌi«ÃÊ H H H H + Et Et ˜iViÃÃ>ÀÞÊ̜ÊVœ˜ÛiÀÌÊÌ iÊ H + + N N V>ÀLˆ˜œ>“ˆ˜iʈ˜ÌœÊ>˜Ê HO N O N H H Et H N H H Et N ˆ“ˆ˜i° Et –H2O Et

Don’t forget the second curved arrow Imine that shows release of the proton

PRACTICE the skill 20.15 À>ÜÊ>Ê«>ÕÈLiʓiV >˜ˆÃ“ÊvœÀÊi>V ʜvÊÌ iÊvœœÜˆ˜}ÊÌÀ>˜ÃvœÀ“>̈œ˜Ã\

O N Et O N [TsOH] [TsOH] MeNH2 EtNH2

–H2O –H2O (a) (b)

APPLY the skill 20.16 *Ài`ˆVÌÊÌ iʓ>œÀÊ«Àœ`ÕVÌÊvœÀÊi>V ʜvÊÌ iÊvœœÜˆ˜}ÊÀi>V̈œ˜Ã\ O

[H+] NH2 NH3 –H O O [H+] –H O (a)2 ? (b) 2 ?

20.17 *Ài`ˆVÌÊÌ iʓ>œÀÊ«Àœ`ÕVÌÊvœÀÊi>V ʜvÊÌ iÊvœœÜˆ˜}ʈ˜ÌÀ>“œiVՏ>ÀÊÀi>V̈œ˜Ã\ O O [H+] [H+] NH2 NH2 (a) H –H2O ? (b) –H2O ?

20.18 `i˜ÌˆvÞÊÌ iÊÀi>VÌ>˜ÌÃÊÌ >ÌÊޜÕÊܜՏ`ÊÕÃiÊ̜ʓ>ŽiÊi>V ʜvÊÌ iÊvœœÜˆ˜}ʈ“ˆ˜iÃ\ N N N

(a) (b) (c)

need more PRACTICE? Try Problems 20.72, 20.86

Many different compounds of the form RNH2 will react with aldehydes and ketones, including compounds in which R is not an alkyl group. In the following examples, the R group of the amine has been replaced with a group that has been highlighted in red:

OH NH2 O [H+] N O [H+] N HO–NH2 H2N–NH2 – – R R H2O R R R R H2O R R An oxime A hydrazone

klein_c20_001-056v1.4.indd 20 30/04/10 16.47 20.6 Nitrogen Nucleophiles 21

LOOKING AHEAD When hydroxylamine (NH2OH) is used as a nucleophile, an oxime is formed. When hydrazine Þ`À>✘iÃÊ>ÀiÊÃÞ˜Ì ïV>ÞÊ (NH2NH2) is used as a nucleophile, a hydrazone is formed. The mechanism for each of these ÕÃivՏ]Ê>ÃÊÜiÊ܈ÊÃiiÊ reactions is directly analogous to the mechanism of imine formation. ˆ˜ÊÌ iÊ`ˆÃVÕÃȜ˜ÊœvÊÌ iÊ 7œvv‡ˆÃ ˜iÀÊÀi`ÕV̈œ˜Ê >ÌiÀʈ˜ÊÌ ˆÃÊV >«ÌiÀ°

PRACTICALLYSPEAKING Beta-Carotene and Vision

iÌ>‡V>ÀœÌi˜iʈÃÊ>ʘ>ÌÕÀ>ÞʜVVÕÀÀˆ˜}ÊVœ“«œÕ˜`ÊvœÕ˜`ʈ˜Ê“>˜ÞÊ œÀ>˜}i‡VœœÀi`Ê vÀՈÌÃÊ >˜`Ê Ûi}iÌ>LiÃ]Ê ˆ˜VÕ`ˆ˜}Ê V>ÀÀœÌÃ]Ê ÃÜiiÌÊ «œÌ>̜iÃ]Ê «Õ“«Žˆ˜Ã]Ê “>˜}œiÃ]Ê V>˜Ì>œÕ«iÃ]Ê >˜`Ê >«ÀˆVœÌÃ°Ê ÃÊ “i˜Ìˆœ˜i`ʈ˜ÊÌ iÊV >«ÌiÀʜ«i˜iÀ]ÊLiÌ>‡V>ÀœÌi˜iʈÃʎ˜œÜ˜Ê̜ÊLiÊ }œœ`Ê vœÀÊ ÞœÕÀÊ iÞiÃ°Ê /œÊ ՘`iÀÃÌ>˜`Ê Ü Þ]Ê ÜiÊ “ÕÃÌÊ iÝ«œÀiÊ Ü >ÌÊ >««i˜ÃÊ̜ÊLiÌ>‡V>ÀœÌi˜iʈ˜ÊޜÕÀÊLœ`Þ°Ê“ˆ˜iÊvœÀ“>̈œ˜Ê«>ÞÃÊ>˜Ê ˆ“«œÀÌ>˜ÌÊÀœiʈ˜ÊÌ iÊ«ÀœViÃð

ı-carotene iÌ>‡V>ÀœÌi˜iÊ ˆÃÊ “iÌ>Lœ- ˆâi`ʈ˜ÊÌ iʏˆÛiÀÊ̜ʫÀœ`ÕViÊ ÛˆÌ>“ˆ˜ÊÊ­>ÃœÊV>i`ÊÀ®\ OH Vitamin A (Retinol)

6ˆÌ>“ˆ˜Ê Ê ˆÃÊ Ì i˜Ê œÝˆ`ˆâi`]Ê >˜`ʜ˜iʜvÊÌ iÊ`œÕLiÊLœ˜`ÃÊ OH ՘`iÀ}œiÃÊ ˆÃœ“iÀˆâ>̈œ˜Ê ÌœÊ This bond This group «Àœ`ÕViÊ££‡cis‡Àï˜>\ undergoes is oxidized isomerization 11-cis-retinal H O

/ iÊÀiÃՏ̈˜}Ê>`i Þ`iÊÌ i˜Ê

Ài>VÌÃÊ ÜˆÌ Ê >˜Ê >“ˆ˜œÊ }ÀœÕ«Ê H2N–Protein œvÊ>Ê«ÀœÌiˆ˜Ê­V>i`ʜ«Ãˆ˜®ÊÌœÊ «Àœ`ÕViÊ À œ`œ«Ãˆ˜]Ê Ü ˆV Ê «œÃÃiÃÃiÃÊ>˜Êˆ“ˆ˜iʓœˆiÌÞ\ 11-cis-retinal H O H N Protein

Rhodopsin

ÃÊ`iÃVÀˆLi`ʈ˜Ê-iV̈œ˜Ê£Ç°£Î]ÊÀ œ`œ«Ãˆ˜ÊV>˜Ê>LÜÀLÊ>Ê« œ- Ê `iwVˆi˜VÞÊ œvÊ ÛˆÌ>“ˆ˜Ê Ê V>˜Ê i>`Ê ÌœÊ º˜ˆ} ÌÊ Lˆ˜`˜iÃÃ]»Ê ̜˜Êœvʏˆ} Ì]ʈ˜ˆÌˆ>̈˜}Ê>Ê« œÌœˆÃœ“iÀˆâ>̈œ˜ÊœvÊÌ iÊcisÊ`œÕLiÊLœ˜`Ê >Ê Vœ˜`ˆÌˆœ˜Ê Ì >ÌÊ «ÀiÛi˜ÌÃÊ Ì iÊ iÞiÃÊ vÀœ“Ê >`ÕÃ̈˜}Ê ÌœÊ `ˆ“ÞÊ ˆÌÊÊ ÌœÊvœÀ“Ê>ÊtransÊ`œÕLiÊLœ˜`°Ê/ iÊÀiÃՏ̈˜}ÊV >˜}iʈ˜Ê}iœ“iÌÀÞÊ i˜ÛˆÀœ˜“i˜Ìð ÌÀˆ}}iÀÃÊ>ÊÈ}˜>ÊÌ >ÌʈÃÊՏ̈“>ÌiÞÊ`iÌiVÌi`ÊLÞÊÌ iÊLÀ>ˆ˜Ê>˜`ʈ˜ÌiÀ- «ÀiÌi`Ê>ÃÊۈȜ˜°

klein_c20_001-056v1.4.indd 21 30/04/10 16.47 22 CHAPTER 20 Aldehydes and Ketones

CONCEPTUAL CHECKPOINT

20.19 *Ài`ˆVÌÊÌ iÊ«Àœ`ÕVÌʜvÊi>V ʜvÊÌ iÊvœœÜˆ˜}ÊÀi>V̈œ˜Ã\ 20.20 `i˜ÌˆvÞÊÌ iÊÀi>VÌ>˜ÌÃÊÌ >ÌÊޜÕÊܜՏ`ÊÕÃiÊ̜ʓ>ŽiÊi>V ʜvÊ

+ Ì iÊvœœÜˆ˜}ÊVœ“«œÕ˜`Ã\ O [H ] HO–NH2 N NH2 – OH N (a) H2O ? (a) (b) O [H+]

H2N–NH2 – (b) H2O ?

Secondary Amines RR In acidic conditions, an aldehyde or ketone will react O N with a secondary amine to form an enamine: [H+] Enamines are compounds in which the nitrogen R lone pair is delocalized by the presence of an adjacent N H C5C double bond. A mechanism for enamine forma- R An enamine tion is shown in Mechanism 20.7

MECHANISM 20.7 ENAMINE FORMATION

Proton transfer Nucleophilic attack Proton transfer

H R + H N O O + OH OH H¬A R A H R N N + R R R The carbonyl The amine attacks The tetrahedral Carbinolamine group the protonated intermediate is protonated, carbonyl to is deprotonated rendering it generate to form a Proton transfer more electrophilic a tetrahedral carbinolamine intermediate + H¬A The OH group is protonated thereby converting Loss of a it into an excellent Proton transfer leaving group leaving group

+ R R R + R H H – N A N H2O O H R N The intermediate Water leaves Enamine is deprotonated and a C“N double R to generate an imine bond forms

Note: There is experimental evidence that the first two steps only rarely occurs), because this sequence enables a more of this mechanism (protonation and nucleophilic attack) more effective comparison of all acid-catalyzed mechanisms in this likely occur either simultaneously or in the reverse order of what chapter and also unifies the rationale behind proton transfers, is shown above. Most nitrogen nucleophiles are sufficiently as we will discuss in Sections 20.6 and 20.7. Interested students nucleophilic to attack a carbonyl group directly, before can learn more from the following literature references: protonation occurs. Nevertheless, the first two steps of the 1. J. Am. Chem. Soc., 1974, 96(26), 7998–09 mechanism above have been drawn in the order shown (which 2. J. Org. Chem., 2007, 72(22), 8202–8215

klein_c20_001-056v1.4.indd 22 30/04/10 16.47 20.6 Nitrogen Nucleophiles 23

This mechanism of enamine formation is identical to the mechanism of imine formation except for the last step:

R + H R N N

+ [H ] RNH2 RNH O 2

An imine

R + R R R N N H + [H ] R2NH

R2NH

An enamine

The difference in the iminium ions explains the different outcomes for the two reactions. During imine formation, the nitrogen atom of the iminium ion possesses a proton that can be removed as the final step of the mechanism. In contrast, during enamine formation, the nitrogen atom of the iminium ion does not possess a proton. As a result, elimination from the adjacent carbon is necessary in order to yield a neutral species.

SKILLBUILDER 20.4 DRAWING THE MECHANISM OF ENAMINE FORMATION

Et Et O N LEARN the skill À>ÜÊ>Ê«>ÕÈLiʓiV >˜ˆÃ“ÊvœÀÊÌ iÊvœœÜˆ˜}ÊÀi>V̈œ˜\ [H2SO4] Et2NH – H2O SOLUTION / iÊÀi>V̈œ˜Ê>LœÛiʈÃÊ>˜ÊiÝ>“«iʜvÊi˜>“ˆ˜iÊvœÀ“>̈œ˜°Ê/ iʓiV >˜ˆÃ“ÊV>˜ÊLiÊ`ˆÛˆ`i`ʈ˜ÌœÊ Ìܜʫ>ÀÌÃ\Ê­£®ÊvœÀ“>̈œ˜ÊœvÊÌ iÊV>ÀLˆ˜œ>“ˆ˜iÊ>˜`Ê­Ó®ÊvœÀ“>̈œ˜ÊœvÊÌ iÊi˜>“ˆ˜i° œÀ“>̈œ˜ÊœvÊÌ iÊV>ÀLˆ˜œ>“ˆ˜iʈ˜ÛœÛiÃÊÌ ÀiiʓiV >˜ˆÃ̈VÊÃÌi«Ã\

Proton transfer Nucleophilic attack Proton transfer

7 i˜Ê `À>܈˜}Ê Ì iÃiÊ Ì ÀiiÊ ÃÌi«Ã]Ê “>ŽiÊ ÃÕÀiÊ ÌœÊ «>ViÊ Ì iÊ i>`Ê >˜`Ê Ì>ˆÊ œvÊ iÛiÀÞÊ VÕÀÛi`Ê >ÀÀœÜʈ˜ÊˆÌÃÊ«ÀiVˆÃiʏœV>̈œ˜]Ê>˜`ʓ>ŽiÊÃÕÀiÊ̜ʫ>ViÊ>Ê«œÃˆÌˆÛiÊV >À}iÃʈ˜ÊÌ iˆÀÊ>««Àœ«Àˆ>ÌiÊÊ œV>̈œ˜Ã°Ê œÌˆViÊÌ >ÌÊiÛiÀÞÊÃÌi«ÊÀiµÕˆÀiÃÊÌܜÊVÕÀÛi`Ê>ÀÀœÜð

The tail of this arrow Don’t forget the should be placed on a lone pair positive charges H Et Et STEP 1 + H O N O + + H Et HO N H Et HO N Et À>ÜÊÌ iÊÌ ÀiiÊÃÌi«ÃÊ HEtN N N ˜iViÃÃ>ÀÞÊ̜ÊvœÀ“Ê>Ê Et V>ÀLˆ˜œ>“ˆ˜i° Et Et Et

Carbinolamine Don’t forget the second curved arrow that shows release of the proton

klein_c20_001-056v1.4.indd 23 30/04/10 16.47 24 CHAPTER 20 Aldehydes and Ketones

˜ÊÌ iÊÃiVœ˜`Ê«>ÀÌʜvÊÌ iʓiV >˜ˆÃ“]ÊÌ iÊV>ÀLˆ˜œ>“ˆ˜iʈÃÊVœ˜ÛiÀÌi`ʈ˜ÌœÊ>˜Êi˜>“ˆ˜iÊ Ûˆ>Ê>ÊÌ Àii‡ÃÌi«Ê«ÀœViÃðÊ"˜ViÊ>}>ˆ˜]ÊÌ ˆÃÊÃiµÕi˜ViÊLi}ˆ˜ÃÊ>˜`Êi˜`ÃÊÜˆÌ Ê«ÀœÌœ˜ÊÌÀ>˜ÃviÀÃ\

Proton transfer Loss of a leaving group Proton transfer

The tail of this arrow Don’t forget the should be placed on a lone pair positive charges

Et H Et Et + Et Et + Et STEP 2 H N HO N + O N H Et N À>ÜÊÌ iÊÌ ÀiiÊÃÌi«ÃÊ Et HEtN H Et N ˜iViÃÃ>ÀÞÊ̜ÊVœ˜ÛiÀÌÊÌ iÊ – H Et H2O Et V>ÀLˆ˜œ>“ˆ˜iʈ˜ÌœÊ>˜Ê i˜>“ˆ˜i°

Don’t forget the second curved arrow Enamine that shows release of the proton

PRACTICE the skill 20.21 À>ÜÊ>Ê«>ÕÈLiʓiV >˜ˆÃ“ÊvœÀÊi>V ʜvÊÌ iÊvœœÜˆ˜}ÊÀi>V̈œ˜Ã\ Et Et O N

[TsOH] O [H2SO4] Et2NH Et2NH N –H O –H O (a) 2 (b) 2

O N

[H2SO4] Me2NH – (c) H2O

APPLY the skill 20.22 *Ài`ˆVÌÊÌ iʓ>œÀÊ«Àœ`ÕVÌÊvœÀÊi>V ʜvÊÌ iÊvœœÜˆ˜}ÊÀi>V̈œ˜Ã\

O [H+] H N N H O

– [H+] (a) H2O ? (b) ? – H2O

20.23 *Ài`ˆVÌÊÌ iʓ>œÀÊ«Àœ`ÕVÌÊvœÀÊi>V ʜvÊÌ iÊvœœÜˆ˜}ʈ˜ÌÀ>“œiVՏ>ÀÊÀi>V̈œ˜Ã\

NH H O O [H+] [H+] N – – H2O ? H2O ? (a) (b)

20.24 `i˜ÌˆvÞÊÌ iÊÀi>VÌ>˜ÌÃÊÌ >ÌÊޜÕÊܜՏ`ÊÕÃiÊ̜ʓ>ŽiÊi>V ʜvÊÌ iÊvœœÜˆ˜}Êi˜>“ˆ˜iÃ\

N N N (a) (b) (c)

need more PRACTICE? Try Problem 20.72

klein_c20_001-056v1.4.indd 24 30/04/10 16.47 20.6 Nitrogen Nucleophiles 25

Wolff-Kishner Reduction At the end of the previous section, we noted that ketones can be converted into hydrazones. This transformation has practical utility, because hydrazones are readily reduced under strongly basic conditions:

NNH2 H H

KOH/H2O heat

A hydrazone (82%)

This transformation is called the Wolff-Kishner reduction, named after the German chem- ist Ludwig Wolff (University of Jena) and the Russian chemist N. M. Kishner (University of Moscow). This provides a two-step procedure for reducing a ketone to an alkane:

NH2 O N H H [H+] H N¬NH KOH/H O 2 2 2 + N2 –H2O heat

(80%)

The second part of the Wolff-Kishner reduction is believed to proceed via Mechanism 20.8.

MECHANISM 20.8 THE WOLFF-KISHNER REDUCTION

H Proton transfer Proton transfer H H H N NH - N - N O HN N OH N N HH

One of the - The intermediate protons is is protonated removed, forming a resonance-stabilized Proton - intermediate OH transfer

Another proton Proton is removed Loss of a transfer leaving group - HH O H HN N HH - The carbanion Nitrogen gas is protonated, is expelled, generating the product generating a carbanion

Notice that four of the five steps in the mechanism are proton transfers, the exception being the loss of N2 gas to generate a carbanion. This step warrants special attention, because forma- tion of a carbanion in a solution of aqueous hydroxide is thermodynamically unfavorable (sig- nificantly uphill in energy). Why, then, does this step occur? It is true that the equilibrium for this step greatly disfavors formation of the carbanion, and therefore, only a very small number of molecules will initially lose N2 to form the carbanion. However, the resulting N2 gas then

klein_c20_001-056v1.4.indd 25 30/04/10 16.47 26 CHAPTER 20 Aldehydes and Ketones

bubbles out of the reaction mixture, and the equilibrium is adjusted to form more nitrogen gas, which again leaves the reaction mixture. The evolution of nitrogen gas ultimately renders this step irreversible and forces the reaction to completion. As a result, the yields for this process are generally very good.

CONCEPTUAL CHECKPOINT O 20.25 *Ài`ˆVÌÊÌ iÊ«Àœ`ÕVÌʜvÊÌ iÊÌܜ‡ÃÌi«Ê«ÀœVi`ÕÀiÊLiœÜ]Ê>˜`Ê`À>ÜÊ>Ê + – “iV >˜ˆÃ“ÊvœÀʈÌÃÊvœÀ“>̈œ˜\ 1) [H ], H2N–NH2, H2O 2) KOH / H2O, heat ?

20.7 Mechanism Strategies

Compare the mechanistic steps for the formation of acetals, imines, and enamines (Figure 20.8). Each of the mechanisms has been divided into two parts, and in all cases, the first part consists of the same three steps. In addition, even the second part of each mechanism begins with the same first two steps (proton transfer followed by loss of a leaving group). In other words, these three mechanisms are identical until the fifth step, in which water is lost (loss of a leaving group), shown in red in Figure 20.8. Rather than viewing these reactions as three separate, unrelated reactions, it is best to view them as nearly identical with different endings. Acetal formation has one additional nucleophile attack, giving a total of seven steps. In contrast, imine formation and enamine formation do not exhibit a nucleophilic attack during the second part of the mecha- nism, giving a total of only six steps.

FORMATION OF HEMIACETAL FORMATION OF ACETAL

Proton Nucleophilic Proton Proton Loss of a Nucleophilic Proton transfer attack transfer transfer leaving group attack transfer

FORMATION OF CARBINOLAMINE FORMATION OF IMINE

Proton Nucleophilic Proton Proton Loss of a Proton transfer attack transfer transfer leaving group transfer

FORMATION OF CARBINOLAMINE FORMATION OF ENAMINE

Proton Nucleophilic Proton Proton Loss of a Proton transfer attack transfer transfer leaving group transfer

FIGURE 20.8 ÊVœ“«>ÀˆÃœ˜ÊœvÊÌ iÊÃiµÕi˜ViʜvÊÃÌi«ÃÊvœÀÊ>ViÌ>]ʈ“ˆ˜i]Ê>˜`Êi˜>“ˆ˜iÊvœÀ“>̈œ˜°

In each of these mechanisms, there are four proton transfer steps. In order to draw the mechanism correctly, it is critical to draw these proton transfers properly. To do so, it will be helpful to remember the following rules that dictate when and why proton transfers occur in acid-catalyzed conditions: s 4HECARBONYLGROUPSHOULDBEPROTONATEDBEFOREITISATTACKED4HISGENERATESAMORE powerful electrophile, and it avoids formation of a negative charge that would occur if the nucleophile attacked the carbonyl directly.

klein_c20_001-056v1.4.indd 26 30/04/10 16.47 20.7 Mechanism Strategies 27

s !VOIDFORMATIONOFTWOPOSITIVECHARGESONASINGLEINTERMEDIATE4HISTYPEOFINTERMEDI- ate will generally be too high in energy to form. s 4HELEAVINGGROUPSHOULDBECOMENEUTRALWHENITLEAVES$ONOTEXPELHYDROXIDEASA leaving group; rather, it should first be protonated so that it can leave as water. s !TTHEENDOFTHEMECHANISM APROTONTRANSFERISUSEDTOFORMANEUTRALPRODUCT The four rules above correspond with each of the four proton transfers, respectively. These four rules can be consolidated into one master rule that defines when and why proton transfer steps are utilized: In acidic conditions, all reagents, intermediates, and leaving groups should either be neutral (no charge) or bear one positive charge. All of the proton transfers in the mechanism occur in order to fulfill this requirement. Acetals, imines, and enamines can be converted back into ketones by treatment with excess water under acid-catalyzed conditions:

H O+ RO OR 3

R O N + H3O

R R N + H3O

Each of the reactions above is called a hydrolysis reaction, because in each case bonds are cleaved by treatment with water. These three reactions are essentially the reverse of the reactions we have seen. The following procedure should be used in drawing the mechanisms for the above hydrolysis reactions:

1. Begin by drawing all of the intermediates without any curved arrows. For example, suppose that we want to draw the mechanism for hydrolysis of an acetal to form a ketone:

RO OR + O H3O

We did not learn the mechanism for this reaction; however, we did learn the mechanism for acetal formation. Think about the first intermediate in acetal formation (a protonated car- bonyl), and then draw that intermediate as the last intermediate of the hydrolysis reaction:

OR + H O O

OR

The last intermediate should be a protonated carbonyl

Continue working backward until you have drawn all of the intermediates. 2. Then, working forward, draw the curved arrows that are necessary to transform each inter- mediate into the next intermediate. At each stage, make sure you are following the master rule for proton transfers in acid-catalyzed conditions.

The skill of being able to draw the reverse of a known mechanism is incredibly important and will be used again for other reactions in the remaining chapters of this book. The following example illustrates this procedure.

klein_c20_001-056v1.4.indd 27 30/04/10 16.47 28 CHAPTER 20 Aldehydes and Ketones

SKILLBUILDER 20.5 DRAWING THE MECHANISM OF A HYDROLYSIS REACTION

LEARN the skill *Àœ«œÃiÊ>ʓiV >˜ˆÃ“ÊvœÀÊÌ iÊvœœÜˆ˜}ÊÌÀ>˜ÃvœÀ“>̈œ˜\

O O O

+ H3O + HO OH

SOLUTION / ˆÃʈÃÊ>Ê Þ`ÀœÞÈÃÊÀi>V̈œ˜Êˆ˜ÊÜ ˆV Ê>ÊVÞVˆVÊ>ViÌ>ÊˆÃʜ«i˜i`Ê̜ÊvœÀ“Ê>ʎi̜˜i°Ê7iÊÌ iÀivœÀiÊ iÝ«iVÌÊÌ iʓiV >˜ˆÃ“Ê̜ÊLiÊÌ iÊÀiÛiÀÃiʜvÊ>ViÌ>ÊvœÀ“>̈œ˜°Ê i}ˆ˜ÊLÞÊVœ˜Ãˆ`iÀˆ˜}Ê>ÊœvÊÌ iÊ ˆ˜ÌiÀ“i`ˆ>ÌiÃʈ˜ÛœÛi`ʈ˜Ê>ViÌ>ÊvœÀ“>̈œ˜\

OH OH OH OH

+ H H + O O + + O + HO O HO O H O O O O O O H H

KetoneFirst Hemiacetal Acetal intermediate

7iÊ Ãˆ“«ÞÊ `À>ÜÊ >Ê œvÊ Ì iÃiÊ ˆ˜ÌiÀ“i`ˆ>ÌiÃÊ ˆ˜Ê ÀiÛiÀÃiÊ œÀ`iÀÊ ÃœÊ Ì >ÌÊ Ì iÊ wÀÃÌÊ ˆ˜ÌiÀ“i`ˆ>ÌiÊ STEP 1 >LœÛiÊ­ ˆ} ˆ} Ìi`®ÊLiVœ“iÃÊÌ iʏ>ÃÌʈ˜ÌiÀ“i`ˆ>ÌiʜvÊÌ iÊ Þ`ÀœÞÈÃʓiV >˜ˆÃ“\ À>ÜÊ>Êˆ˜ÌiÀ“i`ˆ>ÌiÃÊ vœÀÊ>ViÌ>ÊvœÀ“>̈œ˜Êˆ˜Ê ÀiÛiÀÃiʜÀ`iÀ° OH OH OH OH

+ H + H + O + + O O O O O O H O O HO O HO O H H

Acetal Hemiacetal Last Ketone intermediate

Þ`ÀœÞÈÃʜvÊÌ iÊ>ViÌ>Ê“ÕÃÌʈ˜ÛœÛiÊÌ iÃiʈ˜ÌiÀ“i`ˆ>ÌiÃ]ʈ˜ÊÌ iʜÀ`iÀÊà œÜ˜Ê>LœÛi°ÊvÊ>˜ÞÊ œvÊÌ iʈ˜ÌiÀ“i`ˆ>ÌiÃÊ >ÃÊ>ʘi}>̈ÛiÊV >À}i]ÊÌ i˜ÊޜÕÊ >Ûiʓ>`iÊ>ʓˆÃÌ>Ži° 7ˆÌ ÊÌ iʈ˜ÌiÀ“i`ˆ>ÌiÃÊ«>Vi`ʈ˜ÊÌ iÊVœÀÀiVÌʜÀ`iÀ]ÊÌ iÊw˜>ÊÃÌi«ÊˆÃÊ̜Ê`À>ÜÊÌ iÊÀi>}i˜ÌÃÊ >˜`ÊVÕÀÛi`Ê>ÀÀœÜÃÊÌ >ÌÊà œÜÊ œÜÊi>V ʈ˜ÌiÀ“i`ˆ>ÌiʈÃÊÌÀ>˜ÃvœÀ“i`ʈ˜ÌœÊÌ iʘiÝÌʈ˜ÌiÀ“i`ˆ- >Ìi°Ê i}ˆ˜ÊÜˆÌ ÊÌ iÊ>ViÌ>]Ê>˜`ÊܜÀŽÊvœÀÜ>À`Ê՘̈ÊÀi>V ˆ˜}ÊÌ iʎi̜˜i°Ê/ ˆÃÊÀiµÕˆÀiÃÊÌ >ÌÊ ÞœÕÀÊ>ÀÀœÜ‡«Õà ˆ˜}ÊΈÃÊ>Àiʈ˜Ê}œœ`Êà >«i° LOOKING BACK >ŽiÊÃÕÀiÊ̜ÊÕÃiʜ˜ÞÊÌ iÊÀi>}i˜ÌÃÊÌ >ÌÊ>ÀiÊ«ÀœÛˆ`i`]Ê>˜`ʜLiÞÊÌ iʓ>ÃÌiÀÊÀՏiÊvœÀÊ + vÊޜÕʘii`ÊÌœÊ «ÀœÌœ˜ÊÌÀ>˜ÃviÀðÊœÀÊiÝ>“«i]ÊÌ ˆÃÊ«ÀœLi“ʈ˜`ˆV>ÌiÃÊÌ >ÌÊÎ" ʈÃÊ>Û>ˆ>Li°Ê/ ˆÃʓi>˜ÃÊÌ >ÌÊ + «œˆÃ ÊޜÕÀÊ>ÀÀœÜ‡ HÎ" Êà œÕ`ÊLiÊÕÃi`ÊvœÀÊ«ÀœÌœ˜>̈˜}]Ê>˜`ÊÓ"Êà œÕ`ÊLiÊÕÃi`ÊvœÀÊ`i«ÀœÌœ˜>̈˜}°Ê œÊ˜œÌÊ «Õà ˆ˜}ÊΈÃ]Ê}œÊ ÕÃiÊ Þ`ÀœÝˆ`iʈœ˜Ã]Ê>ÃÊÌ iÞÊ>ÀiʘœÌÊ«ÀiÃi˜Ìʈ˜ÊÃÕvwVˆi˜ÌʵÕ>˜ÌˆÌÞÊ՘`iÀÊ>Vˆ`‡V>Ì>Þâi`ÊVœ˜`ˆ- ̜Ê-iV̈œ˜ÊÈ°n° ̈œ˜Ã°Ê««ˆV>̈œ˜ÊœvÊÌ iÃiÊÀՏiÃÊ}ˆÛiÃÊÌ iÊvœœÜˆ˜}Ê>˜ÃÜiÀ\

klein_c20_001-056v1.4.indd 28 30/04/10 16.47 20.7 Mechanism Strategies 29

OH OH OH

+ H + O + H O O HO O O O + O O H O H O O H H H H H H

Acetal Hemiacetal

STEP 2 H + À>ÜÊÌ iÊVÕÀÛi`Ê>ÀÀœÜÃÊ O >˜`ʘiViÃÃ>ÀÞÊÀi>}i˜ÌÃÊ H H vœÀÊi>V ÊÃÌi«ÊœvÊÌ iÊ “iV >˜ˆÃ“° OH

H + O O + HO O O H H H HO OH +

Ketone œÌˆViÊÌ iÊÕÃiʜvÊiµÕˆˆLÀˆÕ“Ê>ÀÀœÜÃ]ÊLiV>ÕÃiÊÌ iÊ«ÀœViÃÃʈÃÊ}œÛiÀ˜i`ÊLÞÊ>˜ÊiµÕˆˆLÀˆÕ“]Ê>ÃÊ ˜œÌi`ʈ˜Ê«ÀiۈœÕÃÊÃiV̈œ˜Ã°

PRACTICE the skill 20.26 *Àœ«œÃiÊ>Ê«>ÕÈLiʓiV >˜ˆÃ“ÊvœÀÊi>V ʜvÊÌ iÊvœœÜˆ˜}Ê Þ`ÀœÞÈÃÊÀi>V̈œ˜Ã\

O EtO OEt N O H + + H3O H3O + 2 EtOH + N (a) (b)

N O H O+ 3 + (c) MeNH2

O H O O + H3O (d) HO OH

APPLY the skill 20.27 *Àœ«œÃiÊ>Ê«>ÕÈLiʓiV >˜ˆÃ“ÊvœÀÊÌ iÊÀi>V̈œ˜ÊLiœÜ\ O O N [H2SO4] OH

H2N HO

Hint: ÌʈÃʘœÌʘiViÃÃ>ÀÞÊ̜ʜ«i˜ÊÌ iÊ>ViÌ>Êˆ˜ÌœÊ>ʎi̜˜iÊ­ÇÊÃÌi«Ã®ÊvœœÜi`ÊLÞÊVœÃˆ˜}ÊÌ iÊ ˆ“ˆ˜iÊ­ÈÊÃÌi«Ã®°ÊÊ«>ÕÈLiʓiV >˜ˆÃ“ÊV>˜ÊLiÊ`À>ܘÊÜˆÌ ÊviÜiÀÊÌ >˜Ê£ÎÊÃÌi«Ã°Ê-Ì>ÀÌÊܜÀŽ- ˆ˜}ÊÜˆÌ ÊÌ iÊ>ViÌ>Ê­ÕȘ}ÊÌ iÊÃÌi«ÃʘiViÃÃ>ÀÞÊ̜ʜ«i˜Ê>˜Ê>ViÌ>®]Ê>˜`ʏœœŽÊvœÀÊ>˜Êˆ˜ÌiÀ“i`ˆ-

>ÌiÊÌ >ÌÊV>˜ÊLiÊ>ÌÌ>VŽi`ÊLÞÊÌ iÊ>“ˆ˜œÊ}ÀœÕ«°Ê >ŽiÊÃÕÀiÊ̜Ê>ۜˆ`Ê`À>܈˜}Ê>˜Ê-NÓÊ«ÀœViÃÃÊ >ÌÊ>ÊÃÌiÀˆV>ÞÊ ˆ˜`iÀi`ÊÃÕLÃÌÀ>Ìit

need more PRACTICE? Try Problem 20.65

klein_c20_001-056v1.4.indd 29 30/04/10 16.47 30 CHAPTER 20 Aldehydes and Ketones

The Medically Speaking box below provides two examples of hydrolysis reactions that are exploited in drug design.

MEDICALLYSPEAKING Prodrugs

Methenamine as a Prodrug of Formaldehyde iÌ i˜>“ˆ˜iʈÃÊ«>Vi`ʈ˜ÊëiVˆ>ÊÌ>LiÌÃÊÌ >ÌÊ`œÊ˜œÌÊ`ˆÃ܏ÛiÊ œÀ“>`i Þ`iÊ >ÃÊ>˜ÌˆÃi«ÌˆVÊ«Àœ«iÀ̈iÃÊ>˜`ÊV>˜ÊLiÊi“«œÞi`ʈ˜Ê >ÃÊÌ iÞÊÌÀ>ÛiÊÌ ÀœÕ} ÊÌ iÊ>Vˆ`ˆVÊi˜ÛˆÀœ˜“i˜ÌʜvÊÌ iÊÃ̜“>V ÊLÕÌÊ Ì iÊÌÀi>̓i˜ÌʜvÊÕÀˆ˜>ÀÞÊÌÀ>VÌʈ˜viV̈œ˜ÃÊ`ÕiÊ̜ʈÌÃÊ>LˆˆÌÞÊ̜ÊÀi>VÌÊ `œÊ`ˆÃ܏Ûiʜ˜ViÊÌ iÞÊÀi>V ÊÌ iÊL>ÈVÊi˜ÛˆÀœ˜“i˜ÌʜvÊÌ iʈ˜ÌiÃ̈- ÜˆÌ Ê˜ÕViœ« ˆiÃÊ«ÀiÃi˜Ìʈ˜ÊÕÀˆ˜i°ÊœÜiÛiÀ]ÊvœÀ“>`i Þ`iÊV>˜Ê ˜>ÊÌÀ>VÌ°Ê iÌ i˜>“ˆ˜iʈÃÊÌ iÀiLÞÊÀii>Ãi`ʈ˜ÊÌ iʈ˜ÌiÃ̈˜>ÊÌÀ>VÌ]Ê LiÊ̜݈VÊÜ i˜ÊiÝ«œÃi`ÊÌœÊœÌ iÀÊÀi}ˆœ˜ÃʜvÊÌ iÊLœ`Þ°Ê/ iÀivœÀi]Ê Ü iÀiÊ ˆÌÊ ˆÃÊ ÃÌ>LiÊ Õ˜`iÀÊ L>ÈVÊ Vœ˜`ˆÌˆœ˜Ã°Ê "˜ViÊ ˆÌÊ Ài>V iÃÊ Ì iÊ Ì iÊÕÃiʜvÊvœÀ“>`i Þ`iÊ>ÃÊ>˜Ê>˜ÌˆÃi«ÌˆVÊ>}i˜ÌÊÀiµÕˆÀiÃÊ>ʓiÌ œ`Ê >Vˆ`ˆVÊ i˜ÛˆÀœ˜“i˜ÌÊ œvÊ Ì iÊ ÕÀˆ˜>ÀÞÊ ÌÀ>VÌ]Ê “iÌ i˜>“ˆ˜iÊ ˆÃÊ Þ`Àœ- vœÀÊ ÃiiV̈ÛiÊ `iˆÛiÀÞÊ ÌœÊ Ì iÊ ÕÀˆ˜>ÀÞÊ ÌÀ>VÌ°Ê / ˆÃÊ V>˜Ê LiÊ >VVœ“- Þâi`]Ê Àii>Ș}Ê vœÀ“>`i Þ`i]Ê >ÃÊ Ã œÜ˜Ê >LœÛi°Ê ˜Ê Ì ˆÃÊ Ü>Þ]Ê «ˆÃ i`ÊLÞÊÕȘ}Ê>Ê«Àœ`ÀÕ}ÊV>i`ʓiÌ i˜>“ˆ˜i\ “iÌ i˜>“ˆ˜iʈÃÊÕÃi`Ê>ÃÊ>Ê«Àœ`ÀÕ}ÊÌ >ÌÊi˜>LiÃÊ`iˆÛiÀÞʜvÊvœÀ“- N >`i Þ`iÊëiVˆwV>ÞÊ̜ÊÌ iÊÕÀˆ˜>ÀÞÊÌÀ>VÌ°Ê/ ˆÃʓiÌ œ`Ê«ÀiÛi˜ÌÃÊ Ì iÊÃÞÃÌi“ˆVÊÀii>ÃiʜvÊvœÀ“>`i Þ`iʈ˜ÊœÌ iÀʜÀ}>˜ÃʜvÊÌ iÊLœ`ÞÊ N N Ü iÀiʈÌÊܜՏ`ÊLiÊ̜݈V° N Methenamine CONCEPTUAL CHECKPOINT / ˆÃÊVœ“«œÕ˜`ʈÃÊ>ʘˆÌÀœ}i˜Ê>˜>œ}ÕiʜvÊ>˜Ê>ViÌ>°Ê/ >ÌʈÃ]Êi>V Ê 20.28 ÃÊà œÜ˜Ê>LœÛi]ʓiÌ i˜>“ˆ˜iʈÃÊ Þ`ÀœÞâi`ʈ˜Ê>µÕiœÕÃÊ V>ÀLœ˜Ê>̜“ʈÃÊVœ˜˜iVÌi`Ê̜ÊÌܜʘˆÌÀœ}i˜Ê>̜“Ã]ÊÛiÀÞʓÕV ʏˆŽiÊ >Vˆ`Ê̜ʫÀœ`ÕViÊvœÀ“>`i Þ`iÊ>˜`Ê>““œ˜ˆ>°Ê À>ÜÊ>ʓiV >˜ˆÃ“Ê >˜Ê >ViÌ>Ê ˆ˜Ê Ü ˆV Ê >Ê V>ÀLœ˜Ê >̜“Ê ˆÃÊ Vœ˜˜iVÌi`Ê ÌœÊ ÌÜœÊ œÝÞ}i˜Ê à œÜˆ˜}ÊvœÀ“>̈œ˜Êœvʜ˜iʓœiVՏiʜvÊvœÀ“>`i Þ`iÊ­Ì iÊÀi“>ˆ˜- >̜“ðÊÊV>ÀLœ˜Ê>̜“ÊÌ >ÌʈÃÊVœ˜˜iVÌi`Ê̜ÊÌÜœÊ iÌiÀœ>̜“ÃÊ­"Ê ˆ˜}ÊwÛiʓœiVՏiÃʜvÊvœÀ“>`i Þ`iÊ>ÀiÊi>V ÊÀii>Ãi`Êۈ>Ê>Êȓˆ>ÀÊ œÀÊ ®ÊV>˜Ê՘`iÀ}œÊ>Vˆ`‡V>Ì>Þâi`Ê Þ`ÀœÞÈÃ\ ÃiµÕi˜ViÊ œvÊ ÃÌi«Ã®°Ê / iÊ Àii>ÃiÊ œvÊ i>V Ê “œiVՏiÊ œvÊ vœÀ“>`i- Þ`iʈÃÊ`ˆÀiV̏ÞÊ>˜>œ}œÕÃÊ̜ÊÌ iÊ Þ`ÀœÞÈÃʜvÊ>˜Ê>ViÌ>°Ê/œÊ}iÌÊ ZZ O + H3O ޜÕÊÃÌ>ÀÌi`]ÊÌ iÊwÀÃÌÊÌܜÊÃÌi«ÃÊ>ÀiÊ«ÀœÛˆ`i`ÊLiœÜ\

RR R R H “ N N Z O or N O + H H >V Ê œvÊ Ì iÊ V>ÀLœ˜Ê >̜“ÃÊ ˆ˜Ê “iÌ i˜>“ˆ˜iÊ V>˜Ê LiÊ Þ`ÀœÞâi`]Ê N N N N Àii>Ș}ÊvœÀ“>`i Þ`i\ N N + H

N + O + H O+ N 3 + 6 4 NH4 N N H2O N H H N N Formaldehyde N H

Imines as Prodrugs / iÊ Vœ“«œÕ˜`Ê i݈ÃÌÃÊ «Àˆ“>ÀˆÞÊ ˆ˜Ê Ì ˆÃÊ ˆœ˜ˆVÊ vœÀ“]Ê Ü ˆV Ê V>˜- / iʈ“ˆ˜iʓœˆiÌÞʈÃÊÕÃi`ʈ˜ÊÌ iÊ`iÛiœ«“i˜Ìʜvʓ>˜ÞÊ«Àœ`ÀÕ}Ã°Ê ˜œÌÊ VÀœÃÃÊ Ì iÊ ˜œ˜«œ>ÀÊ i˜ÛˆÀœ˜“i˜ÌÊ œvÊ Ì iÊ Lœœ`‡LÀ>ˆ˜Ê L>ÀÀˆiÀ°Ê iÀiÊÜiÊ܈ÊiÝ«œÀiʜ˜iÊÃÕV ÊiÝ>“«i° *Àœ}>Lˆ`iÊ ˆÃÊ >Ê «Àœ`ÀÕ}Ê `iÀˆÛ>̈ÛiÊ ÕÃi`Ê ÌœÊ ÌÀi>ÌÊ «>̈i˜ÌÃÊ Ü œÊ / iÊVœ“«œÕ˜`ÊLiœÜ]ÊL‡>“ˆ˜œLÕÌÞÀˆVÊ>Vˆ`]ʈÃÊ>˜Êˆ“«œÀÌ>˜ÌÊ iÝ ˆLˆÌÊÌ iÊÃޓ«Ìœ“ÃʜvÊ>Ê`iwVˆi˜VÞʜvÊL‡>“ˆ˜œLÕÌÞÀˆVÊ>Vˆ`\ ˜iÕÀœÌÀ>˜Ã“ˆÌÌiÀ\ O OH b O H2N gaOH N F NH2 g-aminobutyric acid

Ê `iwVˆi˜VÞÊ œvÊ Ì ˆÃÊ Vœ“«œÕ˜`Ê V>˜Ê V>ÕÃiÊ Vœ˜ÛՏȜ˜Ã°Ê `“ˆ˜ˆÃ̇ Progabide iÀˆ˜}Ê L‡>“ˆ˜œLÕÌÞÀˆVÊ >Vˆ`Ê `ˆÀiV̏ÞÊ ÌœÊ >Ê «>̈i˜ÌÊ ˆÃÊ ˜œÌÊ >˜Ê ivviV̈ÛiÊ ÌÀi>̓i˜Ì]Ê LiV>ÕÃiÊ Ì iÊ Vœ“«œÕ˜`Ê `œiÃÊ ˜œÌÊ Ài>`ˆÞÊ VÀœÃÃÊ Ì iÊ Lœœ`‡LÀ>ˆ˜ÊL>ÀÀˆiÀ°Ê7 ÞʘœÌ¶ÊÌÊ« ÞȜœ}ˆV>Ê«]ÊÌ iÊ>“ˆ˜œÊ}ÀœÕ«Ê Cl ˆÃÊ«ÀœÌœ˜>Ìi`Ê>˜`ÊÌ iÊV>ÀLœÝޏˆVÊ>Vˆ`ʓœˆiÌÞʈÃÊ`i«ÀœÌœ˜>Ìi`\ / iÊ V>ÀLœÝޏˆVÊ >Vˆ`Ê >ÃÊ Lii˜Ê Vœ˜ÛiÀÌi`Ê ÌœÊ >˜Ê >“ˆ`i]Ê >˜`Ê Ì iÊ H O >“ˆ˜œÊ}ÀœÕ«Ê >ÃÊLii˜ÊVœ˜ÛiÀÌi`ʈ˜ÌœÊ>˜Êˆ“ˆ˜iÊ­ ˆ} ˆ} Ìi`®°ÊÌÊ H + - N « ÞȜœ}ˆV>Ê«]ÊÌ ˆÃÊVœ“«œÕ˜`Êi݈ÃÌÃÊ«Àˆ“>ÀˆÞÊ>ÃÊ>ʘiÕÌÀ>ÊVœ“ O - H «œÕ˜`ʭ՘V >À}i`®]Ê>˜`ʈÌÊV>˜ÊÌ iÀivœÀiÊVÀœÃÃÊÌ iÊLœœ`‡LÀ>ˆ˜ÊL>À-

klein_c20_001-056v1.4.indd 30 30/04/10 16.47 20.8 Sulfur Nucleophiles 31

ÀˆiÀ°Ê"˜Viʈ˜ÊÌ iÊLÀ>ˆ˜]ʈÌʈÃÊVœ˜ÛiÀÌi`Ê̜ÊL‡>“ˆ˜œLÕÌÞÀˆVÊ>Vˆ`Êۈ>Ê *Àœ}>Lˆ`iʈÃʍÕÃÌʜ˜iÊiÝ>“«iʈ˜ÊÜ ˆV ÊÌ iʈ“ˆ˜iʓœˆiÌÞÊ >ÃÊLii˜Ê Þ`ÀœÞÈÃʜvÊÌ iʈ“ˆ˜iÊ>˜`Ê>“ˆ`iʓœˆïiÃ\ ÕÃi`ʈ˜ÊÌ iÊ`iÛiœ«“i˜ÌʜvÊ>Ê«Àœ`ÀÕ}°

OH O

N hydrolysis F NH2 O

H2N OH

Cl

20.8 Sulfur Nucleophiles

In acidic conditions, an aldehyde or ketone will react with two equivalents of a thiol to form a thioacetal:

O RS SR [H+] + + 2 RSH H2O

Thioacetal

The mechanism of this transformation is directly analogous to acetal formation, with sulfur atoms taking the place of oxygen atoms. If a compound with two SH groups is used, a cyclic thioacetal is formed:

O [H+] S S + + H O HS SH 2 Cyclic thioacetal

When treated with Raney nickel, thioacetals undergo desulfurization, yielding an alkane:

HH S S Raney Ni RR RR

Raney Ni is a spongy form of nickel that has adsorbed hydrogen atoms. It is these hydrogen atoms that ultimately replace the sulfur atoms, although a discussion of the mechanism for desulfurization is beyond the scope of this text. The reactions above provide us with another two-step method for the reduction of a ketone:

O HH

1) [H+],HS SH 2) Raney Ni

This method involves formation of the thioacetal followed by desulfurization with Raney nickel. It is the third method we have encountered for achieving this type of transformation. The other two methods are the Clemmensen reduction (Section 19.6) and the Wolff-Kishner reduction (Section 20.6).

klein_c20_001-056v1.4.indd 31 30/04/10 16.47 32 CHAPTER 20 Aldehydes and Ketones

CONCEPTUAL CHECKPOINT

20.29 *Ài`ˆVÌÊÌ iʓ>œÀÊ«Àœ`ÕVÌÊvœÀÊi>V ÊÀi>V̈œ˜ÊLiœÜ\ 20.30 À>ÜÊÌ iÊÃÌÀÕVÌÕÀiʜvÊÌ iÊVÞVˆVÊVœ“«œÕ˜`ÊÌ >ÌʈÃÊ«Àœ- O `ÕVi`ÊÜ i˜Ê>Vi̜˜iʈÃÊÌÀi>Ìi`ÊÜˆÌ Ê£]·«Àœ«>˜i`ˆÌ ˆœÊˆ˜ÊÌ iÊ «ÀiÃi˜ViʜvÊ>˜Ê>Vˆ`ÊV>Ì>ÞÃÌ° 1) [H+], HS SH O 2) Raney Ni (a) ? HS SH Acetone 1,3-propanedithiol O

H 1) [H+], HS SH 2) Raney Ni (b) ?

20.9 Hydrogen Nucleophiles

When treated with a hydride reducing agent, such as LAH or sodium borohydride (NaBH4), aldehydes and ketones are reduced to alcohols:

1) LAH

2) H2O O OH

RR RR

NaBH4, MeOH

These reactions were discussed in Section 13.4, and we saw that LAH and NaBH4 both func- tion as delivery agents of hydride (H−). The precise mechanism of action for these reagents has been heavily investigated and is somewhat complex. Nevertheless, the simplified version shown in Mechanism 20.9 will be sufficient for our purposes.

MECHANISM 20.9 THE REDUCTION OF KETONES OR ALDEHYDES WITH HYDRIDE AGENTS

Nucleophilic attack Proton transfer

- H O O O O H H RR RR RR H Lithium aluminium hydride (LAH ) H The resulting H - functions as a delivery agent tetrahedral intermediate is – HHAl of hydride ions (H ) protonated to form an alcohol H

In the first step of the mechanism, the reducing agent delivers a hydride ion, which attacks the carbonyl group, producing a tetrahedral intermediate. This intermediate is then treated with a proton source to yield the product. This simplified mechanism does not take into account many important observations, such as the role of the lithium cation (Li+). For example, when

klein_c20_001-056v1.4.indd 32 30/04/10 16.47 20.10 Carbon Nucleophiles 33

LOOKING BACK 12-crown-4 is added to the reaction mixture, the lithium ions are solvated (as described in Þ`Àˆ`iÊV>˜˜œÌÊv՘V̈œ˜Ê>ÃÊ Section 14.4), and reduction does not occur. Clearly, the lithium cation plays a pivotal role >ʏi>ۈ˜}Ê}ÀœÕ«ÊLiV>ÕÃiÊ in the mechanism. However, a full treatment of the mechanism of hydride reducing agents is ˆÌʈÃÊ̜œÊÃÌÀœ˜}ÞÊL>ÈV°Ê beyond the scope of this text, and the simplified version above will suffice. ­-iiÊ-iV̈œ˜ÊÇ°n°® The reduction of a carbonyl group with LAH or NaBH4 is not a reversible process, because hydride does not function as a leaving group. Notice that the mechanism above employs one- way arrows (rather than equilibrium arrows) to signify that the reverse process is insignificant.

CONCEPTUAL CHECKPOINT

20.31 *Ài`ˆVÌÊ Ì iÊ “>œÀÊ «Àœ`ÕVÌÊ vœÀÊ i>V Ê œvÊ Ì iÊ vœœÜˆ˜}Ê 20.32 7 i˜Ê ÓÊ “œiÃÊ œvÊ Li˜â>`i Þ`iÊ >ÀiÊ ÌÀi>Ìi`Ê ÜˆÌ Ê Ài>V̈œ˜Ã\ Ü`ˆÕ“Ê Þ`ÀœÝˆ`i]Ê>ÊÀi>V̈œ˜ÊœVVÕÀÃʈ˜ÊÜ ˆV ʣʓœiʜvÊLi˜â>- O `i Þ`iʈÃʜ݈`ˆâi`Ê­}ˆÛˆ˜}ÊLi˜âœˆVÊ>Vˆ`®ÊÜ ˆiÊÌ iÊœÌ iÀʓœiʜvÊ 1) LAH Li˜â>`i Þ`iʈÃÊÀi`ÕVi`Ê­}ˆÛˆ˜}ÊLi˜âޏÊ>Vœ œ®\ 2) H2O ? (a) O O OH

H O H 1) NaOH OH + H 2) H3O H NaBH4, MeOH ? / ˆÃÊÀi>V̈œ˜]ÊV>i`ÊÌ iÊ >˜˜ˆââ>ÀœÊÀi>V̈œ˜]ʈÃÊLiˆiÛi`ÊÌœÊ (b) œVVÕÀÊۈ>ÊÌ iÊvœœÜˆ˜}ʓiV >˜ˆÃ“\ÊÊ Þ`ÀœÝˆ`iʈœ˜ÊÃiÀÛiÃÊ>ÃÊ >ʘÕViœ« ˆiÊ̜Ê>ÌÌ>VŽÊÌ iÊV>ÀLœ˜ÞÊ}ÀœÕ«ÊœvÊLi˜â>`i Þ`i]Ê O }i˜iÀ>̈˜}Ê>ÊÌiÌÀ> i`À>Êˆ˜ÌiÀ“i`ˆ>Ìi°Ê/ ˆÃÊÌiÌÀ> i`À>Êˆ˜ÌiÀ- “i`ˆ>ÌiÊÌ i˜Êv՘V̈œ˜ÃÊ>ÃÊ>Ê Þ`Àˆ`iÊÀi`ÕVˆ˜}Ê>}i˜ÌÊLÞÊ`iˆÛiÀ- 1) LAH ˆ˜}Ê>Ê Þ`Àˆ`iʈœ˜Ê̜Ê>˜œÌ iÀʓœiVՏiʜvÊLi˜â>`i Þ`i°Ê˜ÊÌ ˆÃÊ 2) H O (c) 2 ? Ü>Þ]ʜ˜iʓœiVՏiʈÃÊÀi`ÕVi`ÊÜ ˆiÊÌ iÊœÌ iÀʈÃʜ݈`ˆâi`° ­>® 1Ș}ÊÌ iÊiÝ«>˜>̈œ˜Ê>LœÛi]Ê`À>ÜÊÌ iʓiV >˜ˆÃ“ÊœvÊÌ iÊ O >˜˜ˆââ>ÀœÊÀi>V̈œ˜° + ­L® 7 >ÌʈÃÊÌ iÊv՘V̈œ˜ÊœvÊÎ" ʈ˜ÊÌ iÊÃiVœ˜`ÊÃÌi«¶ 1) NaBH4 2) MeOH ? ­V® 7>ÌiÀÊ>œ˜iʈÃʘœÌÊÃÕvwVˆi˜ÌÊ̜Ê>VVœ“«ˆÃ ÊÌ iÊv՘V̈œ˜ÊœvÊ (d) Ì iÊÃiVœ˜`ÊÃÌi«°Ê Ý«>ˆ˜°

20.10 Carbon Nucleophiles

Grignard Reagents When treated with a Grignard reagent, aldehydes and ketones are converted into alcohols, accompanied by the formation of a new C–C bond:

O H3C OH O OH 1) CH3MgBr 1) CH3MgBr 2) H O 2) H O 2 H 2 CH3 H

Grignard reactions were discussed in more detail in Section 13.6. The precise mechanism of action for these reagents has been heavily investigated and is fairly complex. The simplified ver- sion shown in Mechanism 20.10 will be sufficient for our purposes.

klein_c20_001-056v1.4.indd 33 30/04/10 16.47 34 CHAPTER 20 Aldehydes and Ketones

MECHANISM 20.10 THE REACTION BETWEEN A GRIGNARD REAGENT AND A KETONE OR ALDEHYDE

Nucleophilic attack Proton transfer - H O O O O H H R R R R R R R R - The Grignard reagent The resulting R functions as a nucleophile tetrahedral intermediate is and attacks the carbonyl group protonated to form an alcohol

LOOKING BACK >ÀL>˜ˆœ˜ÃÊÀ>ÀiÞÊv՘V̈œ˜Ê Grignard reactions are not reversible because carbanions generally do not function as leaving >Ãʏi>ۈ˜}Ê}ÀœÕ«ÃÊLiV>ÕÃiÊ groups. Notice that the mechanism above employs one-way arrows (rather than equilibrium Ì iÞÊ>ÀiÊ}i˜iÀ>ÞÊÃÌÀœ˜}ÞÊ arrows) to signify that the reverse process is insignificant. L>ÈV°Ê­-iiÊ-iV̈œ˜ÊÇ°n°®

CONCEPTUAL CHECKPOINT

20.33 *Ài`ˆVÌÊÌ iʓ>œÀÊ«Àœ`ÕVÌʜvÊi>V ÊÀi>V̈œ˜ÊLiœÜ\ 20.34 `i˜ÌˆvÞÊÌ iÊÀi>}i˜ÌÃʘiViÃÃ>ÀÞÊ̜Ê>VVœ“«ˆÃ Êi>V ʜvÊ O Ì iÊÌÀ>˜ÃvœÀ“>̈œ˜ÃÊLiœÜ\ OH Me OH 1) EtMgBr 2) H2O ? (a) (a) O OH OH H 1) PhMgBr 2) H2O ? (b) (b) O O

1) PhMgBr O + 2) H3O ? (c)

Cyannohydrin Formation When treated with hydrogen cyanide (HCN), aldehydes and ketones are converted into cya- nohydrins, which are characterized by the presence of a cyano group and a hydroxyl group connected to the same carbon atom:

O HCN HO CN

A cyanohydrin

This reaction was studied extensively by Arthur Lapworth (University of Manchester) and was found to occur more rapidly in mildly basic conditions. In the presence of a catalytic amount of base, a small amount of hydrogen cyanide is deprotonated to give cyanide ions, which catalyze the reaction (Mechanism 20.11).

klein_c20_001-056v1.4.indd 34 30/04/10 16.47 20.10 Carbon Nucleophiles 35

MECHANISM 20.11 CYANOHYDRIN FORMATION

Nucleophilic attack Proton transfer

- O OH - O CN H CN CN CN

The cyanide ion functions The tetrahedral intermediate as a nucleophile and attacks the carbonyl is protonated, group, forming a tetrahedral intermediate generating a cyanohydrin

In the first step, a cyanide ion attacks the carbonyl to produce a tetrahedral intermediate. This intermediate then abstracts a proton from HCN, regenerating a cyanide ion. In this way, cya- nide functions as a catalyst for the addition of HCN to the carbonyl group. Rather than using a catalytic amount of base to form cyanide ions, the reaction can simply be performed in a mixture of HCN and cyanide ions (from KCN). The process is reversible, and the yield of products is therefore determined by equilibrium concentrations. For most aldehydes and unhindered ketones, the equilibrium favors formation of the cyanohydrin:

O HO CN KCN, HCN

H3C CH3 R R 78%

O HO CN

H KCN, HCN H

88% HCN is a liquid at room temperature and is extremely hazardous to handle because it is highly toxic and volatile (b.p. = 26°C). To avoid the dangers associated with handling HCN, cyano- hydrins can also be prepared by treating a ketone or aldehyde with potassium cyanide and an alternate source of protons, such as HCl:

O KCN, HCl HO CN

Cyanohydrins are useful in syntheses, because the cyano group can be further treated to yield a range of products. Two examples are shown below: N

HO C 1) LAH HO NH2

2) H2O R H R H O

+ H3O HO C heat OH R H In the first example, the cyano group is reduced to an amino group. In the second example, the cyano group is hydrolyzed to give a carboxylic acid. Both of these reactions and their mecha- nisms will be explored in more detail in the next chapter.

klein_c20_001-056v1.4.indd 35 30/04/10 16.47 36 CHAPTER 20 Aldehydes and Ketones

CONCEPTUAL CHECKPOINT

20.35 *Ài`ˆVÌÊÌ iʓ>œÀÊ«Àœ`ÕVÌÊvœÀÊi>V ÊÀi>V̈œ˜ÊLiœÜ\ 20.36 `i˜ÌˆvÞÊÌ iÊÀi>}i˜ÌÃʘiViÃÃ>ÀÞÊ̜Ê>VVœ“«ˆÃ Êi>V ʜvÊ O Ì iÊÌÀ>˜ÃvœÀ“>̈œ˜ÃÊLiœÜ\ 1) KCN, HCN OH O 2) LAH OH 3) H2O ? (a) OH (a)

O OH 1) KCN, HCl OH H + 2) H3O , heat ? NH 2 (b) (b)

PRACTICALLYSPEAKING Cyanohydrin Derivatives in Nature

“Þ}`>ˆ˜ÊˆÃÊ>ʘ>ÌÕÀ>ÞʜVVÕÀÀˆ˜}ÊVœ“«œÕ˜`ÊvœÕ˜`ʈ˜ÊÌ iÊ«ˆÌÃʜvÊ / ˆÃʏ>ÃÌÊÃÌi«Ê­}i˜iÀ>̈œ˜ÊœvÊ Ê}>îʈÃÊÕÃi`Ê>ÃÊ>Ê`ivi˜ÃiʓiV - >«ÀˆVœÌÃ]Ê܈`ÊV iÀÀˆiÃ]Ê>˜`Ê«i>V ið >˜ˆÃ“ÊLÞʓ>˜ÞÊëiVˆiÃʜvʓˆˆ«i`iðÊ/ iʓˆˆ«i`iÃʓ>˜Õv>VÌÕÀiÊ vʈ˜}iÃÌi`]ÊÌ ˆÃÊVœ“«œÕ˜`ʈÃʓiÌ>Lœˆâi`Ê̜ʫÀœ`ÕViʓ>˜- >˜`ÊÃ̜Àiʓ>˜`iœ˜ˆÌÀˆi]Ê>˜`ʈ˜Ê>ÊÃi«>À>ÌiÊVœ“«>À̓i˜Ì]ÊÌ iÞÊ `iœ˜ˆÌÀˆi]Ê >Ê VÞ>˜œ Þ`Àˆ˜]Ê Ü ˆV Ê ˆÃÊ Vœ˜ÛiÀÌi`Ê LÞÊ i˜âޓiÃÊ ˆ˜ÌœÊ Ã̜ÀiÊ i˜âޓiÃÊ Ì >ÌÊ >ÀiÊ V>«>LiÊ œvÊ V>Ì>Þ∘}Ê Ì iÊ Vœ˜ÛiÀȜ˜Ê œvÊ Li˜â>`i Þ`iÊ>˜`Ê Ê}>Ã]Ê>Ê̜݈VÊVœ“«œÕ˜`\ “>˜`iœ˜ˆÌÀˆiʈ˜ÌœÊLi˜â>`i Þ`iÊ>˜`Ê °Ê/œÊÜ>À`ʜvvÊ«Ài`>- ̜ÀÃ]Ê>ʓˆˆ«i`iÊ܈Ê“ˆÝÊÌ iÊVœ˜Ìi˜ÌÃʜvÊÌ iÊÌܜÊVœ“«>À̓i˜ÌÃÊ OH >˜`ÊÃiVÀiÌiÊ Ê}>ð HO OH HO O OH HO O O OH O OH O

C C H N N + HCN

Toxic Amygdalin Mandelonitrile Benzaldehyde

Wittig Reaction Georg Wittig, a German chemist, was awarded the 1979 Nobel Prize in Chemistry for his work with phosphorous compounds and his discovery of a reaction with enormous synthetic utility. Below is an example of this reaction, called the Wittig reaction (pronounced Vittig): H H O C H - + Ph C P Ph H Ph

This reaction can be used to convert a ketone into an alkene by forming a new C–C bond at the location of the carbonyl moiety. The phosphorus-containing reagent that accomplishes this transformation is called a phosphorane, and it belongs to a larger class of compounds called ylides. An ylide is a compound with two oppositely charged atoms adjacent to each

klein_c20_001-056v1.4.indd 36 30/04/10 16.47 20.10 Carbon Nucleophiles 37

other. The phosphorane above exhibits a negative charge on the carbon atom and a positive charge on the phosphorous atom. This ylide does, in fact, have a resonance structure that is free of any charges:

H + Ph H Ph -C P Ph C P Ph H Ph H Ph

However, this resonance structure (with a C5P double bond) does not contribute much character to the overall resonance hybrid, because the p orbitals on C and P are vastly differ- ent in size and do not effectively overlap. A similar argument was used in describing S5O bonds in the previous chapter (Section 19.3). Despite this fact, the phosphorus ylide above, also called a Wittig reagent, is often drawn using either of the resonance structures shown above. A mechanism for the Wittig reaction is shown in Mechanism 20.12.

MECHANISM 20.12 THE WITTIG REACTION

Nucleophilic attack Nucleophilic attack Rearrangement

H Ph – - + - Ph C P Ph H H O O O Ph O Ph H Ph P C + Ph Ph P P + Ph Ph C + Ph C Ph HH HH A Betaine An Oxaphosphetane The Wittig reagent A lone pair on the The oxaphosphetane functions as a nucleophile oxygen atom functions decomposes to produce and attacks the carbonyl group, as a nucleophile and attacks an alkene and forming a tetrahedral intermediate the phosphorus atom in triphenylphosphine oxide an intramolecular attack

BY THE WAY The Wittig reagent is a carbanion and can attack the carbonyl group in the first step of the Ý«iÀˆ“i˜Ì>Êiۈ`i˜ViÊ mechanism, generating an intermediate called a betaine (pronounced “bay-tuh-een”). A betaine ÃÕ}}iÃÌÃÊÌ >ÌÊÌ iÊ is a neutral compound with two oppositely charged atoms that are not adjacent to each other. ˆ˜ÌiÀ“i`ˆ>ÌiÊLiÌ>ˆ˜iʈÃʜ˜ÞÊ The negatively charged oxygen atom then attacks the positively charged phosphorous atom in vœÀ“i`ʈ˜Êˆ“ˆÌi`ÊV>ÃiðÊ˜Ê an intramolecular nucleophilic attack, generating an oxaphosphetane. This compound then œÌ iÀÊV>ÃiÃ]ʈÌÊ>««i>ÀÃÊÌ >ÌÊ rearranges to give the alkene product. Ì iÊ7ˆÌ̈}ÊÀi>}i˜Ìʓ>ÞÊÀi>VÌÊ Wittig reagents are easily prepared by treating triphenylphosphine with an alkyl halide fol- ÜˆÌ ÊÌ iÊV>ÀLœ˜ÞÊVœ“«œÕ˜`Ê ˆ˜Ê>ÊQÓ³ÓRÊVÞVœ>``ˆÌˆœ˜Ê lowed by a strong base: «ÀœViÃÃ]Ê`ˆÀiV̏ÞÊ}i˜iÀ>̈˜}Ê Ì iʜÝ>« œÃ« iÌ>˜i°Ê/ iÊ Ph Ph H I + - 1) CH3 “iV >˜ˆÃ“ÊvœÀÊÌ ˆÃÊÀi>V̈œ˜Ê Ph P Ph P C ˆÃÊÃ̈Ê՘`iÀʈ˜ÛiÃ̈}>̈œ˜° 2) BuLi Ph Ph H Triphenylphosphine Wittig reagent

klein_c20_001-056v1.4.indd 37 30/04/10 16.47 38 CHAPTER 20 Aldehydes and Ketones

The mechanism of formation for Wittig reagents involves an SN2 reaction followed by depro- tonation:

Ph Ph H - + Ph H H3CI + CH3CH2CH2CH2 Li + - Ph P Ph P C H Ph P C Ph Ph H Ph H Triphenylphosphine

Since the first step is an SN2 process, the regular restrictions of SN2 processes apply. Specifically, primary alkyl halides will react more readily than secondary alkyl halides, and tertiary alkyl halides cannot be used. The Wittig reaction is useful for preparing mono-, di-, or trisubstituted alkenes. Tetrasubstituted alkenes are more difficult to prepare due to steric hindrance in the transition states. The following exercise illustrates how to choose the reagents for a Wittig reaction.

SKILLBUILDER 20.6 PLANNING AN ALKENE SYNTHESIS WITH A WITTIG REACTION

LEARN the skill `i˜ÌˆvÞÊÌ iÊÀi>}i˜ÌÃʘiViÃÃ>ÀÞÊ̜ʫÀi«>ÀiÊÌ iÊVœ“«œÕ˜`ÊLiœÜÊÕȘ}Ê>Ê7ˆÌ̈}ÊÀi>V̈œ˜\

SOLUTION i}ˆ˜ÊLÞÊvœVÕȘ}ʜ˜ÊÌ iÊÌܜÊV>ÀLœ˜Ê>̜“ÃʜvÊÌ iÊ`œÕLiÊLœ˜`°Ê"˜iÊV>ÀLœ˜Ê>̜“Ê“ÕÃÌÊ >ÛiÊLii˜Ê>ÊV>ÀLœ˜ÞÊ}ÀœÕ«]ÊÜ ˆiÊÌ iÊœÌ iÀʓÕÃÌÊ >ÛiÊLii˜Ê>Ê7ˆÌ̈}ÊÀi>}i˜Ì°Ê/ ˆÃÊ}ˆÛiÃÊ ÌܜʫœÌi˜Ìˆ>ÊÀœÕÌiÃÊ̜ÊiÝ«œÀi\

STEP 1 1Ș}Ê>ÊÀiÌÀœÃÞ˜Ì ïVÊ >˜>ÞÈÃ]Ê`iÌiÀ“ˆ˜iÊÌ iÊ ÌܜʫœÃÈLiÊÃiÌÃʜvÊ Ài>VÌ>˜ÌÃÊÌ >ÌÊVœÕ`ÊLiÊ ÕÃi`Ê̜ÊvœÀ“ÊÌ iÊ 5 Ê Lœ˜`° H H O Ph3P O PPh + 3 +

Method 1 Method 2

i̽ÃÊVœ“«>ÀiÊÌ iÃiÊÌܜʓiÌ œ`ÃÊLÞÊvœVÕȘ}ʜ˜ÊÌ iÊ7ˆÌ̈}ÊÀi>}i˜Ìʈ˜Êi>V ÊV>Ãi°Ê,iV>ÊÌ >ÌÊ

Ì iÊ7ˆÌ̈}ÊÀi>}i˜ÌʈÃÊ«Ài«>Ài`ÊLÞÊ>˜Ê-NÓÊ«ÀœViÃÃ]Ê>˜`ÊÜiÊÌ iÀivœÀiʓÕÃÌÊVœ˜Ãˆ`iÀÊÃÌiÀˆVÊv>V- ̜ÀÃÊ`ÕÀˆ˜}ʈÌÃÊ«Ài«>À>̈œ˜°Ê iÌ œ`Ê£ÊÀiµÕˆÀiÃÊÌ iÊÕÃiʜvÊ>ÊÃiVœ˜`>ÀÞÊ>ŽÞÊ >ˆ`i\

X Ph3P 1) PPh3 2) BuLi

2° Alkyl halide

klein_c20_001-056v1.4.indd 38 30/04/10 16.47 20.10 Carbon Nucleophiles 39

LÕÌʓiÌ œ`ÊÓÊÀiµÕˆÀiÃÊÌ iÊÕÃiʜvÊ>Ê«Àˆ“>ÀÞÊ>ŽÞÊ >ˆ`i\

H STEP 2 œ˜Ãˆ`iÀÊ œÜÊޜÕÊ ÜœÕ`ʓ>ŽiÊi>V Ê X 1) PPh3 PPh3 «œÃÈLiÊ7ˆÌ̈}ÊÀi>}i˜Ì]Ê 2) BuLi >˜`Ê`iÌiÀ“ˆ˜iÊÜ ˆV Ê “iÌ œ`ʈ˜ÛœÛiÃÊÌ iʏiÃÃÊ 1° ÃÕLÃ̈ÌÕÌi`Ê>ŽÞÊ >ˆ`i° Alkyl halide

iÌ œ`Ê ÓÊ ˆÃÊ ˆŽiÞÊ ÌœÊ LiÊ “œÀiÊ ivwVˆi˜Ì]Ê LiV>ÕÃiÊ >Ê «Àˆ“>ÀÞÊ >ŽÞÊ >ˆ`iÊ ÜˆÊ ՘`iÀ}œÊ -NÓÊ “œÀiÊÀ>«ˆ`ÞÊÌ >˜Ê>ÊÃiVœ˜`>ÀÞÊ>ŽÞÊ >ˆ`i°Ê/ iÀivœÀi]ÊÌ iÊvœœÜˆ˜}ÊܜՏ`ÊLiÊÌ iÊ«ÀiviÀÀi`Ê ÃÞ˜Ì iÈÃ\

- O + P Ph Ph Ph

PRACTICE the skill 20.37 `i˜ÌˆvÞÊÌ iÊÀi>}i˜ÌÃʘiViÃÃ>ÀÞÊ̜ʫÀi«>ÀiÊi>V ʜvÊÌ iÊvœœÜˆ˜}ÊVœ“«œÕ˜`ÃÊÕȘ}Ê >Ê7ˆÌ̈}ÊÀi>V̈œ˜\

(a) (b) (c)

(d) (e)

APPLY the skill 20.38 œ˜Ãˆ`iÀÊÌ iÊÃÌÀÕVÌÕÀiʜvÊLiÌ>‡V>ÀœÌi˜i]ʓi˜Ìˆœ˜i`Êi>ÀˆiÀʈ˜ÊÌ ˆÃÊV >«ÌiÀ\

b-carotene

iÈ}˜Ê>ÊÃÞ˜Ì iÈÃʜvÊLiÌ>ÊV>ÀœÌi˜iÊÕȘ}ÊÌ iÊVœ“«œÕ˜`ÊLiœÜÊ>ÃÊޜÕÀʜ˜ÞÊÜÕÀViʜvÊV>À- Lœ˜Ê>̜“Ã\

Br

20.39 `i˜ÌˆvÞÊÌ iÊÀi>}i˜ÌÃʘiViÃÃ>ÀÞÊ̜Ê>VVœ“«ˆÃ Êi>V ʜvÊÌ iÊÌÀ>˜ÃvœÀ“>̈œ˜ÃÊLiœÜ\

OH

(a) (b)

need more PRACTICE? Try Problems 20.51–20.53

klein_c20_001-056v1.4.indd 39 30/04/10 16.47 40 CHAPTER 20 Aldehydes and Ketones

20.11 Baeyer-Villiger Oxidation of Aldehydes and Ketones

When treated with a peroxy acid, ketones can be con- O O RCO3H verted into esters via the insertion of an oxygen atom: R R R R O This reaction, discovered by Adolf von Baeyer and Victor Villiger in 1899, is called the Baeyer- Villiger oxidation. This process is believed to proceed via Mechanism 20.13.

MECHANISM 20.13 THE BAEYER-VILLIGER OXIDATION

Nucleophilic attack

- - H O O O Proton transfer O Rearrangement O O O R + O R O O O O + RR R R R O The peroxyacid H O A proton is transferred O The carbonyl group OH functions as a nucleophile from one location + H is reformed, R and attacks the carbonyl group, to another. with simultaneous forming a tetrahedral intermediate This step can occur migration of an intramolecularly, alkyl group because it would involve a five-membered transition state

The peroxy acid attacks the carbonyl group of the ketone, giving a tetrahedral intermediate that then undergoes an intramolecular proton transfer (or two successive intermolecular proton transfers). Finally, the C5O double bond is re-formed by migration of an R group. This rear- rangement produces the ester. In much the same way, treatment of a cyclic ketone with a peroxy acid yields a cyclic ester, or lactone.

O O

RCO3H O

A lactone

When an unsymmetrical ketone is treated with a peroxy acid, formation of the ester is regiose- lective; for example:

O O RCO3H O

In this case, the oxygen atom is inserted on the left side of the carbonyl group, rather than the right side. This occurs because the isopropyl group migrates more rapidly than the methyl group during the rearrangement step of the mechanism. The migration rates of different groups, or migratory aptitude, can be summarized as follows:

H > 3° > 2°, Ph > 1° > methyl

klein_c20_001-056v1.4.indd 40 30/04/10 16.47 20.12 Synthesis Strategies 41

A hydrogen atom will migrate more rapidly than a tertiary alkyl group, which will migrate more rapidly than a secondary alkyl group or phenyl group. Below is one more example that illustrates this concept: O O H O H RCO3H

In this example, the oxygen atom is inserted on the right side of the carbonyl, because the hydro- gen atom exhibits a greater migratory aptitude than the phenyl group.

CONCEPTUAL CHECKPOINT

20.40 *Ài`ˆVÌÊÌ iʓ>œÀÊ«Àœ`ÕVÌʜvÊi>V ÊÀi>V̈œ˜ÊLiœÜ\ O O O

H RCO3H ? RCO3H ? RCO3H ?

(a) (b) (c)

20.12 Synthesis Strategies

Recall from Chapter 12 that there are two main questions to ask when approaching a synthesis problem: 1. Is there any change in the carbon skeleton? 2. Is there any change in the ? Let’s focus on these issues separately, beginning with functional groups.

Functional Group Interconversion In previous chapters, we learned how to interconvert many different functional groups (Figure 20.9). The reactions in this chapter expand the playing field by opening up the frontier of alde- hydes and ketones. You should be able to fill in the reagents for each transformation in Figure 20.9. If you are having trouble, refer to Figure 13.13 for help. Then, you should be able to make a list of the various products than can be made from aldehydes and ketones and identify the required reagents in each case.

Reactions covered in this chapter O OH X

FIGURE 20.9 ՘V̈œ˜>Ê}ÀœÕ«ÃÊÌ >ÌÊV>˜ÊLiÊ ˆ˜ÌiÀVœ˜ÛiÀÌi`ÊÕȘ}ÊÀi>V̈œ˜ÃÊ Ì >ÌÊÜiÊ >Ûiʏi>À˜i`ÊÌ ÕÃÊv>À°

klein_c20_001-056v1.4.indd 41 30/04/10 16.47 42 CHAPTER 20 Aldehydes and Ketones

Reactions Involving a Change in Carbon Skeleton In this chapter, we have seen three C–C bond-forming reactions: (1) a Grignard reaction, (2) cyanohydrin formation, and (3) a Wittig reaction:

O C

H – + Ph 1) CH3MgBr KCN, HCN C P Ph 2) H2O H Ph

N H H C H3C OH C OH C C C

We have only seen one C–C bond-breaking reaction: the Baeyer-Villiger oxidation: O O RCO H C 3 C C C O

These four reactions should be added to your list of reactions that can change a carbon skeleton. Let’s get some practice using these reactions.

SKILLBUILDER 20.7 PROPOSING A SYNTHESIS

LEARN the skill *Àœ«œÃiÊ>˜ÊivwVˆi˜ÌÊÃÞ˜Ì iÈÃÊvœÀÊÌ iÊvœœÜˆ˜}ÊÊ ÌÀ>˜ÃvœÀ“>̈œ˜\

SOLUTION STEP 1 Ü>ÞÃÊLi}ˆ˜Ê>ÊÃÞ˜Ì iÈÃÊ«ÀœLi“ÊLÞÊ>Έ˜}ÊÌ iÊvœœÜˆ˜}ÊÌܜʵÕiÃ̈œ˜Ã\ ˜Ã«iVÌÊÜ iÌ iÀÊÌ iÀiÊ 1. ˆÃÊ>ÊV >˜}iʈ˜ÊÌ iÊ Is there any change in the carbon skeleton?Ê9iðÊ/ iÊ«Àœ`ÕVÌÊ >ÃÊÌܜÊ>``ˆÌˆœ˜>ÊV>ÀLœ˜Ê V>ÀLœ˜ÊÎii̜˜Ê>˜`ɜÀÊ >̜“ð >ÊV >˜}iʈ˜ÊÌ iʈ`i˜ÌˆÌÞÊ 2. Is there any change in the functional groups?Ê œ°Ê œÌ ÊÌ iÊÃÌ>À̈˜}ʓ>ÌiÀˆ>Ê>˜`ÊÌ iÊ œÀʏœV>̈œ˜ÊœvÊÌ iÊ «Àœ`ÕVÌÊ >ÛiÊ>Ê`œÕLiÊLœ˜`ʈ˜ÊÌ iÊiÝ>VÌÊÃ>“iʏœV>̈œ˜°ÊvÊÜiÊ`iÃÌÀœÞÊÌ iÊ`œÕLiÊLœ˜`ʈ˜Ê v՘V̈œ˜>Ê}ÀœÕ«Ã° Ì iÊ«ÀœViÃÃʜvÊ>``ˆ˜}ÊÌ iÊÌܜÊV>ÀLœ˜Ê>̜“Ã]ÊÜiÊ܈Ê˜ii`Ê̜ʓ>ŽiÊÃÕÀiÊÌ >ÌÊÜiÊ`œÊÜʈ˜Ê ÃÕV Ê>ÊÜ>ÞÊÌ >ÌÊÜiÊV>˜ÊÀiÃ̜ÀiÊÌ iÊ`œÕLiÊLœ˜`°

œÜʏi̽ÃÊVœ˜Ãˆ`iÀÊ œÜÊÜiʓˆ} Ìʈ˜ÃÌ>ÊÌ iÊ>``ˆÌˆœ˜>ÊÌܜ‡V>ÀLœ˜Ê>̜“Ã°Ê / iÊvœœÜˆ˜}Ê q ÊLœ˜`ʈÃÊÌ iʜ˜iÊÌ >Ìʘii`ÃÊ̜ÊLiʓ>`i\

˜ÊÌ ˆÃÊV >«ÌiÀ]ÊÜiÊ >ÛiÊÃii˜ÊÌ ÀiiÊ q ÊLœ˜`‡vœÀ“ˆ˜}ÊÀi>V̈œ˜Ã°Êi̽ÃÊVœ˜- È`iÀÊi>V ʜ˜iÊ>ÃÊ>Ê«œÃÈLˆˆÌÞ°

klein_c20_001-056v1.4.indd 42 30/04/10 16.47 20.12 Synthesis Strategies 43

STEP 2 7iÊV>˜Êˆ““i`ˆ>ÌiÞÊÀՏiʜÕÌÊVÞ>˜œ Þ`Àˆ˜ÊvœÀ“>̈œ˜]Ê>ÃÊÌ >ÌÊ«ÀœViÃÃʈ˜ÃÌ>Ãʜ˜Þʜ˜iÊ 7 i˜ÊÌ iÀiʈÃÊ>ÊV >˜}iÊ V>ÀLœ˜Ê>̜“]ʘœÌÊÌܜ°Ê-œÊi̽ÃÊVœ˜Ãˆ`iÀÊvœÀ“ˆ˜}ÊÌ iÊ q ÊLœ˜`ÊÜˆÌ ÊiˆÌ iÀÊ>ÊÀˆ}˜>À`ÊÀi>V- ˆ˜ÊÌ iÊV>ÀLœ˜ÊÎii̜˜]Ê Ìˆœ˜ÊœÀÊ>Ê7ˆÌ̈}ÊÀi>V̈œ˜° Vœ˜Ãˆ`iÀÊ>ÊœvÊÌ iÊ q Ê 5 Lœ˜`‡vœÀ“ˆ˜}ÊÀi>V̈œ˜ÃÊ ÊÀˆ}˜>À`ÊÀi>}i˜ÌÊܜ˜½ÌÊ>ÌÌ>VŽÊ>Ê Ê`œÕLiÊLœ˜`]ÊÜÊÕȘ}Ê>ÊÀˆ}˜>À`ÊÀi>V̈œ˜Ê >˜`Ê>ÊœvÊÌ iÊ q ÊLœ˜`‡ ܜՏ`ÊÀiµÕˆÀiÊwÀÃÌÊVœ˜ÛiÀ̈˜}ÊÌ iÊ 5 Ê`œÕLiÊLœ˜`ʈ˜ÌœÊ>Êv՘V̈œ˜>Ê}ÀœÕ«ÊÌ >ÌÊV>˜ÊLiÊ LÀi>Žˆ˜}ÊÀi>V̈œ˜ÃÊÌ >ÌÊ >ÌÌ>VŽi`ÊLÞÊ>ÊÀˆ}˜>À`ÊÀi>}i˜Ì]ÊÃÕV Ê>ÃÊ>ÊV>ÀLœ˜ÞÊ}ÀœÕ«\ ޜÕÊ >Ûiʏi>À˜i`ÊÜÊv>À° HO H O H

1) EtMgBr

2) H2O

/ ˆÃÊ Ài>V̈œ˜Ê V>˜Ê ˆ˜`ii`Ê LiÊ ÕÃi`Ê ÌœÊ vœÀ“Ê Ì iÊ VÀÕVˆ>Ê q Ê Lœ˜`°Ê /œÊ ÕÃiÊ Ì ˆÃÊ “iÌ œ`Ê œvÊ q ÊLœ˜`ÊvœÀ“>̈œ˜]ÊÜiʓÕÃÌÊwÀÃÌÊvœÀ“ÊÌ iʘiViÃÃ>ÀÞÊ>`i Þ`i]ÊÌ i˜Ê«iÀvœÀ“ÊÌ iÊÀˆ}˜>À`Ê Ài>V̈œ˜]Ê>˜`ÊÌ i˜Êw˜>ÞÊÀiÃ̜ÀiÊÌ iÊ`œÕLiÊLœ˜`ʈ˜ÊˆÌÃÊ«Àœ«iÀʏœV>̈œ˜°Ê/ ˆÃÊV>˜ÊLiÊ>VVœ“- «ˆÃ i`ÊÜˆÌ ÊÌ iÊvœœÜˆ˜}ÊÀi>}i˜ÌÃ\

# 1) BH3 THF

2) H2O2 , NaOH HO OH H O H

PCC 1) EtMgBr 1) TsCl, py

2) H2O 2) NaOEt, heat

C¬C bond-forming reaction / ˆÃÊ«ÀœÛˆ`iÃÊÕÃÊÜˆÌ Ê>ÊvœÕÀ‡ÃÌi«Ê«ÀœVi`ÕÀi]Ê>˜`ÊÌ ˆÃÊ>˜ÃÜiÀʈÃÊViÀÌ>ˆ˜ÞÊÀi>ܘ>Li° i̽ÃʘœÜÊiÝ«œÀiÊÌ iÊ«œÃÈLˆˆÌÞʜvÊ«Àœ«œÃˆ˜}Ê>ÊÃÞ˜Ì iÈÃÊÜˆÌ Ê>Ê7ˆÌ̈}Ê Ài>V̈œ˜°Ê,iV>ÊÌ >ÌÊ>Ê7ˆÌ̈}ÊÀi>V̈œ˜ÊV>˜ÊLiÊÕÃi`Ê̜ÊvœÀ“Ê>Ê 5 ÊLœ˜`]ÊÃœÊ ÜiÊvœVÕÃʜ˜ÊvœÀ“>̈œ˜ÊœvÊÌ ˆÃÊLœ˜`\ / ˆÃÊLœ˜`ÊV>˜ÊLiÊvœÀ“i`ʈvÊÜiÊÃÌ>ÀÌÊÜˆÌ Ê>ʎi̜˜iÊ>˜`ÊÕÃiÊÌ iÊvœœÜˆ˜}Ê7ˆÌ̈}Ê Ài>}i˜Ì\

O

PPh3

/œÊÕÃiÊÌ ˆÃÊÀi>V̈œ˜]ÊÜiʓÕÃÌÊwÀÃÌÊvœÀ“ÊÌ iʘiViÃÃ>ÀÞʎi̜˜iÊvÀœ“ÊÌ iÊÃÌ>À̈˜}Ê>Ži˜i\

O

/ ˆÃÊV>˜ÊLiÊ>VVœ“«ˆÃ i`ÊÜˆÌ Êœâœ˜œÞÈðÊ/ ˆÃÊ}ˆÛiÃÊ>ÊÌܜ‡ÃÌi«Ê«ÀœVi`ÕÀiÊvœÀÊ>VVœ“«ˆÃ - ˆ˜}ÊÌ iÊ`iÈÀi`ÊÌÀ>˜ÃvœÀ“>̈œ˜\ʜ✘œÞÈÃÊvœœÜi`ÊLÞÊ>Ê7ˆÌ̈}ÊÀi>V̈œ˜°Ê/ ˆÃÊ>««Àœ>V ʈÃÊ `ˆvviÀi˜ÌÊÌ >˜ÊœÕÀÊwÀÃÌÊ>˜ÃÜiÀ°Ê˜ÊÌ ˆÃÊ>««Àœ>V ]ÊÜiÊ>ÀiʘœÌÊ>ÌÌ>V ˆ˜}Ê>ÊÌܜ‡V>ÀLœ˜ÊV >ˆ˜]ÊLÕÌÊ À>Ì iÀ]ÊÜiÊ>ÀiÊwÀÃÌÊiÝ«iˆ˜}Ê>ÊV>ÀLœ˜Ê>̜“Ê>˜`ÊÌ i˜Ê>ÌÌ>V ˆ˜}Ê>ÊÌ Àii‡V>ÀLœ˜ÊV >ˆ˜°

klein_c20_001-056v1.4.indd 43 30/04/10 16.47 44 CHAPTER 20 Aldehydes and Ketones

˜Ê ÃՓ“>ÀÞ]Ê ÜiÊ >ÛiÊ `ˆÃVœÛiÀi`Ê ÌÜœÊ «>ÕÈLiÊ “iÌ œ`Ã°Ê œÌ Ê “iÌ œ`ÃÊ >ÀiÊ VœÀÀiVÌÊ >˜ÃÜiÀÃÊ̜ÊÌ ˆÃÊ«ÀœLi“]ÊLÕÌÊÌ iʓiÌ œ`Êi“«œÞˆ˜}ÊÌ iÊ7ˆÌ̈}ÊÀi>V̈œ˜ÊˆÃʏˆŽiÞÊ̜ÊLiʓœÀiÊ ivwVˆi˜Ì]ÊLiV>ÕÃiʈÌÊÀiµÕˆÀiÃÊviÜiÀÊÃÌi«Ã° # 1) BH3 THF

2) H2O2, NaOH 3) PCC 4) EtMgBr

5) H2O 6) TsCl, py 7) NaOEt, heat

1) O3 2) DMS

3) Ph3P

PRACTICE the skill 20.41 *Àœ«œÃiÊ>˜ÊivwVˆi˜ÌÊÃÞ˜Ì iÈÃÊvœÀÊi>V ʜvÊÌ iÊvœœÜˆ˜}ÊÌÀ>˜ÃvœÀ“>̈œ˜Ã\

OH

(a) (b)

OH OH

(c) (d) O

Br N

(e) (f)

O O O EtO OEt (g) O O

APPLY the skill 20.42 1Ș}Ê>˜ÞÊVœ“«œÕ˜`ÃʜvÊޜÕÀÊV œœÃˆ˜}]ʈ`i˜ÌˆvÞÊ>ʓiÌ œ`ÊvœÀÊ«Ài«>Àˆ˜}Êi>V ʜvÊ Ì iÊvœœÜˆ˜}ÊVœ“«œÕ˜`ðÊYour only limitation is that the compounds you use can have no more than two carbon atoms°ÊœÀÊ«ÕÀ«œÃiÃʜvÊVœÕ˜Ìˆ˜}ÊV>ÀLœ˜Ê>̜“Ã]ÊޜÕʓ>Þʈ}˜œÀiÊÌ iÊ « i˜ÞÊ}ÀœÕ«ÃʜvÊ>Ê7ˆÌ̈}ÊÀi>}i˜Ì°Ê/ >ÌʈÃ]ÊޜÕÊ>ÀiÊ«iÀ“ˆÌÌi`Ê̜ÊÕÃiÊ7ˆÌ̈}ÊÀi>}i˜Ìð

NH N O O O O

(a) (b) (c) (d)

HO N OH OH

HO H2N (e) (f) (g) (h) O

need more PRACTICE? Try Problems 20.55, 20.58, 20.67–20.69, 20.71, 20.75

klein_c20_001-056v1.4.indd 44 30/04/10 16.47 20.13 Spectroscopic Analysis of Aldehydes and Ketones 45

20.13 Spectroscopic Analysis of Aldehydes and Ketones

Aldehydes and ketones exhibit several characteristic signals in their infrared (IR) and nuclear magnetic resonance (NMR) spectra. We will now summarize these characteristic signals.

IR Signals The carbonyl group produces a strong signal in an IR spectrum, generally around 1715 or 1720 cm−1. However, a conjugated carbonyl will produce a signal at a lower wavenumber as a result of electron delocalization via resonance effects:

O O

LOOKING BACK œÀÊ>˜ÊiÝ«>˜>̈œ˜ÊœvÊÌ ˆÃÊ ivviVÌ]ÊÃiiÊ-iV̈œ˜Ê£x°Î° 1715 cm –1 1680 cm –1

Ring strain has the opposite effect on a carbonyl group. That is, increasing ring strain tends to increase the wavenumber of absorption:

O O O

1715 cm –1 1745 cm –1 1780 cm –1

Aldehydes generally exhibit one or two signals (C–H stretching) between 2700 and 2850 cm−1 (Figure 20.10).

100

80

2719 60 O H

40 % Transmittance

20 1726

0 FIGURE 20.10 4000 3500 3000 2500 2000 1500 1000 Ϫ ˜Ê,ÊëiVÌÀՓʜvÊ>˜Ê>`i Þ`i° Wavenumber (cm 1)

klein_c20_001-056v1.4.indd 45 30/04/10 16.47 46 CHAPTER 20 Aldehydes and Ketones

1H NMR Signals In a 1H NMR spectrum, the carbonyl group itself does not produce a signal. However, it has a pronounced effect on the chemical shift of neighboring protons. We saw in Section 16.5 that a carbonyl group adds +1 ppm to the chemical shift of its neighbors: O R R R R HH HH

~1.2 ppm ~2.2 ppm Aldehydic protons generally produce signals around 10 ppm. These signals can usually be iden- tified with relative ease, because very few signals appear that far downfield in a 1H NMR spec- trum (Figure 20.11).

O

H 5

1 2 2

FIGURE 20.11 10 9 8 7 6 5 4 3 2 Ê£Ê ,ÊëiVÌÀՓʜvÊ>˜Ê >`i Þ`i° Chemical Shift (ppm)

13C NMR Signals In a 13C NMR spectrum, the carbon atom of a carbonyl group will generally produce a weak signal near 200 ppm. This signal can often be identified with relative ease, because very few signals appear that far downfield in a 13C NMR spectrum (Figure 20.12).

O

209.1

220 200 180 160 140 120 100 80 60 40 20 0 FIGURE 20.12 Ê£Î Ê ,ÊëiVÌÀՓʜvÊ>ʎi̜˜i° Chemical Shift (ppm)

CONCEPTUAL CHECKPOINT

20.43 œ“«œÕ˜`Ê Ê >ÃÊ “œiVՏ>ÀÊ vœÀ“Տ>Ê £äH£ä"Ê >˜`Ê iÝ ˆLˆÌÃÊ >Ê •£ 1) [H+], HS SH ÃÌÀœ˜}ÊÈ}˜>Ê>ÌÊ£ÇÓäÊV“ ʈ˜ÊˆÌÃÊ,ÊëiVÌÀՓ°Ê/Ài>̓i˜ÌÊÜˆÌ Ê£]ӇiÌ - Compound A >˜i`ˆÌ ˆœÊvœœÜi`ÊLÞÊ,>˜iÞʘˆVŽiÊ>vvœÀ`ÃÊÌ iÊ«Àœ`ÕVÌÊà œÜ˜ÊLiœÜ°Ê 2) Raney Ni `i˜ÌˆvÞÊÌ iÊÃÌÀÕVÌÕÀiʜvÊVœ“«œÕ˜`Ê°

klein_c20_001-056v1.4.indd 46 30/04/10 16.47 Review of Concepts and Vocabulary 47

REVIEW OF REACTIONS SYNTHETICALLY USEFUL REACTIONS

1. Þ`À>ÌiÊœÀ“>̈œ˜ O [H+], H O RCO H 2. ViÌ>ÊœÀ“>̈œ˜ HO OH 2 3 O 1 15 3. ÞVˆVÊViÌ>Ê O [H+] H C PPh œÀ“>̈œ˜ ROH, –H2O 2 3 [H+] 4. ÞVˆVÊ/ ˆœ>Vi̜Ê RO OR OH HO 14 œÀ“>̈œ˜ 2 HCN, KCN + –H2O [H ] 5. iÃՏvÕÀˆâ>̈œ˜ OH SH 1) RMgBr HS 6. “ˆ˜iÊœÀ“>̈œ˜ O O 2) H2O 13 –H O 2 1) LAH CN 7. ˜>“ˆ˜iÊœÀ“>̈œ˜ 3 2) H2O OH [H+] [H+] RNH NH NH 8. "݈“iÊœÀ“>̈œ˜ 2 [H+] [H+] 2 2 –H O S S –H2O NH OH 2 12 9. R2NH 2 R Þ`À>✘iÊœÀ“>̈œ˜ –H2O –H2O OH 4 10. 7œvv‡ˆÃ ˜iÀÊ,i`ÕV̈œ˜ R N 11 11. ,i`ÕV̈œ˜ÊœvÊ>Êi̜˜i R R NH2 Raney N OH N 12. Àˆ}˜>À`Ê,i>V̈œ˜ Ni N 6 13. Þ>˜œ Þ`Àˆ˜ÊœÀ“>̈œ˜ 7 9 5 85% 14. 7ˆÌ̈}Ê,i>V̈œ˜ 8 HH 15. >iÞiÀ‡6ˆˆ}iÀÊ"݈`>̈œ˜ NaOH, H2O, heat

10

REVIEW OF CONCEPTS AND VOCABULARY

SECTION 20.1 UÊ / iÊ«œÃˆÌˆœ˜ÊœvÊiµÕˆˆLÀˆÕ“ʈÃÊ`i«i˜`i˜Ìʜ˜ÊÌ iÊ>LˆˆÌÞʜvÊÌ iÊ UÊ œÌ Ê>`i Þ`iÃÊ>˜`ʎi̜˜iÃÊVœ˜Ì>ˆ˜Ê>Êcarbonyl group]Ê>˜`Ê ˜ÕViœ« ˆiÊ̜Êv՘V̈œ˜Ê>ÃÊ>ʏi>ۈ˜}Ê}ÀœÕ«° LœÌ Ê>ÀiÊVœ““œ˜Êˆ˜Ê˜>ÌÕÀiÊ>˜`ʈ˜`ÕÃÌÀÞÊ>˜`ʜVVÕ«ÞÊ>ÊVi˜ÌÀ>Ê SECTION 20.5 Àœiʈ˜ÊœÀ}>˜ˆVÊV i“ˆÃÌÀÞ° UÊ 7 i˜Ê>˜Ê>`i Þ`iʜÀʎi̜˜iʈÃÊÌÀi>Ìi`ÊÜˆÌ ÊÜ>ÌiÀ]ÊÌ iÊV>À- SECTION 20.2 Lœ˜ÞÊ}ÀœÕ«ÊV>˜ÊLiÊVœ˜ÛiÀÌi`ʈ˜ÌœÊ>Êhydrate°Ê/ iÊiµÕˆˆL- UÊ / iÊÃÕvwÝʺ‡>»Êˆ˜`ˆV>ÌiÃÊ>˜Ê>`i Þ`ˆVÊ}ÀœÕ«]Ê>˜`ÊÌ iÊÃÕvwÝÊ ÀˆÕ“Ê}i˜iÀ>ÞÊv>ۜÀÃÊÌ iÊV>ÀLœ˜ÞÊ}ÀœÕ«]ÊiÝVi«Ìʈ˜ÊÌ iÊV>ÃiÊ º‡œ˜i»ÊˆÃÊÕÃi`ÊvœÀʎi̜˜ið œvÊ ÛiÀÞÊ Ãˆ“«iÊ >`i Þ`iÃ]Ê œÀÊ Ži̜˜iÃÊ ÜˆÌ Ê ÃÌÀœ˜}Ê iiVÌÀœ˜‡ UÊ ˜Ê˜>“ˆ˜}Ê>`i Þ`iÃÊ>˜`ʎi̜˜iÃ]ʏœV>˜ÌÃÊà œÕ`ÊLiÊ>ÃÈ}˜i`Ê ÜˆÌ `À>܈˜}ÊÃÕLÃ̈ÌÕi˜Ìð ÜÊ>ÃÊ̜Ê}ˆÛiÊÌ iÊV>ÀLœ˜ÞÊ}ÀœÕ«ÊÌ iʏœÜiÃÌʘՓLiÀÊ«œÃÈLi° UÊ ˜Ê>Vˆ`ˆVÊVœ˜`ˆÌˆœ˜Ã]Ê>˜Ê>`i Þ`iʜÀʎi̜˜iÊ܈ÊÀi>VÌÊÜˆÌ ÊÌÜœÊ “œiVՏiÃʜvÊ>Vœ œÊ̜ÊvœÀ“Ê>˜Êacetal° SECTION 20.3 UÊ ˜ÊÌ iÊ«ÀiÃi˜ViʜvÊ>˜Ê>Vˆ`]ÊÌ iÊV>ÀLœ˜ÞÊ}ÀœÕ«ÊˆÃÊ«ÀœÌœ˜>Ìi`Ê UÊ `i Þ`iÃÊV>˜ÊLiÊ«Ài«>Ài`Êۈ>ʜ݈`>̈œ˜ÊœvÊ«Àˆ“>ÀÞÊ>Vœ œÃ]Ê ÌœÊvœÀ“Ê>ÊÛiÀÞÊ«œÜiÀvՏÊiiVÌÀœ« ˆi° œâœ˜œÞÈÃʜvÊ>Ži˜iÃ]ʜÀÊ Þ`ÀœLœÀ>̈œ˜‡œÝˆ`>̈œ˜ÊœvÊÌiÀ“ˆ˜>Ê UÊ / iʓiV >˜ˆÃ“ÊvœÀÊ>ViÌ>ÊvœÀ“>̈œ˜ÊV>˜ÊLiÊ`ˆÛˆ`i`ʈ˜ÌœÊÌÜœÊ >ŽÞ˜ið «>ÀÌÃ\ UÊ i̜˜iÃÊ V>˜Ê LiÊ «Ài«>Ài`Ê Ûˆ>Ê œÝˆ`>̈œ˜Ê œvÊ ÃiVœ˜`>ÀÞÊ >Vœ- 1.Ê / iÊwÀÃÌÊÌ ÀiiÊÃÌi«ÃÊ«Àœ`ÕViÊ>Êhemiacetal° œÃ]ʜ✘œÞÈÃʜvÊ>Ži˜iÃ]Ê>Vˆ`‡V>Ì>Þâi`Ê Þ`À>̈œ˜ÊœvÊÌiÀ“ˆ- ˜>Ê>ŽÞ˜iÃ]ʜÀÊÀˆi`i‡ À>vÌÃÊ>Vޏ>̈œ˜° 2.Ê / iʏ>ÃÌÊvœÕÀÊÃÌi«ÃÊVœ˜ÛiÀÌÊÌ iÊ i“ˆ>ViÌ>Ê̜Ê>˜Ê>ViÌ>° UÊ œÀÊ “>˜ÞÊ Ãˆ“«iÊ >`i Þ`iÃ]Ê Ì iÊ iµÕˆˆLÀˆÕ“Ê v>ۜÀÃÊ vœÀ“>- SECTION 20.4 ̈œ˜ÊœvÊÌ iÊ>ViÌ>ÆÊ œÜiÛiÀ]ÊvœÀʓœÃÌʎi̜˜iÃ]ÊÌ iÊiµÕˆˆLÀˆÕ“Ê UÊ / iÊ iiVÌÀœ« ˆˆVˆÌÞÊ œvÊ >Ê V>ÀLœ˜ÞÊ }ÀœÕ«Ê `iÀˆÛiÃÊ vÀœ“Ê ÀiÜ- v>ۜÀÃÊÀi>VÌ>˜ÌÃÊÀ>Ì iÀÊÌ >˜Ê«Àœ`ÕVÌð ˜>˜ViÊivviVÌÃÊ>ÃÊÜiÊ>Ãʈ˜`ÕV̈ÛiÊivviVÌð UÊ ˜Ê>`i Þ`iʜÀʎi̜˜iÊ܈ÊÀi>VÌÊÜˆÌ Êœ˜iʓœiVՏiʜvÊ>Ê`ˆœÊ UÊ `i Þ`iÃÊ>ÀiʓœÀiÊÀi>V̈ÛiÊÌ >˜ÊŽi̜˜iÃÊ>ÃÊ>ÊÀiÃՏÌʜvÊÃÌiÀˆVÊ ÌœÊvœÀ“Ê>ÊVÞVˆVÊ>ViÌ>° ivviVÌÃÊ>˜`ÊiiVÌÀœ˜ˆVÊivviVÌð UÊ / iÊÀiÛiÀÈLˆˆÌÞʜvÊ>ViÌ>ÊvœÀ“>̈œ˜Êi˜>LiÃÊ>ViÌ>ÃÊ̜Êv՘V- UÊ Ê}i˜iÀ>Ê“iV >˜ˆÃ“ÊvœÀʘÕViœ« ˆˆVÊ>``ˆÌˆœ˜Ê՘`iÀÊL>ÈVÊ Ìˆœ˜Ê>ÃÊ«ÀœÌiV̈˜}Ê}ÀœÕ«ÃÊvœÀʎi̜˜iÃʜÀÊ>`i Þ`iðÊViÌ>ÃÊ Vœ˜`ˆÌˆœ˜Ãʈ˜ÛœÛiÃÊÌܜÊÃÌi«Ã\ >ÀiÊÃÌ>LiÊ՘`iÀÊÃÌÀœ˜}ÞÊL>ÈVÊVœ˜`ˆÌˆœ˜Ã° 1.Ê ÕViœ« ˆˆVÊ>ÌÌ>VŽÊ̜Ê}i˜iÀ>ÌiÊ>Êtetrahedral intermediate° UÊ i“ˆ>ViÌ>ÃÊ>ÀiÊ}i˜iÀ>ÞÊ`ˆvwVՏÌÊ̜ʈ܏>ÌiÊ՘iÃÃÊÌ iÞÊ>ÀiÊ 2.Ê *ÀœÌœ˜ÊÌÀ>˜ÃviÀ VÞVˆV°

klein_c20_001-056v1.4.indd 47 30/04/10 16.47 48 CHAPTER 20 Aldehydes and Ketones

SECTION 20.6 UÊ Àˆ}˜>À`ÊÀi>V̈œ˜ÃÊ>ÀiʘœÌÊÀiÛiÀÈLi]ÊLiV>ÕÃiÊV>ÀL>˜ˆœ˜ÃÊ`œÊ UÊ ˜Ê>Vˆ`ˆVÊVœ˜`ˆÌˆœ˜Ã]Ê>˜Ê>`i Þ`iʜÀʎi̜˜iÊ܈ÊÀi>VÌÊÜˆÌ Ê>Ê ˜œÌÊv՘V̈œ˜Ê>Ãʏi>ۈ˜}Ê}ÀœÕ«Ã° «Àˆ“>ÀÞÊ>“ˆ˜iÊ̜ÊvœÀ“Ê>˜Êimine° UÊ 7 i˜ÊÌÀi>Ìi`ÊÜˆÌ Ê Þ`Àœ}i˜ÊVÞ>˜ˆ`iÊ­ ®]Ê>`i Þ`iÃÊ>˜`Ê UÊ / iÊwÀÃÌÊÌ ÀiiÊÃÌi«Ãʈ˜Êˆ“ˆ˜iÊvœÀ“>̈œ˜Ê«Àœ`ÕViÊ>Êcarbinol- Ži̜˜iÃÊ >ÀiÊ Vœ˜ÛiÀÌi`Ê ˆ˜ÌœÊ cyanohydrins°Ê œÀÊ “œÃÌÊ >`i- amine]Ê >˜`Ê Ì iÊ >ÃÌÊ Ì ÀiiÊ ÃÌi«ÃÊ Vœ˜ÛiÀÌÊ Ì iÊ V>ÀLˆ˜œ>“ˆ˜iÊ Þ`iÃÊ>˜`Ê՘ ˆ˜`iÀi`ʎi̜˜iÃ]ÊÌ iÊiµÕˆˆLÀˆÕ“Êv>ۜÀÃÊvœÀ“>- ˆ˜ÌœÊ>˜Êˆ“ˆ˜i° ̈œ˜ÊœvÊÌ iÊVÞ>˜œ Þ`Àˆ˜°

UÊ >˜ÞÊ`ˆvviÀi˜ÌÊVœ“«œÕ˜`ÃʜvÊÌ iÊvœÀ“Ê, ÓÊ܈ÊÀi>VÌÊÜˆÌ Ê UÊ / iÊWittig reactionÊV>˜ÊLiÊÕÃi`Ê̜ÊVœ˜ÛiÀÌÊ>ʎi̜˜iÊ̜Ê>˜Ê >`i Þ`iÃÊ>˜`ʎi̜˜iÃÆÊvœÀÊiÝ>“«i\ >Ži˜i°Ê/ iÊWittig reagentÊÌ >ÌÊ>VVœ“«ˆÃ iÃÊÌ ˆÃÊÌÀ>˜ÃvœÀ- “>̈œ˜ÊˆÃÊV>i`Ê>Êphosphorane]ÊÜ ˆV ÊLiœ˜}ÃÊ̜Ê>ʏ>À}iÀÊ 1.Ê 7 i˜Ê Þ`À>∘iÊ ˆÃÊ ÕÃi`Ê >ÃÊ >Ê ˜ÕViœ« ˆiÊ ­ ÓNHÓ®]Ê >Ê hydrazoneʈÃÊvœÀ“i`° V>ÃÃʜvÊVœ“«œÕ˜`ÃÊV>i`Êylides°

2.Ê 7 i˜Ê Þ`ÀœÝޏ>“ˆ˜iʈÃÊÕÃi`Ê>ÃÊ>ʘÕViœ« ˆiÊ­ Ó"®]Ê UÊ / iʓiV >˜ˆÃ“ÊœvÊ>Ê7ˆÌ̈}ÊÀi>V̈œ˜Êˆ˜ÛœÛiÃʈ˜ˆÌˆ>ÊvœÀ“>̈œ˜Ê >˜ÊoximeʈÃÊvœÀ“i`° œvÊ>Êbetaine]ÊÜ ˆV Ê՘`iÀ}œiÃÊ>˜Êˆ˜ÌÀ>“œiVՏ>ÀʘÕViœ« ˆˆVÊ oxaphosphetane UÊ ˜Ê >Vˆ`ˆVÊ Vœ˜`ˆÌˆœ˜Ã]Ê >˜Ê >`i Þ`iÊ œÀÊ Ži̜˜iÊ ÜˆÊ Ài>VÌÊ ÜˆÌ Ê >ÌÌ>VŽ]Ê }i˜iÀ>̈˜}Ê >˜Ê °Ê ,i>ÀÀ>˜}i“i˜ÌÊ >ÊÃiVœ˜`>ÀÞÊ>“ˆ˜iÊ̜ÊvœÀ“Ê>˜Êenamine°Ê/ iʓiV >˜ˆÃ“ÊœvÊ }ˆÛiÃÊÌ iÊ«Àœ`ÕVÌ° i˜>“ˆ˜iÊvœÀ“>̈œ˜ÊˆÃʈ`i˜ÌˆV>Ê̜ÊÌ iʓiV >˜ˆÃ“Êœvʈ“ˆ˜iÊvœÀ- UÊ *Ài«>À>̈œ˜ÊœvÊ7ˆÌ̈}ÊÀi>}i˜ÌÃʈ˜ÛœÛiÃÊ>˜Ê-NÓÊÀi>V̈œ˜]Ê>˜`Ê “>̈œ˜ÊiÝVi«ÌÊvœÀÊÌ iʏ>ÃÌÊÃÌi«° Ì iÊÀi}Տ>ÀÊÀiÃÌÀˆV̈œ˜ÃʜvÊ-NÓÊ«ÀœViÃÃiÃÊ>««Þ° UÊ ˜ÊÌ iÊWolff-Kishner reduction]Ê>Ê Þ`À>✘iʈÃÊÀi`ÕVi`Ê̜Ê>˜Ê SECTION 20.11 >Ž>˜iÊ՘`iÀÊÃÌÀœ˜}ÞÊL>ÈVÊVœ˜`ˆÌˆœ˜Ã° UÊ ÊBaeyer-Villiger oxidationÊVœ˜ÛiÀÌÃÊ>ʎi̜˜iÊ̜Ê>˜ÊiÃÌiÀÊLÞÊ SECTION 20.7 ˆ˜ÃiÀ̈˜}Ê>˜ÊœÝÞ}i˜Ê>̜“ʘiÝÌÊ̜ÊÌ iÊV>ÀLœ˜ÞÊ}ÀœÕ«°Ê ÞVˆVÊ Ži̜˜iÃÊ«Àœ`ÕViÊVÞVˆVÊiÃÌiÀÃÊV>i`Êlactones° UÊ ˜Ê>Vˆ`ˆVÊVœ˜`ˆÌˆœ˜Ã]Ê>ÊÀi>}i˜ÌÃ]ʈ˜ÌiÀ“i`ˆ>ÌiÃ]Ê>˜`ʏi>ۈ˜}Ê }ÀœÕ«ÃÊiˆÌ iÀÊà œÕ`ÊLiʘiÕÌÀ>Ê­˜œÊV >À}i®ÊœÀÊà œÕ`ÊLi>ÀÊ UÊ 7 i˜Ê>˜Ê՘Ãޓ“iÌÀˆV>ÊŽi̜˜iʈÃÊÌÀi>Ìi`ÊÜˆÌ Ê>Ê«iÀœÝÞÊ>Vˆ`]Ê œ˜iÊ«œÃˆÌˆÛiÊV >À}i° vœÀ“>̈œ˜Ê œvÊ Ì iÊ iÃÌiÀÊ ˆÃÊ Ài}ˆœÃiiV̈Ûi]Ê >˜`Ê Ì iÊ «Àœ`ÕVÌÊ ˆÃÊ migratory aptitude UÊ HydrolysisÊ œvÊ >ViÌ>Ã]Ê ˆ“ˆ˜iÃ]Ê >˜`Ê i˜>“ˆ˜iÃÊ Õ˜`iÀÊ >Vˆ`ˆVÊ `iÌiÀ“ˆ˜i`ÊLÞÊÌ iÊ ÊœvÊi>V Ê}ÀœÕ«Ê˜iÝÌÊ Vœ˜`ˆÌˆœ˜ÃÊ«Àœ`ÕViÃʎi̜˜iÃʜÀÊ>`i Þ`ið ̜ÊÌ iÊV>ÀLœ˜Þ° SECTION 20.12 SECTION 20.8 UÊ / ˆÃÊ V >«ÌiÀÊ iÝ«œÀi`Ê Ì ÀiiÊ q Ê Lœ˜`‡vœÀ“ˆ˜}Ê Ài>V̈œ˜Ã\Ê UÊ ˜Ê>Vˆ`ˆVÊVœ˜`ˆÌˆœ˜Ã]Ê>˜Ê>`i Þ`iʜÀʎi̜˜iÊ܈ÊÀi>VÌÊÜˆÌ ÊÌÜœÊ ­£®Ê>ÊÀˆ}˜>À`ÊÀi>V̈œ˜]Ê­Ó®ÊVÞ>˜œ Þ`Àˆ˜ÊvœÀ“>̈œ˜]Ê>˜`ʭήÊ>Ê iµÕˆÛ>i˜ÌÃÊ œvÊ >Ê Ì ˆœÊ ÌœÊ vœÀ“Ê >Ê thioacetal°Ê vÊ >Ê Vœ“«œÕ˜`Ê 7ˆÌ̈}ÊÀi>V̈œ˜° ÜˆÌ ÊÌܜÊ-Ê}ÀœÕ«ÃʈÃÊÕÃi`]Ê>ÊVÞVˆVÊÌ ˆœ>ViÌ>ÊˆÃÊvœÀ“i`° UÊ / ˆÃÊV >«ÌiÀÊiÝ«œÀi`ʜ˜Þʜ˜iÊ q ÊLœ˜`‡LÀi>Žˆ˜}ÊÀi>V̈œ˜\Ê UÊ 7 i˜ÊÌÀi>Ìi`ÊÜˆÌ Ê,>˜iÞʘˆVŽi]ÊÌ ˆœ>ViÌ>ÃÊ՘`iÀ}œÊdesul- Ì iÊ >iÞiÀ‡6ˆˆ}iÀʜ݈`>̈œ˜° furizationÊ̜Êވi`Ê>ʓiÌ Þi˜iÊ}ÀœÕ«° SECTION 20.13 SECTION 20.9 UÊ >ÀLœ˜ÞÊ }ÀœÕ«ÃÊ «Àœ`ÕViÊ >Ê ÃÌÀœ˜}Ê ,Ê Ãˆ}˜>Ê >ÀœÕ˜`Ê UÊ 7 i˜ÊÌÀi>Ìi`ÊÜˆÌ Ê>Ê Þ`Àˆ`iÊÀi`ÕVˆ˜}Ê>}i˜Ì]ÊÃÕV Ê>ÃʏˆÌ ˆÕ“Ê £Ç£xÊ V“•£°Ê Ê Vœ˜Õ}>Ìi`Ê V>ÀLœ˜ÞÊ «Àœ`ÕViÃÊ >Ê Ãˆ}˜>Ê >ÌÊ >Ê >Õ“ˆ˜Õ“Ê Þ`Àˆ`iÊ ­®Ê œÀÊ Ãœ`ˆÕ“Ê LœÀœ Þ`Àˆ`iÊ ­ > {®]Ê œÜiÀÊÜ>Ûi˜Õ“LiÀ]ÊÜ ˆiÊÀˆ˜}ÊÃÌÀ>ˆ˜Êˆ˜VÀi>ÃiÃÊÌ iÊÜ>Ûi˜Õ“- >`i Þ`iÃÊ>˜`ʎi̜˜iÃÊ>ÀiÊÀi`ÕVi`Ê̜Ê>Vœ œÃ° LiÀʜvÊ>LÜÀ«Ìˆœ˜° / iÊÀi`ÕV̈œ˜ÊœvÊ>ÊV>ÀLœ˜ÞÊ}ÀœÕ«ÊÜˆÌ ÊʜÀÊ >  ʈÃʘœÌÊ UÊ { UÊ `i Þ`ˆVÊ qÊ Lœ˜`ÃÊ iÝ ˆLˆÌÊ œ˜iÊ œÀÊ ÌÜœÊ Ãˆ}˜>ÃÊ LiÌÜii˜Ê >ÊÀiÛiÀÈLiÊ«ÀœViÃÃ]ÊLiV>ÕÃiÊ Þ`Àˆ`iÊ`œiÃʘœÌÊv՘V̈œ˜Ê>ÃÊ>Ê ÓÇääÊ>˜`ÊÓnxäÊV“•£° i>ۈ˜}Ê}ÀœÕ«° UÊ ˜Ê>Ê £Ê ,ÊëiVÌÀՓ]Ê>ÊV>ÀLœ˜ÞÊ}ÀœÕ«Ê>``Ãʳ£Ê««“ÊÌœÊ SECTION 20.10 Ì iÊV i“ˆV>Êà ˆvÌʜvʈÌÃʘiˆ} LœÀÃ]Ê>˜`Ê>˜Ê>`i Þ`ˆVÊ«ÀœÌœ˜Ê UÊ 7 i˜ÊÌÀi>Ìi`ÊÜˆÌ Ê>ÊÀˆ}˜>À`Ê>}i˜Ì]Ê>`i Þ`iÃÊ>˜`ʎi̜˜iÃÊ «Àœ`ÕViÃÊ>ÊÈ}˜>Ê>ÀœÕ˜`Ê£äÊ««“° >ÀiÊVœ˜ÛiÀÌi`ʈ˜ÌœÊ>Vœ œÃ]Ê>VVœ“«>˜ˆi`ÊLÞÊÌ iÊvœÀ“>̈œ˜Ê UÊ ˜Ê>Ê£Î Ê ,ÊëiVÌÀՓ]Ê>ÊV>ÀLœ˜ÞÊ}ÀœÕ«Ê«Àœ`ÕViÃÊ>ÊÜi>ŽÊ œvÊ>ʘiÜÊ q ÊLœ˜`° È}˜>Ê˜i>ÀÊÓääÊ««“°

KEY TERMINOLOGY

acetalÊ UUU `iÃՏvÕÀˆâ>̈œ˜Ê UUU ˆ“ˆ˜iÊ UUU ÌiÌÀ> i`À>Êˆ˜ÌiÀ“i`ˆ>ÌiÊ UUU >iÞiÀ‡6ˆˆ}iÀʜ݈`>̈œ˜Ê UUU i˜>“ˆ˜iÊ UUU >V̜˜iÊ UUU Ì ˆœ>ViÌ>Ê UUU LiÌ>ˆ˜iÊ UUU i“ˆ>ViÌ>Ê UUU “ˆ}À>̜ÀÞÊ>«ÌˆÌÕ`iÊ UUU 7ˆÌ̈}ÊÀi>V̈œ˜Ê UUU V>ÀLˆ˜œ>“ˆ˜iÊ UUU Þ`À>ÌiÊ UUU œÝ>« œÃ« iÌ>˜iÊ UUU 7ˆÌ̈}ÊÀi>}i˜ÌÊ UUU V>ÀLœ˜ÞÊ}ÀœÕ«Ê UUU Þ`À>✘iÊ UUU œÝˆ“iÊ UUU 7œvv‡ˆÃ ˜iÀÊÀi`ÕV̈œ˜Ê UUU VÞ>˜œ Þ`Àˆ˜ÃÊ UUU Þ`ÀœÞÈÃÊ UUU « œÃ« œÀ>˜iÊ UUU ޏˆ`iÊ UUU

klein_c20_001-056v1.4.indd 48 30/04/10 16.47 Skillbuilder Review 49

SKILLBUILDER REVIEW

20.1 NAMING ALDEHYDES AND KETONES

STEP 1 œœÃiÊÌ iʏœ˜}iÃÌÊ STEP 2 AND 3 `i˜ÌˆvÞÊ STEP 4 ÃÃi“LiÊÌ iÊ STEP 5 ÃÈ}˜ÊÌ iÊ V >ˆ˜ÊVœ˜Ì>ˆ˜ˆ˜}ÊÌ iÊV>ÀLœ˜ÞÊ Ì iÊÃÕLÃ̈ÌÕi˜ÌÃ]Ê>˜`Ê>ÃÈ}˜Ê ÃÕLÃ̈ÌÕi˜ÌÃÊ>« >LïV>Þ° Vœ˜w}ÕÀ>̈œ˜ÊœvÊ>˜ÞÊV ˆÀ>ˆÌÞÊ }ÀœÕ«]Ê>˜`ʘՓLiÀÊÌ iÊV >ˆ˜Ê œV>˜Ìð Vi˜ÌiÀ° ÃÌ>À̈˜}ÊvÀœ“ÊÌ iÊi˜`ÊVœÃiÃÌÊ ÌœÊÌ iÊV>ÀLœ˜ÞÊ}ÀœÕ«°

8 4,4-dimethyl 8 79 79

2 46 2 46 R 1 1 O O 3 5 3 5 6-ethyl 4,4-dimethyl-6-ethyl (R)-4,4-dimethyl-6-ethyl O 3-nonanone O -3-nonanone Try problems 20.1–20.4, 20.44–20.49 20.2 DRAWING THE MECHANISM OF ACETAL FORMATION

STEP 1 À>ÜÊÌ iÊÌ ÀiiÊÃÌi«ÃʘiViÃÃ>ÀÞÊvœÀÊ i“ˆ>ViÌ>ÊvœÀ“>̈œ˜° COMMENTS

Proton transfer Nucleophilic attack Proton transfer UÊ ÛiÀÞÊÃÌi«Ê >ÃÊÌܜÊVÕÀÛi`Ê>ÀÀœÜÃ°Ê À>ÜÊÌ i“Ê«ÀiVˆÃiÞ°

+ H UÊ œÊ˜œÌÊvœÀ}iÌÊÌ iÊV >À}ið O O + OH OH UÊ À>ÜÊi>V ÊÃÌi«ÊÃi«>À>ÌiÞ° H¬A R¬O¬H A H O+ OR R

STEP 2 À>ÜÊÌ iÊvœÕÀÊÃÌi«ÃÊÌ >ÌÊVœ˜ÛiÀÌÊÌ iÊ i“ˆ>ViÌ>Êˆ˜ÌœÊ>˜Ê>ViÌ>°

Proton transfer Loss of a leaving group Nucleophilic attack Proton transfer

H + H R R + H OH + O – + R¬O¬H H¬A H2O O O A OR OR OR OR OR

Try Problems 20.8–20.10, 20.57, 20.62, 20.67

20.3 DRAWING THE MECHANISM OF IMINE FORMATION

STEP 1 À>ÜÊÌ iÊÌ ÀiiÊÃÌi«ÃʘiViÃÃ>ÀÞÊvœÀÊV>ÀLˆ˜œ>“ˆ˜iÊvœÀ“>̈œ˜° COMMENTS UÊ >V Ê«>ÀÌʜvÊÌ iʓiV >˜ˆÃ“ÊLi}ˆ˜ÃÊ Proton transfer Nucleophilic attack Proton transfer ÜˆÌ Ê>Ê«ÀœÌœ˜ÊÌÀ>˜ÃviÀÊ>˜`Êi˜`ÃÊ ÜˆÌ Ê>Ê«ÀœÌœ˜ÊÌÀ>˜ÃviÀ° + H UÊ ÛiÀÞÊÃÌi«Ê >ÃÊÌܜÊVÕÀÛi`Ê>ÀÀœÜÃ°Ê O O OH OH + ¬ H¬A R NH2 A >ŽiÊÃÕÀiÊ̜Ê`À>ÜÊÌ i“Ê«ÀiVˆÃiÞ° H H UÊ œÊ˜œÌÊvœÀ}iÌÊÌ iÊ«œÃˆÌˆÛiÊV >À}iÃ°Ê N N + / iÀiÊà œÕ`ÊLiʘœÊ˜i}>̈ÛiÊ RH R V >À}ið STEP 2 À>ÜÊÌ iÊÌ ÀiiÊÃÌi«ÃÊÌ >ÌÊVœ˜ÛiÀÌÊÌ iÊV>ÀLˆ˜œ>“ˆ˜iʈ˜ÌœÊ>˜Êˆ“ˆ˜i° UÊ À>ÜÊi>V ÊÃÌi«ÊÃi«>À>ÌiÞ]Ê vœœÜˆ˜}ÊÌ iÊ«ÀiVˆÃiʜÀ`iÀʜvÊ ÃÌi«Ã° Proton transfer Loss of a leaving group Proton transfer

H + H R + H R OH + O – N N H¬A H2O R–NH2 H H N N R R Try Problems 20.15–20.18, 20.72, 20.86

klein_c20_001-056v1.4.indd 49 30/04/10 16.47 50 CHAPTER 20 Aldehydes and Ketones

20.4 DRAWING THE MECHANISM OF ENAMINE FORMATION

STEP 1 À>ÜÊÌ iÊÌ ÀiiÊÃÌi«ÃʘiViÃÃ>ÀÞÊvœÀÊV>ÀLˆ˜œ>“ˆ˜iÊvœÀ“>̈œ˜° COMMENTS UÊ >V Ê«>ÀÌʜvÊÌ iʓiV >˜ˆÃ“Ê Proton transfer Nucleophilic attack Proton transfer Li}ˆ˜ÃÊÜˆÌ Ê>Ê«ÀœÌœ˜ÊÌÀ>˜ÃviÀÊ >˜`Êi˜`ÃÊÜˆÌ Ê>Ê«ÀœÌœ˜Ê + H ÌÀ>˜ÃviÀ° OO OH OH + H¬A R2NH A UÊ ÛiÀÞÊÃÌi«Ê >ÃÊÌܜÊVÕÀÛi`Ê H R >ÀÀœÜÃ°Ê >ŽiÊÃÕÀiÊ̜Ê`À>ÜÊ N N + Ì i“Ê«ÀiVˆÃiÞ° R R R UÊ œÊ˜œÌÊvœÀ}iÌÊÌ iÊ«œÃˆÌˆÛiÊ V >À}iðÊ/ iÀiÊà œÕ`ÊLiʘœÊ STEP 2 À>ÜÊÌ iÊÌ ÀiiÊÃÌi«ÃÊÌ >ÌÊVœ˜ÛiÀÌÊÌ iÊV>ÀLˆ˜œ>“ˆ˜iʈ˜ÌœÊ>˜Êi˜>“ˆ˜i° ˜i}>̈ÛiÊV >À}ið UÊ À>ÜÊi>V ÊÃÌi«ÊÃi«>À>ÌiÞÊ Proton transferLoss of a leaving group Nucleophilic attack vœœÜˆ˜}ÊÌ iÊ«ÀiVˆÃiʜÀ`iÀʜvÊ ÃÌi«Ã° H + H R + R R R OH + O – R NH H¬A H2O N 2 N R R N N H R R Try Problems 20.21–20.24, 20.72

20.5 DRAWING THE MECHANISM OF A HYDROLYSIS REACTION

STEP 1 7œÀŽˆ˜}ÊL>VŽÜ>À`Ã]Ê`À>ÜÊ>Êˆ˜ÌiÀ“i`ˆ>Ìið STEP 2 first draw this intermediate UÊvÌiÀÊ`À>܈˜}Ê>Êˆ˜ÌiÀ“i`ˆ>ÌiÃ]ÊÌ i˜Ê`À>ÜÊ>Ê + H Ài>}i˜ÌÃÊ>˜`ÊVÕÀÛi`Ê>ÀÀœÜÃÊÕȘ}ÊÌ iÊvœœÜˆ˜}ÊÀՏiÃ\ O O UʘÊ>Vˆ`ˆVÊVœ˜`ˆÌˆœ˜Ã]Ê>ÊÀi>}i˜ÌÃ]ʈ˜ÌiÀ“i`ˆ>ÌiÃ]Ê>˜`Ê i>ۈ˜}Ê}ÀœÕ«ÃÊiˆÌ iÀÊà œÕ`ÊLiʘiÕÌÀ>ÊœÀÊà œÕ`Ê Li>Àʜ˜iÊ«œÃˆÌˆÛiÊV >À}i° then draw the other intermediates, UÊ1Ãiʜ˜ÞÊÌ œÃiÊÀi>}i˜ÌÃÊÌ >ÌÊ>ÀiÊ>Ài>`ÞÊ«ÀiÃi˜Ì° working backward Try Problems 20.26, 20.27, 20.65

20.6 PLANNING AN ALKENE SYNTHESIS WITH A WITTIG REACTION

ÊEXAMPLE `i˜ÌˆvÞÊ STEP 1 1Ș}Ê>ÊÀiÌÀœÃÞ˜Ì ïVÊ>˜>ÞÈÃ]Ê`iÌiÀ“ˆ˜iÊÌ iÊÌÜœÊ STEP 2 œ˜Ãˆ`iÀÊ œÜÊޜÕÊ ÊÌ iÊÀi>VÌ>˜ÌÃÊޜÕÊ «œÃÈLiÊÃiÌÃʜvÊÀi>VÌ>˜ÌÃÊÌ >ÌÊVœÕ`ÊLiÊÕÃi`Ê̜ÊvœÀ“ÊÌ iÊ ÜœÕ`ʓ>ŽiÊi>V Ê«œÃÈLiÊ ÊܜՏ`ÊÕÃiÊ̜ʫÀi«>ÀiÊ 5 ÊLœ˜`° 7ˆÌ̈}ÊÀi>}i˜Ì]Ê>˜`Ê`iÌiÀ“ˆ˜iÊ ÊÌ ˆÃÊVœ“«œÕ˜`Êۈ>Ê>Ê Ü ˆV ʓiÌ œ`ʈ˜ÛœÛiÃÊÌ iʏiÃÃÊ 7ˆÌ̈}ÊÀi>V̈œ˜° ÃÕLÃ̈ÌÕÌi`Ê>ŽÞÊ >ˆ`i°

H O H PPh3 x PPh3 O X Will undergo SN2 + + more readily 2˚ 1˚ Method 1 Method 2 Alkyl halide Alkyl halide Try Problems 20.37–20.39, 20.51–20.53 20.7 PROPOSING A SYNTHESIS

STEP 1 i}ˆ˜ÊLÞÊ STEP 2 vÊÌ iÀiʈÃÊ>ÊV >˜}iʈ˜ÊÌ iÊV>ÀLœ˜ÊÎii̜˜]Ê CONSIDERATIONS >Έ˜}ÊÌ iÊvœœÜˆ˜}ÊÌÜœÊ Vœ˜Ãˆ`iÀÊ>ÊœvÊÌ iÊ q ÊLœ˜`‡vœÀ“ˆ˜}ÊÀi>V̈œ˜ÃÊ>˜`Ê ,i“i“LiÀÊÌ >ÌÊÌ iÊ`iÈÀi`Ê«Àœ`ÕVÌÊà œÕ`ÊLiÊÌ iÊ µÕiÃ̈œ˜Ã\ >ÊœvÊÌ iÊ q ÊLœ˜`‡LÀi>Žˆ˜}ÊÀi>V̈œ˜ÃÊÌ >ÌÊޜÕÊ >ÛiÊ “>œÀÊ«Àœ`ÕVÌʜvÊޜÕÀÊ«Àœ«œÃi`ÊÃÞ˜Ì iÈð i>À˜i`ÊÜÊv>À £°ÊÃÊÌ iÀiÊ>ÊV >˜}iʈ˜ÊÌ iÊ >ŽiÊÃÕÀiÊÌ >ÌÊÌ iÊÀi}ˆœV i“ˆV>ÊœÕÌVœ“iʜvÊi>V Ê V>ÀLœ˜ÊÎii̜˜¶ q ÊLœ˜`‡vœÀ“ˆ˜}ÊÀi>V̈œ˜Ãʈ˜ÊÌ ˆÃÊV >«ÌiÀ ÃÌi«ÊˆÃÊVœÀÀiVÌ° UÊÀˆ}˜>À`ÊÀi>V̈œ˜ Ó°ÊÃÊÌ iÀiÊ>ÊV >˜}iʈ˜ÊÌ iÊ Ü>ÞÃÊÌ ˆ˜ŽÊL>VŽÜ>À`Ê­ÀiÌÀœÃÞ˜Ì ïVÊ>˜>ÞÈîÊ>ÃÊÜiÊ v՘V̈œ˜>Ê}ÀœÕ«Ã¶ UÊ Þ>˜œ Þ`Àˆ˜ÊvœÀ“>̈œ˜ >ÃÊvœÀÜ>À`]Ê>˜`ÊÌ i˜ÊÌÀÞÊ̜ÊLÀˆ`}iÊÌ iÊ}>«° UÊ7ˆÌ̈˜}ÊÀi>V̈œ˜ œÃÌÊÃÞ˜Ì iÈÃÊ«ÀœLi“ÃÊ܈Ê >ÛiʓՏ̈«iÊVœÀÀiVÌÊ q ÊLœ˜`‡LÀi>Žˆ˜}ÊÀi>V̈œ˜Ãʈ˜ÊÌ ˆÃÊV >«ÌiÀ >˜ÃÜiÀÃ°Ê œÊ˜œÌÊviiÊÌ >ÌÊޜÕÊ >ÛiÊ̜Êw˜`ÊÌ iʺœ˜i»Ê UÊ >iÞiÀ‡6ˆˆ}iÀʜ݈`>̈œ˜ VœÀÀiVÌÊ>˜ÃÜiÀ° Try Problems 20.41, 20.42, 20.55, 20.58, 20.67–20.69, 20.71, 20.75 AU/ED: long page klein_c20_001-056v1.4.indd 50 30/04/10 16.47 Practice Problems 51

Note:Ê œÃÌʜvÊÌ iÊ*ÀœLi“ÃÊ>ÀiÊ>Û>ˆ>LiÊÜˆÌ ˆ˜ PRACTICE PROBLEMS WileyPLUS]Ê>˜Êœ˜ˆ˜iÊÌi>V ˆ˜}Ê>˜`ʏi>À˜ˆ˜}Ê܏Ṏœ˜°

20.44 *ÀœÛˆ`iÊ>ÊÃÞÃÌi“>̈VÊ­1* ®Ê˜>“iÊvœÀÊi>V ʜvÊÌ iÊvœ- OO œÜˆ˜}ÊVœ“«œÕ˜`Ã\ H O O (a) H O O (a) (b)

(b) F3C CF3 H3C CH3 O O

(c) H (d) 20.51 À>ÜÊ Ì iÊ «Àœ`ÕVÌÃÊ œvÊ i>V Ê 7ˆÌ̈}Ê Ài>V̈œ˜Ê LiœÜ°Ê vÊ ÌܜÊÃÌiÀiœˆÃœ“iÀÃÊ>ÀiÊ«œÃÈLi]Ê`À>ÜÊLœÌ ÊÃÌiÀiœˆÃœ“iÀÃ\Ê

O Ph Ph O O Ph P H Ph H HO ? (e) (f) Br (a)

Ph O 20.45 À>ÜÊÌ iÊÃÌÀÕVÌÕÀiÊvœÀÊi>V ÊVœ“«œÕ˜`ÊLiœÜ\ Ph P Ph Ph H ­>®Ê «Àœ«>˜i`ˆ> ? ­L®Ê {‡« i˜ÞLÕÌ>˜> (b) Ph ­V®Ê (S)‡Î‡« i˜ÞLÕÌ>˜>

­`®Ê Î]Î]x]x‡ÌiÌÀ>“iÌ Þ‡{‡ i«Ì>˜œ˜i Ph O ­i®Ê (R)‡Î‡ Þ`ÀœÝÞ«i˜Ì>˜> Ph P Ph H ­v®Ê “iÌ>‡ Þ`ÀœÝÞ>Vi̜« i˜œ˜i H ? ­}®Ê Ó]{]ȇÌÀˆ˜ˆÌÀœLi˜â>`i Þ`i (c) ­ ®Ê ÌÀˆLÀœ“œ>ViÌ>`i Þ`i CH ­ˆ®Ê (3R,4R)‡Î]{‡`ˆ Þ`ÀœÝއӇ«i˜Ì>˜œ˜i Ph 3 O Ph P Ph H 20.46 À>ÜÊ >Ê Vœ˜Ã̈ÌṎœ˜>ÞÊ ˆÃœ“iÀˆVÊ >`i Þ`iÃÊ ÜˆÌ Ê ? “œiVՏ>ÀÊ vœÀ“Տ>Ê {Hn"]Ê >˜`Ê «ÀœÛˆ`iÊ >Ê ÃÞÃÌi“>̈VÊ ­1* ®Ê (d) ˜>“iÊvœÀÊi>V ʈܓiÀ°

20.47 À>ÜÊ >Ê Vœ˜Ã̈ÌṎœ˜>ÞÊ ˆÃœ“iÀˆVÊ >`i Þ`iÃÊ ÜˆÌ Ê 20.52 À>ÜÊÌ iÊÃÌÀÕVÌÕÀiʜvÊÌ iÊ>ŽÞÊ >ˆ`iʘii`i`Ê̜ʫÀi- “œiVՏ>ÀÊvœÀ“Տ>Ê xH£ä"]Ê>˜`Ê«ÀœÛˆ`iÊ>ÊÃÞÃÌi“>̈VÊ­1* ®Ê «>ÀiÊ i>V Ê œvÊ Ì iÊ vœœÜˆ˜}Ê 7ˆÌ̈}Ê Ài>}i˜ÌÃ]Ê >˜`Ê Ì i˜Ê `iÌiÀ- ˜>“iÊvœÀÊi>V ʈܓiÀ°Ê7 ˆV ʜvÊÌ iÃiʈܓiÀÃÊ«œÃÃiÃÃiÃÊ>ÊV ˆ- “ˆ˜iÊÜ ˆV Ê7ˆÌ̈}ÊÀi>}i˜ÌÊ܈ÊLiÊÌ iʓœÃÌÊ`ˆvwVՏÌÊ̜ʫÀi«>Ài°Ê À>ˆÌÞÊVi˜ÌiÀ¶Ê Ý«>ˆ˜ÊޜÕÀÊV œˆVi\Ê

20.48 À>ÜÊ>ÊVœ˜Ã̈ÌṎœ˜>ÞʈܓiÀˆVʎi̜˜iÃÊÜˆÌ Ê“œiV- Ph Ph Տ>ÀÊvœÀ“Տ>Ê H "]Ê>˜`Ê«ÀœÛˆ`iÊ>ÊÃÞÃÌi“>̈VÊ­1* ®Ê˜>“iÊ Ph È £Ó Ph P vœÀÊi>V ʈܓiÀ°Ê Ph P (a) Ph H (b) Ph H 20.49 Ý«>ˆ˜Ê Ü ÞÊ Ì iÊ 1* Ê ˜>“iÊ œvÊ >Ê Vœ“«œÕ˜`Ê ÜˆÊ ˜iÛiÀÊi˜`ÊÜˆÌ ÊÌ iÊÃÕvwÝʺ‡£‡œ˜i°»Ê Ph Ph P 20.50 œÀÊ i>V Ê «>ˆÀÊ œvÊ Ì iÊ vœœÜˆ˜}Ê Vœ“«œÕ˜`Ã]Ê ˆ`i˜ÌˆvÞÊ (c) Ph Ü ˆV ÊVœ“«œÕ˜`ÊܜՏ`ÊLiÊiÝ«iVÌi`Ê̜ÊÀi>VÌʓœÀiÊÀ>«ˆ`ÞÊÜˆÌ Ê >ʘÕViœ« ˆi\Ê

klein_c20_001-056v1.4.indd 51 30/04/10 16.47 52 CHAPTER 20 Aldehydes and Ketones

20.53 - œÜÊ œÜÊ>Ê7ˆÌ̈}ÊÀi>V̈œ˜ÊV>˜ÊLiÊÕÃi`Ê̜ʫÀi«>ÀiÊ 20.60 *Ài`ˆVÌÊÌ iʓ>œÀÊ«Àœ`ÕV̭îÊvœÀÊi>V ÊÀi>V̈œ˜ÊLiœÜ°Ê i>V ʜvÊÌ iÊvœœÜˆ˜}ÊVœ“«œÕ˜`ðʘÊi>V ÊV>Ãi]Ê>ÃœÊà œÜÊ œÜÊ O Ì iÊ7ˆÌ̈}ÊÀi>}i˜ÌÊܜՏ`ÊLiÊ«Ài«>Ài`\Ê

1) LAH

2) H2O (a)

(a) (b) O

1) PhMgBr

2) H2O (b)

O (c) (C6H5)3P=CH2 (c) 20.54 œœÃiÊ>ÊÀˆ}˜>À`ÊÀi>}i˜ÌÊ>˜`Ê>ʎi̜˜iÊÌ >ÌÊV>˜ÊLiÊ ÕÃi`Ê̜ʫÀœ`ÕViÊi>V ʜvÊÌ iÊvœœÜˆ˜}ÊVœ“«œÕ˜`Ã\Ê O ­>®Ê ·“iÌ Þ‡Î‡«i˜Ì>˜œÊ ­L®Ê £‡iÌ ÞVÞVœ iÝ>˜œ HCN, KCN ­V®Ê ÌÀˆ« i˜Þ“iÌ >˜œÊ ­`®Ê x‡« i˜Þ‡x‡˜œ˜>˜œ (d) 20.55 9œÕÊ >ÀiÊ ÜœÀŽˆ˜}Ê ˆ˜Ê >Ê >LœÀ>̜ÀÞ]Ê >˜`Ê ÞœÕÊ >ÀiÊ }ˆÛi˜Ê Ì iÊ Ì>ÃŽÊ œvÊ Vœ˜ÛiÀ̈˜}Ê VÞVœ«i˜Ìi˜iÊ ˆ˜ÌœÊ £]x‡«i˜- 20.61 -Ì>À̈˜}Ê ÜˆÌ Ê VÞVœ«i˜Ì>˜œ˜iÊ >˜`Ê ÕȘ}Ê >˜ÞÊ œÌ iÀÊ Ì>˜i`ˆœ°Ê 9œÕÀÊ wÀÃÌÊ Ì œÕ} ÌÊ ˆÃÊ Ãˆ“«ÞÊ ÌœÊ «iÀvœÀ“Ê >˜Ê œâœ˜- Ài>}i˜ÌÃÊ œvÊ ÞœÕÀÊ V œœÃˆ˜}]Ê ˆ`i˜ÌˆvÞÊ œÜÊ ÞœÕÊ ÜœÕ`Ê «Ài«>ÀiÊ œÞÈÃÊ vœœÜi`Ê LÞÊ Ài`ÕV̈œ˜Ê ÜˆÌ Ê ]Ê LÕÌÊ ÞœÕÀÊ >LÊ ˆÃÊ ˜œÌÊ i>V ʜvÊÌ iÊvœœÜˆ˜}ÊVœ“«œÕ˜`Ã\Ê iµÕˆ««i`Ê vœÀÊ >˜Ê œâœ˜œÞÈÃÊ Ài>V̈œ˜°Ê -Õ}}iÃÌÊ >˜Ê >ÌiÀ˜>̈ÛiÊ OH COOH “iÌ œ`Ê vœÀÊ Vœ˜ÛiÀ̈˜}Ê VÞVœ«i˜Ìi˜iÊ ˆ˜ÌœÊ £]x‡«i˜Ì>˜i`ˆœ°ÊÊ œÀÊ i«]ÊÃiiÊ-iV̈œ˜Ê£Î°{Ê­Ài`ÕV̈œ˜ÊœvÊiÃÌiÀÃÊ̜Ê}ˆÛiÊ>Vœ œÃ®°Ê (a) (b) (c) 20.56 *Ài`ˆVÌÊ Ì iÊ “>œÀÊ «Àœ`ÕVÌ­Ã®Ê vÀœ“Ê Ì iÊ ÌÀi>̓i˜ÌÊ œvÊ >Vi̜˜iÊÜˆÌ ÊÌ iÊvœœÜˆ˜}ÊVœ“«œÕ˜`Ã\Ê O + + OH NH ­>®Ê Q RÊ]Ê ÎÊ]Ê­•Ó"®Ê ­L®Ê Q RÊ]Ê ÎNHÓÊ]Ê­•Ó"® 2 + + O ­V®Ê Q RÊ]ÊiÝViÃÃÊ Ì"]Ê­•Ó"®Ê ­`®Ê Q RÊ]Ê­ Î®Ó Ê]Ê­•Ó"® + + ­i®Ê Q RÊ]Ê ÓNHÓÊ]Ê­•Ó"®Ê ­v®Ê Q RÊ]Ê Ó"Ê]Ê­•Ó"® (d) (e)

­}®Ê > {]Ê i"Ê ­ ®Ê *  20.62 ÕÌ>À>`i Þ`iÊ ˆÃÊ >Ê }iÀ“ˆVˆ`>Ê >}i˜ÌÊ Ì >ÌÊ ˆÃÊ Ãœ“i- ­ˆ®Ê  ]Ê Ê ­®Ê Ì } ÀÊvœœÜi`ÊLÞÊÓ" ̈“iÃÊÕÃi`Ê̜ÊÃÌiÀˆˆâiʓi`ˆV>ÊiµÕˆ«“i˜ÌÊ̜œÊÃi˜ÃˆÌˆÛiÊ̜ÊLiÊ ­Ž®Ê ­ H ® P5    Ê ­®Ê ÊvœœÜi`ÊLÞÊ " È x Î Ó Î Ó i>Ìi`Ê ˆ˜Ê >˜Ê >Õ̜V>Ûi°Ê ˜Ê “ˆ`ÞÊ >Vˆ`ˆVÊ Vœ˜`ˆÌˆœ˜Ã]Ê }ÕÌ>À>- `i Þ`iÊi݈ÃÌÃʈ˜Ê>ÊVÞVˆVÊvœÀ“Ê­LiœÜÊÀˆ} Ì®°Ê À>ÜÊ>Ê«>ÕÈLiÊ 20.57 *Àœ«œÃiÊ >Ê «>ÕÈLiÊ “iV >˜ˆÃ“Ê vœÀÊ Ì iÊ vœœÜˆ˜}Ê “iV >˜ˆÃ“ÊvœÀÊÌ ˆÃÊÌÀ>˜ÃvœÀ“>̈œ˜\Ê ÌÀ>˜ÃvœÀ“>̈œ˜\Ê O O HO O OH O O O [H O+] [H+] 3 HH HO H EtOH Glutaraldehyde

20.58 iۈÃiÊ>˜ÊivwVˆi˜ÌÊÃÞ˜Ì iÈÃÊvœÀÊÌ iÊvœœÜˆ˜}ÊÌÀ>˜ÃvœÀ- 20.63 *Ài`ˆVÌÊ Ì iÊ “>œÀÊ «Àœ`ÕVÌ­Ã®Ê œLÌ>ˆ˜i`Ê Ü i˜Ê i>V Ê œvÊ “>̈œ˜Ê­ÀiV>ÊÌ >ÌÊÊ>`i Þ`iÃÊ>ÀiʓœÀiÊÀi>V̈ÛiÊÌ >˜ÊŽi̜˜iî\Ê Ì iÊvœœÜˆ˜}ÊVœ“«œÕ˜`ÃÊ՘`iÀ}œiÃÊ Þ`ÀœÞÈÃʈ˜ÊÌ iÊ«ÀiÃi˜ViÊ + O O œvÊÎ" \Ê

O N N H HO H O O

(a) (b) (c) 20.59 /Ài>̓i˜ÌÊ œvÊ V>ÌiV œÊ ÜˆÌ Ê vœÀ“>`i- OH Þ`iʈ˜ÊÌ iÊ«ÀiÃi˜ViʜvÊ>˜Ê>Vˆ`ÊV>Ì>ÞÃÌÊ«Àœ`ÕViÃÊ O OCH3 >Ê Vœ“«œÕ˜`Ê ÜˆÌ Ê “œiVՏ>ÀÊ vœÀ“Տ>Ê ÇHÈ"Ó°Ê À>ÜÊÌ iÊÃÌÀÕVÌÕÀiʜvÊÌ ˆÃÊ«Àœ`ÕVÌ° OH O O Catechol (d) (e)

klein_c20_001-056v1.4.indd 52 30/04/10 16.47 Practice Problems 53

20.64 `i˜ÌˆvÞÊ >Ê œvÊ Ì iÊ «Àœ`ÕVÌÃÊ vœÀ“i`Ê Ü i˜Ê Ì iÊ Vœ“- O «œÕ˜`ÊLiœÜʈÃÊÌÀi>Ìi`ÊÜˆÌ Ê>µÕiœÕÃÊ>Vˆ`\ [H+] O HS ? (g) OH N − + ( H2O) O excess H3O ? O O + N (h) [H ] H ethylene glycol ? 20.65 À>ÜÊ>Ê«>ÕÈLiʓiV >˜ˆÃ“ÊvœÀÊi>V ʜvÊÌ iÊvœœÜˆ˜}Ê (1 equivalent) ÌÀ>˜ÃvœÀ“>̈œ˜Ã\ O

+ H O H3O HCN, KCN N (i) N ? (a) H

O [H+] OH ? + O H3O N (j) H2N (b) H OH − ( H2O)

O O + 20.67 [H ] H `i˜ÌˆvÞÊÌ iÊÃÌ>À̈˜}ʓ>ÌiÀˆ>Ãʘii`i`Ê̜ʓ>ŽiÊi>V ʜvÊ HO Ì iÊvœœÜˆ˜}Ê>ViÌ>Ã\ O H2O (c) OH OO O 20.66 *Ài`ˆVÌÊÌ iʓ>œÀÊ«Àœ`ÕV̭îÊvœÀÊi>V ʜvÊÌ iÊvœœÜˆ˜}Ê Ài>V̈œ˜Ã\Ê (a) (b) O

CH3 N

NH2 (a) O OEt O [H+] ? (c) O (d) O − ( H2O) O

20.68 1Ș}Ê iÌ >˜œÊ >ÃÊ ÞœÕÀÊ œ˜ÞÊ ÃœÕÀViÊ œvÊ V>ÀLœ˜Ê >̜“Ã]Ê 1) PhMgBr (b) `iÈ}˜Ê>ÊÃÞ˜Ì iÈÃÊvœÀÊÌ iÊvœœÜˆ˜}ÊVœ“«œÕ˜`\Ê 2) H2O ? O O

CH3CO3H O (c) ?

O 20.69 *Àœ«œÃiÊ>˜ÊivwVˆi˜ÌÊÃÞ˜Ì iÈÃÊvœÀÊi>V ʜvÊÌ iÊvœœÜˆ˜}Ê ÌÀ>˜ÃvœÀ“>̈œ˜Ã\ CH3CO3H (d) ? O

O OO [H+] ? (a) (e) NH

− O ( H2O) O [H+] O O O (f) ? (b) NH2 − ( H2O) O

klein_c20_001-056v1.4.indd 53 30/04/10 16.47 54 CHAPTER 20 Aldehydes and Ketones

20.70 / iÊVœ“«œÕ˜`ÊLiœÜʈÃÊLiˆiÛi`Ê̜ÊLiÊ>ÊÜ>ëʫ iÀœ- 20.75 *Àœ«œÃiÊ>˜ÊivwVˆi˜ÌÊÃÞ˜Ì iÈÃÊvœÀÊi>V ʜvÊÌ iÊvœœÜˆ˜}Ê “œ˜i°Ê À>ÜÊÌ iʓ>œÀÊ«Àœ`ÕVÌÊvœÀ“i`ÊÜ i˜ÊÌ ˆÃÊVœ“«œÕ˜`ʈÃÊ ÌÀ>˜ÃvœÀ“>̈œ˜Ã\ Þ`ÀœÞâi`ʈ˜Ê>µÕiœÕÃÊ>Vˆ`\Ê O (a) O O

O Br (b) 20.71 *Àœ«œÃiÊ>˜ÊivwVˆi˜ÌÊÃÞ˜Ì iÈÃÊvœÀÊi>V ʜvÊÌ iÊvœœÜˆ˜}Ê ÌÀ>˜ÃvœÀ“>̈œ˜Ã\ O

O (a) (c)

MeO OMe

Br OH CN (b) Br (d)

O

NH (c) O (e)

20.72 À>ÜÊ>Ê«>ÕÈLiʓiV >˜ˆÃ“ÊvœÀÊÌ iÊvœœÜˆ˜}ÊÌÀ>˜Ã- vœÀ“>̈œ˜\ Br H O O (f) N NH2 O

[H2SO4] NH 2 [–H2O] N N

20.73 7 i˜ÊVÞVœ iÝ>˜œ˜iʈÃÊÌÀi>Ìi`ÊÜˆÌ ÊÓ"]Ê>˜ÊiµÕˆˆL- (g) ÀˆÕ“ʈÃÊiÃÌ>LˆÃ i`ÊLiÌÜii˜ÊVÞVœ iÝ>˜œ˜iÊ>˜`ʈÌÃÊ Þ`À>Ìi°Ê/ ˆÃÊ iµÕˆˆLÀˆÕ“Ê}Ài>̏ÞÊv>ۜÀÃÊÌ iʎi̜˜i]Ê>˜`ʜ˜ÞÊÌÀ>ViÊ>“œÕ˜ÌÃÊ œvÊÌ iÊ Þ`À>ÌiÊV>˜ÊLiÊ`iÌiVÌi`°Ê˜ÊVœ˜ÌÀ>ÃÌ]ÊÜ i˜ÊVÞVœ«Àœ«>- ˜œ˜iʈÃÊÌÀi>Ìi`ÊÜˆÌ ÊÓ"]ÊÌ iÊÀiÃՏ̈˜}Ê Þ`À>ÌiÊ«Ài`œ“ˆ˜>ÌiÃÊ >ÌÊiµÕˆˆLÀˆÕ“°Ê-Õ}}iÃÌÊ>˜ÊiÝ«>˜>̈œ˜ÊvœÀÊÌ ˆÃÊVÕÀˆœÕÃʜLÃiÀÛ>- (h) ̈œ˜°Ê OO

20.74 œ˜Ãˆ`iÀÊÌ iÊÌ ÀiiÊVœ˜Ã̈ÌṎœ˜>ÊˆÃœ“iÀÃʜvÊ`ˆœÝ>˜iÊ ­ {Hn"Ó®\Ê

O O O O O O 1,2-dioxane 1,3-dioxane 1,4-dioxane

"˜iʜvÊÌ iÃiÊVœ˜Ã̈ÌṎœ˜>ÊˆÃœ“iÀÃʈÃÊÃÌ>LiÊ՘`iÀÊL>ÈVÊVœ˜- `ˆÌˆœ˜ÃÊ>ÃÊÜiÊ>Ãʓˆ`ÞÊ>Vˆ`ˆVÊVœ˜`ˆÌˆœ˜ÃÊ>˜`ʈÃÊÌ iÀivœÀiÊÕÃi`Ê >ÃÊ>ÊVœ““œ˜Ê܏Ûi˜Ì°Ê˜œÌ iÀʈܓiÀʈÃʜ˜ÞÊÃÌ>LiÊ՘`iÀÊ L>ÈVÊVœ˜`ˆÌˆœ˜ÃÊLÕÌÊ՘`iÀ}œiÃÊ Þ`ÀœÞÈÃÊ՘`iÀʓˆ`ÞÊ>Vˆ`ˆVÊ Vœ˜`ˆÌˆœ˜Ã°Ê/ iÊÀi“>ˆ˜ˆ˜}ʈܓiÀʈÃÊiÝÌÀi“iÞÊ՘ÃÌ>LiÊ>˜`Ê «œÌi˜Ìˆ>ÞÊiÝ«œÃˆÛi°Ê`i˜ÌˆvÞÊi>V ʈܓiÀ]Ê>˜`ÊiÝ«>ˆ˜ÊÌ iÊ «Àœ«iÀ̈iÃʜvÊi>V ÊVœ“«œÕ˜`°

klein_c20_001-056v1.4.indd 54 30/04/10 16.47 Integrated Problems 55

INTEGRATED PROBLEMS

20.76 œ“«œÕ˜`Ê Ê >ÃÊ “œiVՏ>ÀÊ vœÀ“Տ>Ê ÇH£{"Ê >˜`Ê 20.78 `i˜ÌˆvÞÊ Ì iÊ ÃÌÀÕVÌÕÀiÃÊ œvÊ Vœ“«œÕ˜`ÃÊ Ê ÌœÊ Ê LiœÜ]Ê Ài>VÌÃÊÜˆÌ ÊÜ`ˆÕ“ÊLœÀœ Þ`Àˆ`iʈ˜Ê“iÌ >˜œÊ̜ÊvœÀ“Ê>˜Ê>Vœ œ°Ê >˜`Ê Ì i˜Ê ˆ`i˜ÌˆvÞÊ Ì iÊ Ài>}i˜ÌÃÊ Ì >ÌÊ V>˜Ê LiÊ ÕÃi`Ê ÌœÊ Vœ˜ÛiÀÌÊ / iÊ £Ê ,ÊëiVÌÀՓʜvÊVœ“«œÕ˜`ÊÊiÝ ˆLˆÌÃʜ˜ÞÊÌܜÊÈ}- VÞVœ iÝi˜iʈ˜ÌœÊVœ“«œÕ˜`Ê Êˆ˜ÊÕÃÌʜ˜iÊÃÌi«° ˜>Ã\Ê>Ê`œÕLiÌÊ­IÊrÊ£Ó®Ê>˜`Ê>ÊÃi«ÌiÌÊ­IÊrÊÓ®°Ê/Ài>̈˜}ÊVœ“«œÕ˜`Ê + Ê ÜˆÌ Ê £]ӇiÌ >˜i`ˆÌ ˆœÊ ­- Ó Ó-®Ê vœœÜi`Ê LÞÊ ,>˜iÞÊ H3O H2CrO4 ˜ˆVŽiÊ}ˆÛiÃÊVœ“«œÕ˜`Ê °Ê A B [H+] ­>®Ê œÜʓ>˜ÞÊÈ}˜>ÃÊ܈Ê>««i>Àʈ˜ÊÌ iÊ£Ê ,ÊëiVÌÀՓʜvÊ NH2NH2 Vœ“«œÕ˜`Ê ¶ (–H2O) £Î KOH / H O ­L®Ê œÜʓ>˜ÞÊÈ}˜>ÃÊ܈Ê>««i>Àʈ˜ÊÌ iÊ Ê ,ÊëiVÌÀՓʜvÊ D 2 C Vœ“«œÕ˜`Ê ¶ heat ­V®Ê iÃVÀˆLiÊ œÜÊޜÕÊVœÕ`ÊÕÃiÊ,ÊëiVÌÀœÃVœ«ÞÊ̜ÊÛiÀˆvÞÊÌ iÊ Vœ˜ÛiÀȜ˜ÊœvÊVœ“«œÕ˜`ÊÊ̜ÊVœ“«œÕ˜`Ê ° 20.79 `i˜ÌˆvÞÊÌ iÊÃÌÀÕVÌÕÀiÃʜvÊVœ“«œÕ˜`ÃÊÊÌœÊ ÊLiœÜ\ O 20.77 1Ș}Ê Ì iÊ ˆ˜vœÀ“>̈œ˜Ê «ÀœÛˆ`i`Ê LiœÜ]Ê `i`ÕViÊ Ì iÊ 1) Br Mg H H ÃÌÀÕVÌÕÀiÃʜvÊVœ“«œÕ˜`ÃÊ]Ê ]Ê ]Ê>˜`Ê \Ê 2 A BC FeBr3 2) H2O

A D PCC 1) O 1) EtMgBr (C10H12) 3 (C H O) HO OH 2) H O 11 16 E D 2) DMS 2 + − [H ], H2O

C 20.80 ˜Ê>`i Þ`iÊÜˆÌ Ê“œiVՏ>ÀÊvœÀ“Տ>Ê {HÈ"ÊiÝ ˆLˆÌÃÊ •£ (C9H10O) N >˜Ê,ÊÈ}˜>Ê>ÌʣǣxÊV“ ° B ­>®Ê *Àœ«œÃiÊÌܜʫœÃÈLiÊÃÌÀÕVÌÕÀiÃÊÌ >ÌÊ>ÀiÊVœ˜ÃˆÃÌi˜ÌÊÜˆÌ Ê + AlCl3 [H ], (CH3)2NH Ì ˆÃʈ˜vœÀ“>̈œ˜° £Î (–H2O) ­L®Ê iÃVÀˆLiÊ œÜÊޜÕÊVœÕ`ÊÕÃiÊ Ê ,ÊëiVÌÀœÃVœ«ÞÊÌœÊ `iÌiÀ“ˆ˜iÊÜ ˆV ʜvÊÌ iÊÌܜʫœÃÈLiÊÃÌÀÕVÌÕÀiÃʈÃÊVœÀÀiVÌ°Ê

20.81 Ê Vœ“«œÕ˜`Ê ÜˆÌ Ê “œiVՏ>ÀÊ vœÀ“Տ>Ê ™H£ä"Ê iÝ ˆLˆÌÃÊ >Ê ÃÌÀœ˜}Ê Ãˆ}˜>Ê >ÌÊ £ÈnÇÊV“•£Êˆ˜ÊˆÌÃÊ,ÊëiVÌÀՓ°Ê/ iÊ£Ê>˜`Ê£Î Ê ,ÊëiVÌÀ>ÊvœÀÊÌ ˆÃÊVœ“«œÕ˜`Ê>ÀiÊà œÜ˜Ê LiœÜ°Ê`i˜ÌˆvÞÊÌ iÊÃÌÀÕVÌÕÀiʜvÊÌ ˆÃÊVœ“«œÕ˜`°Ê

3 Proton NMR

2 2 3

978 654321 Chemical Shift (ppm)

Carbon 13 NMR

128.3

132.5 127.7 31.3 7.9 199.9 136.7

200 180 160 140 120 100 80 60 40 20 0 Chemical Shift (ppm)

klein_c20_001-056v1.4.indd 55 30/04/10 16.47 56 CHAPTER 20 Aldehydes and Ketones

20.82 Ê Vœ“«œÕ˜`Ê ÜˆÌ Ê “œiVՏ>ÀÊ vœÀ“Տ>Ê £ÎH£ä"Ê «Àœ`ÕViÃÊ >Ê ÃÌÀœ˜}Ê Ãˆ}˜>Ê >ÌÊ £ÈÈäÊV“•£Êˆ˜ÊˆÌÃÊ,ÊëiVÌÀՓ°Ê/ iÊ£Î Ê ,ÊëiVÌÀՓÊvœÀÊÌ ˆÃÊVœ“«œÕ˜`ʈÃÊà œÜ˜ÊLiœÜ°Ê `i˜ÌˆvÞÊÌ iÊÃÌÀÕVÌÕÀiʜvÊÌ ˆÃÊVœ“«œÕ˜`°Ê

Carbon 13 NMR 130.0

128.3 132.4 137.5 196.7

200 190 180 170 160 150 140 130 120 110 100 Chemical Shift (ppm)

£ 20.83 ʎi̜˜iÊÜˆÌ Ê“œiVՏ>ÀÊvœÀ“Տ>Ê ™H£n"ÊiÝ ˆLˆÌÃʜ˜Þʜ˜iÊÈ}˜>Êˆ˜ÊˆÌÃÊ Ê ,Ê Ã«iVÌÀՓ°Ê*ÀœÛˆ`iÊ>ÊÃÞÃÌi“>̈VÊ­1* ®Ê˜>“iÊvœÀÊÌ ˆÃÊVœ“«œÕ˜`°Ê

CHALLENGE PROBLEMS

20.84 À>ÜÊ>Ê«>ÕÈLiʓiV >˜ˆÃ“ÊvœÀÊi>V ʜvÊÌ iÊvœœÜˆ˜}ÊÌÀ>˜ÃvœÀ“>̈œ˜Ã\

O H H O N + N H3O + HO H + H3CCH3 (a) O

OCH + H 3 H3O (b)

O O [H SO ] + 2 4 NH2NH2 NH (c) H H N O O [H SO ] OH 2 4 (d) O O OH

OCH3 OCH3

O [H2SO4] O (e) OH OH

OCH3

O O [TsOH] (f) HO OH

20.85 1˜`iÀÊ>Vˆ`‡V>Ì>Þâi`ÊVœ˜`ˆÌˆœ˜Ã]ÊvœÀ“>`i Þ`iÊ«œÞ“iÀˆâiÃÊ̜ʫÀœ`ÕViÊ>ʘՓ- O O + LiÀÊ œvÊ Vœ“«œÕ˜`Ã]Ê ˆ˜VÕ`ˆ˜}Ê «>À>vœÀ“>`i Þ`i°Ê À>ÜÊ >Ê «>ÕÈLiÊ “iV >˜ˆÃ“Ê vœÀÊ Ì ˆÃÊ [H3O ] ÌÀ>˜ÃvœÀ“>̈œ˜\ H H O O

Paraformaldehyde

klein_c20_001-056v1.4.indd 56 30/04/10 16.47